X



トップページ数学
1002コメント374KB
分からない問題はここに書いてね439
レス数が1000を超えています。これ以上書き込みはできません。
0003132人目の素数さん垢版2017/12/21(木) 16:04:22.41ID:Yt9gbXu3
受験数学は全然できなくて無問題
あんなのは所詮公式と解法パターンの丸暗記競争だから
ルービックキューブと一緒でやり方知ってりゃ10秒で解法が組み上がる
大学行ったら数学や物理は勿論、化学だって高校数学なんか全く役に立たないよ
そうはいっても国公立の理系は少なくともセンター数学を受けないと入れない
国立、特に下位駅弁からは同レベルの理系単科私大等と比べて突出した才能が出ない一因でもある
俺も文系からの理系学部進学組みだけど高校で理系だった奴は暗記重視で本質を理解している奴はいなかった印象がある
何でも覚えようとしちゃうのね。理解しようとしないで
今でも私大なら理系学部で入試に数学を課してない所があるはず(理由は前述のとおり)
但し記述式の国語があるから地頭勝負になるけどね
数学や理科といった暗記科目で挽回の効く東大理系前期なんかよりある意味難関
0005132人目の素数さん垢版2017/12/21(木) 16:20:47.53ID:Oq94sUWc
>>1 もうお前に用はない

  ○
 く|)へ
  〉 ヾ○シ
 ̄ ̄7 ヘ/
  /  ノ
  |
 /
`|
0006132人目の素数さん垢版2017/12/21(木) 16:35:01.91ID:wwmE25ut
dt/dx =2x^2 / tx-t^2
同次式の微分方程式の一般解を求める問題です。

写真のところまで解いたのですが左辺の積分がうまくできません。
どなたか教えてください


https://i.imgur.com/uMPcPQb.jpg
0008132人目の素数さん垢版2017/12/21(木) 19:33:08.10ID:yuoQ4qW4
x= -t+t^2/2 C1+/- (1/2)√(C1) t ^(3/2) √(-4 + C1t)
0009132人目の素数さん垢版2017/12/21(木) 20:37:45.26ID:DNai/QEn
abc予想にまつわる傑作問題です。
cを自然数とする。
a+b<cを満たす自然数a,bの組(a,b)の数をf(c)とし、a^2+b^2<c^2を満たす自然数a,bの組(a,b)の数をg(c)とする。
このとき、極限
lim[c→∞] {g(c)/f(c)}c^t
が0でない定数に収束するような実数tを求めよ。
0011132人目の素数さん垢版2017/12/21(木) 23:25:00.32ID:WCd1YHi3
(√5+√7)^2018の小数第100位の値は何か
全く手をつけられません
出来れば解答も添えて考え方を教えてください
0012132人目の素数さん垢版2017/12/22(金) 00:00:16.31ID:rrtMEKVY
・0 < (√7-√5)^2018 
・(√7-√5)^2018 << 10^(-100)
・(√7+√5)^2018 + (√7-√5)^2018  は整数
らから結論は9
0014132人目の素数さん垢版2017/12/22(金) 07:33:01.19ID:6HAjmE3C
(W,≦)を整列集合としてx∈Wに関する命題P(x)が与えられている
このとき、P(minW)が真かつ
Wのxによる切片の要素yに対してP(y)が真ならばP(x)も真であることが示されたら
任意のxについてP(x)が真になることを示して下さい
0016132人目の素数さん垢版2017/12/22(金) 13:46:49.33ID:QFHukmZd
https://ja.m.wikipedia.org/wiki/接ベクトル空間

『m 次元 Cr 級多様体 M と、その中の Cr 級曲線φ :(− ε, ε) → Mを考え φ(0) = p ∈ M とする。
p を含む座標近傍 (U;x1,…,xm) において φ(t) = ( x1(t),…,xm(t)) を t で微分して、...』

とありますが、x1,...,xmの定義域はMの開集合では無いのでしょうか?
0017132人目の素数さん垢版2017/12/22(金) 13:49:13.99ID:tPyfwdLW
1でない正の数a,b,cがこの順に等比数列であるとき、
a<b<c または a>b>c の順になるからこの順に等差数列である

これどういうことですか?
0018132人目の素数さん垢版2017/12/22(金) 13:53:47.22ID:QFHukmZd
>>16
自己解決しました
0019132人目の素数さん垢版2017/12/22(金) 14:35:28.03ID:54XXYVoF
>>12
何をやってるのか全く見えないので説明いれて頂けませんか?
お願いします
0020132人目の素数さん垢版2017/12/22(金) 15:04:14.99ID:L+4ikRzW
>>19
(√5+√7)^2018は整数よりほんの少しだけ小さい数で、その誤差が10のー100乗より明らかに小さい、よって10のー100乗の位は9、と言ってる模様
解答と考え方とは示されてると思う
0021132人目の素数さん垢版2017/12/22(金) 15:11:39.12ID:tPyfwdLW
暇だったので三角関数をいじっていたんですが、どうしても意味のない式しか生まれない。やっぱりサインコサインタンジェントの定義自体が三角関数の大きな要素なので、新しい有用な式はもう生まれないんでしょうか。
加法定理は一般的な三角形において二つの角の大きさを足したサイン・コサインはもう一つの角のサイン・コサインを表す、というものしか結局思いつかなかったですし、それもあっているかどうかもわからないので...もし合ってても意味がない...
0022132人目の素数さん垢版2017/12/22(金) 15:14:13.35ID:RhSNwWfd
>>19
(√7+√5)^2018 - (√7-√5)^2018 - @をまず考える
二項定理から、kを奇数として2018Ck(√7)^(2018-k)(√5)^kの部分が打ち消し合って0となる
よって@の値は整数Nであることがわかる
また、√7-√5~0.4096とすれば

(√7+√5)^2018 = N- (√7-√5)^2018 ~ N- (0.4096)^2018

(0.4096)^2018は小数第780位辺りで初めて0でない数字が現れる
よって、(√7+√5)^2018の整数部分はN-1、少数部分は第780位辺りまで9が続くことが分かる
0024132人目の素数さん垢版2017/12/22(金) 15:27:59.41ID:RhSNwWfd
>>21
どんな式が生まれたのか教えてくれ
一見意味はないかもしれないが、逆にその式が活かせるものを探すことが研究になる
0025132人目の素数さん垢版2017/12/22(金) 15:42:38.16ID:tPyfwdLW
>>24
θ、ℓ、aを三角形の辺の長さ、Rを三角形の外接円とすると、こんんな式が成り立つ。
でも、ややこしい上に辺と辺のサインを求めたほうが楽っていう。
https://i.imgur.com/9CNG5Kj.jpg
0026132人目の素数さん垢版2017/12/22(金) 15:46:51.74ID:PuT/uoxa
>>25
その文字の置き方はやめたほうがいいです
abcと書きましょう
ギリシャ文字は通常は角度を表します
0027132人目の素数さん垢版2017/12/22(金) 15:53:41.94ID:HVJnm64E
角度の2乗から長さの2乗引いてるし珍しい式だな〜と思ってたら、θが長さとは意表を突きすぎだろう…
ある程度常識的な文字の使い方をしないと、中身の信頼性まで失われると思う(読んでももらえない)よ
0028132人目の素数さん垢版2017/12/22(金) 16:17:07.58ID:tPyfwdLW
ああ、すいません。人に見せる予定のない紙だったので....今度からそうします。
0030132人目の素数さん垢版2017/12/22(金) 16:55:42.97ID:tPyfwdLW
あ、一応自明な式ではないのか...でも使い道はないな、間違いなく
0032132人目の素数さん垢版2017/12/22(金) 17:44:01.89ID:tPyfwdLW
>>31
私はサインの二乗とコサインの二乗を足したら1になる式に正弦定理(θ)と余弦定理(θ)を代入してゴリゴリ計算しましたが...

そんなことしなくても導出できると?
0033132人目の素数さん垢版2017/12/22(金) 17:53:40.37ID:Q7HPh6Gr
a^2b^2c^2+R^2(a^2-b^2-c^2)^2=
    (4 R^2)b^2c^2(((a/2)/R)^2+cos(α)^2) <-- 余弦公式
a^2=b^2+c^2-2bc cos(α)
    =4R^2 b^2c^2

note ((a/2)/R)^2+cos(α)^2= cos((Piー2α)/2)^2+cos(α)^2=sin()^2+cos()^2=1
三角形ABCと外接円半径と辺の角の図をかく
0034132人目の素数さん垢版2017/12/22(金) 18:45:38.06ID:rrtMEKVY
>>32
S=(1/2)bc*sin(A)=(1/2)bc*(a/(2R))=abc/(4R)

(abc)^2=16R^2*S^2=16R^2*s(s-a)(s-b)(s-c)
=R^2(a+b+c)(-a+b+c)(a-b+c)(a+b-c)
=R^2(-a^2+(b+c)^2)(a^2-(b-c)^2)
=R^2(-a^2+b^2+c^2+2bc)(a^2-b^2-c^2+2bc)
=R^2{-(-a^2+b^2+c^2)^2+(2bc)^2}

∴ a^2 b^2 c^2+R^2(a^2-b^2-c^2)^2=4R^2 b^2 c^2
0035132人目の素数さん垢版2017/12/22(金) 20:28:40.61ID:tPyfwdLW
なんどもすいません


不定積分log(x+2)dxを求めよという問題で解答では部分積分法使ってたんですが自分は画像の公式をf(x)=logxとして考えました。
この公式って記述無しに使ってokですか?というか教科書にありますか?
また、それをした所、(x+2)log(x+2)-x-2+Cとなったのですが-2+Cをそのまんま積分定数とするときどんな記述をすればいいでしょうか?


https://i.imgur.com/846ykUP.jpg
0036132人目の素数さん垢版2017/12/22(金) 21:29:04.65ID:RhSNwWfd
>>35
-2+Cを改めてC(同じ文字が気になるなら別の文字)と置く
∫ log(x) dx = xlog(x)-x+C

もしくは(普通は) ∫ (x+2)’ log(x+2) dx として処理する

公式の右辺は微分すればf(x)が得られるのは明らか
0037132人目の素数さん垢版2017/12/22(金) 22:28:28.15ID:ulRLSBdQ
If p is a prime number and n is a member of positive numbers,then any p(x) which is a irreducible polynomial of degree n in Z/pZ[x] divides x^(p^n)-x in Z/pZ[x].
But I cannot prove this theorem!!
0038132人目の素数さん垢版2017/12/22(金) 22:50:57.89ID:W3iDm4ES
>>21
は?
0039132人目の素数さん垢版2017/12/23(土) 00:39:53.14ID:AJG6WhTU
理論物理学 難易度総合ランキング

75  [M理論]、[Dブレーン]、{統一場理論}
74  [超弦理論]
73  [ループ量子重力理論]
72  {超重力理論}
71  格子ゲージ理論
70  インフレーション宇宙理論
69  [超対称性理論]、[一般相対論的量子力学(Hawking)]
68  [大統一理論]、{弦理論}
67  Weinberg-Salam理論
66  量子色力学(Quark)
64  素粒子物理学、Yan-Mills理論
63  繰り込み理論・繰り込み群、アノマリー
62  宇宙物理学、{Kalza-Klein理論}
61  一般相対性理論、強い相互作用、弱い相互作用
60  相対論的量子力学、量子電磁気学
59  原子核物理学、放射線物理学、宇宙線物理学
58  量子光学、超低温物理学、プラズマ物理学
57  物性物理学、固体物理学、流体力学
56  量子力学、生物物理学、化学物理学
55  古典量子力学
50  特殊相対性理論
45  統計力学
40  解析力学、電磁気学、光学
35  力学
30  物理数学
25  高校物理、ブルーバックス
0   [トンデモ理論]
-∞  [新興宗教]
0040132人目の素数さん垢版2017/12/23(土) 01:07:32.31ID:UW3Uffha
>>21
>一般的な三角形において二つの角の大きさを足したサイン・コサインはもう一つの角のサイン・コサインを表す
正弦はそうね。余弦は符号が反転する。
三角関数は複素数と組み合わせたりフーリエ変換で使うときに違う面白味が出るので、ぜひそちらを学んでほしいところ。
0041132人目の素数さん垢版2017/12/23(土) 01:58:25.46ID:gDyMI1rU
>>11 >>19

a_n ={(√7 +√5)^(2n)+(√7 -√5)^(2n)}/(2^n)
  =(6+√35)^n +(6-√35)^n
とおくと漸化式が
a_{n+1}= 12a_n - a_{n-1},
a_0=2,a_1=12
となるので、a_n は偶数。

なお、a_n = 2 cosh(n c)は使いません。。。
0042132人目の素数さん垢版2017/12/23(土) 02:01:08.50ID:gDyMI1rU
[前スレ.988]

・長寿ランキング of 他分野

107歳310日 平櫛田中 (1872/02/23〜1979/12/30) 彫刻家
0043132人目の素数さん垢版2017/12/23(土) 08:48:32.05ID:nsgUiKTK
2^24×3^36×11^12を2進法で表すと、末尾には0が連続して24個並ぶ。
3^36×11^12が莫大な数だからでしょうか?
2^24×3^4×11^3を2進法で表すと、そうはいかないでしょうか?
0045132人目の素数さん垢版2017/12/23(土) 10:56:03.57ID:UW3Uffha
>>43
10^24×3^36×11^12を十進法で表すと、末尾には0が連続して24個並ぶ。
3^36×11^12が莫大な数だからでしょうか?
10^24×3^4×11^3を十進法で表すと、そうはいかないでしょうか?
0048132人目の素数さん垢版2017/12/23(土) 13:26:36.92ID:nsgUiKTK
>>45
3=11(2)
11=1011(2)
11(2)と1011(2)は何乗しても末尾に0は付きませんね。
大変よくわかるヒントをありがとうございました。
0049132人目の素数さん垢版2017/12/23(土) 14:56:11.90ID:GaEzdGdo
学校の課題で問題が出されたのですが、全くわかりません。
問題は、
問1 平均値μ=2、および標準偏差σ=2の正規分布に従う確率変数を考える。このとき、この確率変数が次の区間に含ま
れる確率を小数第4位まで計算しなさい。
1 (4, ∞)
2 (-∞, 2.7)
3 (0.88, 5.6)
4 (1.46, 3.24)

問2 ある検問所で記録された車のスピードのデータによると、そこを通過する車は平均時速61.6km、標準偏差7.0kmで、だいたい正規分布に従っている。このとき、次の割合を100分率(パーセント)で小数第1位まで計算しなさい。
1 時速70kmをこえている車は全体の○%である
2 時速49kmよりも遅い車は全体の○%である
3 時速56kmから時速63kmまでの車は全体の ○%である

助けてください
問題解法見てもさっぱりなので答えだけでお願いします
0051132人目の素数さん垢版2017/12/23(土) 15:12:36.31ID:gM/Fpbbq
一般相対性理論を本当に理解している者は、世界に3人といない。
0052132人目の素数さん垢版2017/12/23(土) 15:22:40.41ID:GaEzdGdo
2)ある検問所で記録されたスピードのデータによると、そこを通過する車は平均時速60.5km、標準偏差7.4kmで、大体正規分布に従っている。
このとき
(a)時速70kmを超えている車は全体の何%か
(b)時速48kmよりも遅い車は全体の何%か
(c)時速56kmから時速64kmまでの車は全体の何%か

の答えが(a)10% (b)4.6% (c)41.0%なんだけど
こんだけ近い数字でこんなに答え変わることってあるの?
0054132人目の素数さん垢版2017/12/23(土) 15:36:01.08ID:7ysHe5HG
もし、Fが Complete Ordered Field で x^2=2 && x>0 なるが存在するならが、
√2 は存在する。

の証明がうまくできません。
よろしくお願いします。
0056132人目の素数さん垢版2017/12/23(土) 15:44:07.32ID:GaEzdGdo
>>53
問1の1と2だけでいいから教えてくれよ
他はわかったから
0057132人目の素数さん垢版2017/12/23(土) 15:51:04.70ID:7ysHe5HG
>>52

∫1/(2 Pi s^2)^(1/2)exp[-(1/2) (x-m)^2/s)

で計算するt
(1)0.0996 (10%)
(2)0.04556 (4。6%)
(3)0.41032  (41%)

で合い過ぎなんだが
0058132人目の素数さん垢版2017/12/23(土) 15:52:58.35ID:OPsKXeDc
どうしたら真理に近づくことができるのでしょうか?
0059132人目の素数さん垢版2017/12/23(土) 15:54:57.21ID:GaEzdGdo
>>57
うん、俺が言いたいのは>>50のレスの答えがそのあってる問題と離れすぎてるからおかしいなってなったんだわ
0060132人目の素数さん垢版2017/12/23(土) 15:57:43.74ID:GaEzdGdo
∞っていうのはどういう事なの?
4<Xって解釈であってる?
0061132人目の素数さん垢版2017/12/23(土) 17:18:47.70ID:otikgY+E
行列式についてのラプラスの定理って何の役に立つのでしょうか?
0062132人目の素数さん垢版2017/12/23(土) 18:52:32.21ID:7ysHe5HG
>>55 実数の構成は全順序から導かれるものではないですよ

Complete Ordered Field :An ordered field that satisfies the least upper bound axiom is called
0063132人目の素数さん垢版2017/12/23(土) 18:57:59.92ID:gjIkOkZd
超一流の数学者と超一流の法学者ってどっちの方が頭が良いの?
0064132人目の素数さん垢版2017/12/23(土) 19:07:58.77ID:gjIkOkZd
なぜ何もないのではなく、何かがあるのでしょうか?
0065132人目の素数さん垢版2017/12/23(土) 19:12:44.54ID:4e98Asir
質問の筋が悪過ぎ
0066132人目の素数さん垢版2017/12/23(土) 22:19:51.36ID:a/e7MMcn
2CIsE1Rrさん、用語並べれば私を騙せると思ったかもしれませんけど、私は騙されませんからね(笑)?

あなたは超準解析がわからなかった、それをないことにすることはできません
0067132人目の素数さん垢版2017/12/23(土) 22:48:21.32ID:gGCV2aUH
>>54
定義ジャン
0068132人目の素数さん垢版2017/12/23(土) 22:51:13.70ID:paTeZ+TX
世界一天才が多い国はどこですか?
アメリカかな?
0069132人目の素数さん垢版2017/12/23(土) 23:19:06.41ID:gGCV2aUH
>>68
なんか面白いって思ってるのかなあ
0070132人目の素数さん垢版2017/12/23(土) 23:27:06.51ID:JamHfM57
■モンティホール問題(空箱とダイヤ)

このゲームができるのは1回だけです

外からは中が見えない空箱100個の中のひとつに
ダイヤモンドを1個入れます

その中から1個の箱を選びます

98個の空箱を取り除きます

最後に残った2個の箱の中から1個の箱を選びます

ダイヤモンドが当たる確率は何%でしょうか?
0073132人目の素数さん垢版2017/12/23(土) 23:44:54.92ID:paTeZ+TX
東大理学部数学科でダントツの人と、東大工学部航空宇宙工学科でダントツの人はどっちの方が頭が良いのでしょうか?
0074132人目の素数さん垢版2017/12/23(土) 23:53:48.01ID:paTeZ+TX
全宇宙全世界全次元全階層をくまなく探査したら少しは真理に近づけるような発見とかがあるのでしょうか?
0075132人目の素数さん垢版2017/12/23(土) 23:55:21.91ID:paTeZ+TX
全宇宙全世界全次元全階層をくまなく探査したいのですがどうすれば良いですか?
最低でもワープ技術が無いと話になりませんか?
0079132人目の素数さん垢版2017/12/24(日) 14:03:22.34ID:B54w+L/c
いるわ、また荒らしかしら、頭悪い

(回文)
0081132人目の素数さん垢版2017/12/24(日) 15:35:03.06ID:MV1bjGnQ
山本修身著『よくわかるトポロジー』を読んでいます。

「位相空間 R の部分集合 Q の境界を求めよ」

という問題の解答が Q となっています。

ひどい本ですね。
0082132人目の素数さん垢版2017/12/24(日) 16:58:15.88ID:VYbr+C8Q
■モンティホール問題(空箱とダイヤ)

このゲームができるのは1回だけです

ダイヤモンド1個を外からは中が見えない空箱100個の
中のどれかひとつに入れます

その中から1個の箱を選びます

98個の空箱を取り除きます

最後に残った2個の箱の中から1個の箱を選びます

ダイヤモンドが当たる確率は何%でしょうか?
0083132人目の素数さん垢版2017/12/24(日) 17:31:02.28ID:8R2APNsR
abc予想にちなんで出題された問題です。分かりません。

正整数a,b,cはa+b=cを満たし、a^2+ab+b^2はcの倍数である。
ここで、正整数kを素因数分解したときに現れる素因数の個数をd(k)とする。例えばk=45のとき、45=3^2*5であるから、現れる素因数は3と5の2個で、したがってd(k)=2である。
a,bが動くとき、d(c)の最小値を求めよ。またその最小値を与えるcがどのような数であるか述べよ。
0084132人目の素数さん垢版2017/12/24(日) 17:48:43.53ID:MV1bjGnQ
山本修身著『よくわかるトポロジー』を読んでいます。

M^i = M ⇔ M は開集合

を証明せよ。

これは明らかですね。

それにもかかわらず、解答で、ヘンテコな長い議論をしています。
0087132人目の素数さん垢版2017/12/24(日) 18:04:41.48ID:LJXBwhk0
>>86
これも難問です
よろしくお願いします

ペアノ算術を含む任意の無矛盾な公理系に対し、あるモデルM,Nおよび論理式φが存在して、M|=φかつN|≠φとできることを示せ、という問題がわかりません
0088132人目の素数さん垢版2017/12/24(日) 18:10:03.70ID:8R2APNsR
>>87
お前はこのスレに居ついて楽しいか?今夜抱く女どころか一緒に食事する相手もいないだろ?
0089132人目の素数さん垢版2017/12/24(日) 18:13:08.13ID:LJXBwhk0
>>88
あなたも、このスレに張り付いて自分よりレベルの低い人の質問を馬鹿にするような書き込みをしてますね
クリボッチとかいうやつなんですか?
0090132人目の素数さん垢版2017/12/24(日) 18:25:55.40ID:MV1bjGnQ
山本修身著『よくわかるトポロジー』を読んでいます。

読めば読むほどひどい本です。

山本さんは、工学部出身なんですね。

なぜ素人がトポロジーの本など書いたのでしょうか?
0091132人目の素数さん垢版2017/12/24(日) 18:30:26.22ID:LJXBwhk0
よくわかる、というのは怪しい本ですよ基本
証明が助長とかはともかく、答えが間違ってるのはいけませんね
0093132人目の素数さん垢版2017/12/24(日) 18:37:17.28ID:FU5BkpdK
宇宙はどこにあるのでしょうか?
0095132人目の素数さん垢版2017/12/24(日) 19:36:49.19ID:TRpWg4XU
宇宙は宇宙空間の元である
0096132人目の素数さん垢版2017/12/24(日) 19:57:21.13ID:5JQxpwTX
宇宙空間の元はどこにあるのでしょうか?
0097132人目の素数さん垢版2017/12/24(日) 20:40:03.19ID:qfIuQhn6
3logx+2logy+5logz+log(22-x-y-z)の極値を求めろというのがわかりません
0098132人目の素数さん垢版2017/12/24(日) 22:31:44.31ID:St+DBIAQ
長谷川浩司さんの線型代数p3より

『x_1とx_2の連立一次方程式(1)は行列とベクトルを使って書けるのであった。
......
じつはちょっと考えると、一次式(1)で表される、と言うかわりに線型性を持つと言っても同じであることが分かる。』

と書かれているのですが線型性と連立一次方程式にはどのような関係があるのでしょうか
0099132人目の素数さん垢版2017/12/24(日) 22:43:47.90ID:gNqMtSOQ
連立方程式が与えられれば行列を用いてAx=bと書くことができる
逆にAx=bが与えられれば成分比較して連立方程式が書ける
0100132人目の素数さん垢版2017/12/24(日) 22:45:09.48ID:cmcygqx4
>>97

相乗-相加平均で
(x/3)^3(y/2)^2(z/5)^5(22-x-y-z)≦ 2^11,

(x^3)(y^2)(z^5)(22-x-y-z)≦{(3^3)(2^2)(5^5)}(2^11),

{2log(2)+ 3log(3)+ 5log(5)}+ 11log(2)= 8.83960372947 (極大)

等号成立は x/3 = y/2 = z/5 = 22-x-y-z,
(x,y,z)=(6,4,10)
0101132人目の素数さん垢版2017/12/24(日) 22:52:48.83ID:St+DBIAQ
>>99
連立方程式⇔行列を使った式
ってことですよね

長谷川さんがそれを言ってるようには解釈できないのですが...
0103132人目の素数さん垢版2017/12/24(日) 23:41:30.53ID:TfglMj9F
b=1であるので、0<k<1
P, Qの座標はそれぞれ(k, -k+1), (k, k^3-k^2-k+1)
Pのy座標からQのy座標を引いたものがPQの長さであり、kの関数で表される
0104132人目の素数さん垢版2017/12/24(日) 23:48:11.00ID:IlXtT6jW
ばかはしねばなおりますか?

証明してください。
0105132人目の素数さん垢版2017/12/24(日) 23:58:28.97ID:gNqMtSOQ
>>101
なら本持ってないから知らんわ
0106132人目の素数さん垢版2017/12/25(月) 00:07:29.76ID://NQaQqy
(1)
f'(x)=3x^2+2ax+a
f'(2)=5a+12=7
∴a=-1
(2)
f(x)=x^3-x^2-x+1
f'(x)=3x^2-2x-1
y=(3t^2-2t-1)(x-t)+t^3-t^2-t+1=(3t^2-2t-1)x-2t^3+t^2+1
1=-2t^3+t^2+1⇔tt(2t-1)=0⇔t=0,1/2
(3)
l : y=-x+1
-b+1=b^3-b^2-b+1⇔bb(b-1)=0
∴b=1
PQ=|(-k+1)-(k^3-k^2-k+1)|=|-k^3+k^2|
g(k)=-k^3+k^2=kk(1-k)
g'(k)=-3k^2+2k=k(2-3k)
0<k<1, MaxPQ=Max(|g(k)|)=Max(g(k))=g(2/3)=4/27
0107132人目の素数さん垢版2017/12/25(月) 00:14:31.74ID://NQaQqy
"30%" means that an average student couldn't solve (3)?
hmm...
0108132人目の素数さん垢版2017/12/25(月) 00:51:00.77ID:qSFB4OSb
長谷川線型代数、それちゃんと書き写してみ?
自分勝手に適当に省略して書いてるやろ

すくなくとも旧版(赤)はそういう記述ではなく、省略部分には行列の話がある。
文を読めば、後半で出てくる「一次式(1)」は連立方程式としての関係式を言っているのでは
ないこともわかるだろう。
ここで言ってるのは、連立方程式も見方を変えれば行列を使って表すことのできる(一種の)比例関係
になるってことだ。

ついでにいうと、長谷川線型代数は、元ネタが工学部用講義プリントだから、ガチガチの数学書ではないぞ。
(しかも引用部分って一番最初の導入部のお話部分だろ)
重箱の隅をつつくのなら、齋藤とか佐武あたりの方が面白いんじゃない?
0110132人目の素数さん垢版2017/12/25(月) 01:21:34.91ID:ejCCK94L
>>108
ありがとうございます
省略した部分は本質的に関係ないと思っていました
普通だと読み飛ばすのですが、わざわざ(1)と引用してるし、上に書いた文の後にも
『そこで、具体的な式(1)をはなれて、この性質を線形性とよぶ。あとで触れるように、この性質は連立一次方程式以外でも顔を出し、.......』
と、連立一次方程式と線形性との関係を強調しているように感じモヤモヤが消えないです
0111132人目の素数さん垢版2017/12/25(月) 01:38:52.43ID:HAz1LRxT
>>97

微分すれば
 3/x - 1/(22-x-y-z)= 2/y - 1/(22-x-y-z)= 5/z - 1/(22-x-y-z)= 0,

(x,y,z)=(6,4,10)

log は上に凸だから、これは極大。

>>42

・長寿ランキング of 他分野

93歳    ライナス C.ポーリング(1901/02/28〜1994/08/19) 化学者
0112132人目の素数さん垢版2017/12/25(月) 02:24:27.42ID:F8i6Eo3d
>>110
f:R^2→R^2としてv=(x1,x2)、uは定数ベクトルとします
方程式f(v)=uを考えるとこれはx1とx2の連立方程式とみなすことができて、fが線形写像となっているならば、1次式で表せるということかと思います
0113132人目の素数さん垢版2017/12/25(月) 02:52:57.39ID:ejCCK94L
>>112
ありがとうございます
fが線型性を持つ→fは行列の掛算だとみなせる→f(v)=uは連立一次方程式
ってことですね
納得できました
0114132人目の素数さん垢版2017/12/25(月) 05:34:03.10
テスト

>>111 ビタミンCで長生きしたんやろなあ
0116132人目の素数さん垢版2017/12/25(月) 08:15:33.57ID:7sm5wWUB
>>114

たしか、ビタミンCを飲めば、がんにならないとか言っていたにもかかわらず、がんで死んだんですよね。
0117132人目の素数さん垢版2017/12/25(月) 08:17:25.25ID:7sm5wWUB
>>111

彌永昌吉さん、ハンス・ベーテがまだ登場していないように思うのですが。
0118132人目の素数さん垢版2017/12/25(月) 08:17:51.50ID:7sm5wWUB
あと伏見とかいう人。
0119132人目の素数さん垢版2017/12/25(月) 08:21:39.48ID:7sm5wWUB
John Archibald Wheeler

とかいう人はどうでしょうか?
0120132人目の素数さん垢版2017/12/25(月) 10:43:59.53ID:flpqHK5r
目的変数Yを説明変数Xで回帰する問題を考えています。回帰係数ベクトルをB、誤差項ベクトルをeとすると
Y = XB + e
と表せる問題です。最小二乗法によるBの推定値は

B = (X'X)^(-1) X'Y

で表すことが出来ます。今、Xの分散共分散行列をWとすると、Xの精度行列W^(-1)です。Graphical Lassoのアルゴリズムによりスパースな精度行列W^(-1)の推定値を持っているとき、上記のBの式とW^(-1)の関係はどのようになりますでしょうか。

よろしくお願いいたします。
0123132人目の素数さん垢版2017/12/25(月) 14:26:09.08ID:LxVTtF2g
xをtの関数としたとき、微分方程式x"-tx'+x=tの一般解が求められません
自力で解いたのは、
x=tのとき左辺が0なのでd'Alembertの階数低下法より
u=x/t, v=u'とおいてv'+(-t+(2/t))v = 1となり、
そこから計算したところu = ∫(e^(t^2/2)/t^2)*(∫t^2/e^(t^2/2)dt)dtとなり、詰みました。
もしよければ教えていただけないでしょうか?
0124132人目の素数さん垢版2017/12/25(月) 14:37:04.90ID:HAz1LRxT
>>117-119
トンクス

100歳 60日 彌永昌吉 (1906/04/02〜2006/06/01)
  …… 分かスレ435-637

98歳    Hans Albrecht Bethe (1906/07/02〜2005/03/06)物理学(恒星の核融合、質量公式など)

98歳    伏見康治 (1909/06/29〜2008/05/08) 物理学
  …… 分かスレ435-643

96歳    John Archibald Wheeler (1911/07/09〜2008/04/13)物理学(中性子星、重力崩壊など)
0127132人目の素数さん垢版2017/12/25(月) 16:06:14.63ID:ZEXlck78
宇宙とは何ですか?また、宇宙はどこにあるのでしょうか?
0128132人目の素数さん垢版2017/12/25(月) 17:49:30.88ID:7sm5wWUB
>>124

福原満洲雄という人はどうでしょうか?
0129132人目の素数さん垢版2017/12/25(月) 18:43:50.63ID:Lmzg1YUu
P≠NPなんですか?
0130132人目の素数さん垢版2017/12/25(月) 21:19:06.63ID:Tbui0wkZ
>>123
x(t)=
(1/(2 Sqrt[t^2]))E^(-(t^2/2)) (E^(t^2/2) Sqrt[t^2] t^3 \!\(
\(\*SubscriptBox[\(\[InvisiblePrefixScriptBase]\), \(2\)]\)
\(\*SubscriptBox[\(F\), \(2\)]\)\)\[InvisibleApplication]
(1,1;1/2,2;-(t^2/2))+\[Pi] E^(t^2/2) t^2 erf(t/Sqrt[2]) erfi(Sqrt[t^2]/Sqrt[2])-Sqrt[2 \[Pi]] E^t^2 Sqrt[t^2]
erf(t/Sqrt[2])-Sqrt[2 \[Pi]] t^3 erfi(Sqrt[t^2]/Sqrt[2])+2 E^(t^2/2) Sqrt[t^2] t)-(Subscript[c, 2] Sqrt[-t^2] (2 Sqrt[\[Pi]]
(1-(Sqrt[-t^2] erfi(Sqrt[t^2]/Sqrt[2]))/Sqrt[t^2])+
(2 Sqrt[2] E^(t^2/2) t^2)/(-t^2)^(3/2)-2 Sqrt[\[Pi]]))
/(2 Sqrt[2])+Sqrt[2] Subscript[c, 1] t
0132132人目の素数さん垢版2017/12/25(月) 22:07:27.75ID:LxVTtF2g
>>130
すいません、よくわかりませんorz
0133132人目の素数さん垢版2017/12/25(月) 22:45:26.81ID:V1yhqr/U
超天才数学者と超天才仏教僧はどっちの方が天才であると言えるのでしょうか?天才度はどっちの方が高いのでしょうか?
0134132人目の素数さん垢版2017/12/25(月) 22:52:26.78ID:F8i6Eo3d
神の天才度が高いですね
0135132人目の素数さん垢版2017/12/25(月) 22:57:26.22ID:V1yhqr/U
神と全と無を格の高い順にランク付けするとどうなりますか?
自分はどう考えても無が一番上だと思うのですが。
0136132人目の素数さん垢版2017/12/25(月) 23:04:29.45ID:F8i6Eo3d
神が一番上ですね
0137132人目の素数さん垢版2017/12/25(月) 23:09:34.62ID:V1yhqr/U
理由を教えてください。
0138132人目の素数さん垢版2017/12/25(月) 23:10:31.17ID:F8i6Eo3d
神は全能だからです
0139132人目の素数さん垢版2017/12/25(月) 23:16:59.56ID:V1yhqr/U
全能でも無には勝てないでしょ。
無って何がどうなっても微動だにしないよ。
0140132人目の素数さん垢版2017/12/25(月) 23:19:01.32ID:F8i6Eo3d
無という概念はあり得ないものです
神は常に存在してますから
0141132人目の素数さん垢版2017/12/25(月) 23:19:53.18ID:lL5Q9RV5
何も無さで格付けしたら
無、神、全の順で格が高い
何かを生み出す能力で格付けしたら
神、全、無の順
0142132人目の素数さん垢版2017/12/25(月) 23:24:02.24ID:lL5Q9RV5
神がいれば全ができるため無は無い
無は全が無いことだから神がいない必要がある
つまり
神がいるなら神>全>無=0
神がいないなら無>神=全=0
0143132人目の素数さん垢版2017/12/25(月) 23:31:20.22ID:F8i6Eo3d
では、神がいることは明らかですから、神が最強ということですね
0145132人目の素数さん垢版2017/12/25(月) 23:41:29.45ID:F8i6Eo3d
作ったとかではなく、神はただそこに存在するものですね
0147132人目の素数さん垢版2017/12/25(月) 23:48:04.02ID:F8i6Eo3d
そのようなものはないですね
神は誰の指図も受けずに、ただそこに存在するのです
0148132人目の素数さん垢版2017/12/25(月) 23:48:04.69ID:V1yhqr/U
>>140
>無という概念はあり得ないものです
当たり前です。それを無と呼ぶのですから。

>>142
>神がいれば全ができるため無は無い
当たり前です。無ってのは無いことなんだから。無は無い。当たり前。

>無は全が無いことだから神がいない必要がある
そういうことではありません。
例えば、全があるとします。
そうすると、「無は無い」となります。
つまりこれが無ということになります。
だって無ってのは無いことなんだから。
別の考え方だと、例えば全があるとします。
そうすると、全は「無では無い」
これも無です。
だって無ってのは無いことなんだから。
これを見てなんとなく分かってきたと思いますが、
全てが無なのです。
無は一番上です。
0149132人目の素数さん垢版2017/12/25(月) 23:49:33.26ID:V1yhqr/U
最強は無です。
しかし、なぜ無が最強かはかなり深く考えないと分かりません。
0150132人目の素数さん垢版2017/12/25(月) 23:49:48.42ID:F8i6Eo3d
最強は神ですよ
0151132人目の素数さん垢版2017/12/25(月) 23:52:32.74ID:V1yhqr/U
最強は無です。
神ですら無には勝てません。
0152132人目の素数さん垢版2017/12/25(月) 23:53:53.18ID:F8i6Eo3d
でも、無は現実ではないですが、神は現実に存在していますよ?
0154132人目の素数さん垢版2017/12/25(月) 23:56:33.79ID:V1yhqr/U
無はある意味現実である意味非現実なのです。
0155132人目の素数さん垢版2017/12/25(月) 23:57:12.25ID:F8i6Eo3d
非現実なものがどうして現実なものよりも最強なんですか?
0156132人目の素数さん垢版2017/12/25(月) 23:58:18.58ID:lL5Q9RV5
神が存在しない世界で神を作る概念がいたとする
これを前者と名付ける
前者は神ではないので神以外のものを作ることができない
一方、神は全をつくるので神を作り出すことができる

そのため「前者が作る神」と「神が作る神」の両者が存在することになるし、前者は神の存在する世界に存在し続けている

前者が神の存在する世界に存在するなら、神が前者を作ることができる
すると前者の存在は無くてもいい非合理的な概念になる
0157132人目の素数さん垢版2017/12/25(月) 23:59:09.56ID:V1yhqr/U
>>153
無ってのはある意味必ずしも無とは限らないのです。
0159132人目の素数さん垢版2017/12/26(火) 00:04:08.49ID:ha3p3os9
神チューバー
0161132人目の素数さん垢版2017/12/26(火) 00:12:42.25ID:F2YLCYJx
つまり前者は神を産み出した瞬間に消滅する

前者を産み出した概念を前々者とする
前々者は前者のみを作り出すことができる
前者は前々者を作り出せない

前々者が存在するにはそれを作り出す概念が必要であるから、前者以前の世界では次の概念を作り出すための概念のみが存在している

しかし、このような前者以前の概念がいくつもあるのは非合理的
つまり前者は神が存在する以前の時間に対して永遠に存在する必要がある
0162132人目の素数さん垢版2017/12/26(火) 00:15:19.61ID:oROs5baw
>>160
神は何かの物質によってできているわけではありません
0164132人目の素数さん垢版2017/12/26(火) 00:16:32.86ID:F2YLCYJx
神が存在する以前の時間が存在しない場合(神が時間を作ったことを仮定する)
前者は時間も空間も存在しない世界に満たされたただひとつの概念だと言える
0165132人目の素数さん垢版2017/12/26(火) 00:16:41.67ID:oROs5baw
>>163
神はこの世の理から外れていますから、そのような物に縛られる必要はないのです
0168132人目の素数さん垢版2017/12/26(火) 00:19:35.34ID:F2YLCYJx
物理法則である神は様々な条件で観測される事によってその都度異なる姿となって顕現する
0169132人目の素数さん垢版2017/12/26(火) 00:19:39.82ID:oROs5baw
>>166
そのような概念から外れていますからわからないですね
神は神なんです
一つ言えるのは、物質により構成されうるようなものではないということだけですね
0171132人目の素数さん垢版2017/12/26(火) 00:21:25.99ID:oROs5baw
>>170
何でできているという概念は通用しないんですよ
神は神としてそのままの状態で存在しています
0173132人目の素数さん垢版2017/12/26(火) 00:23:37.27ID:oROs5baw
>>172
神は複数存在しませんよ
一人だけです
0175132人目の素数さん垢版2017/12/26(火) 00:26:15.43ID:oROs5baw
>>174
神は自分のことだと答えるでしょうね
0178132人目の素数さん垢版2017/12/26(火) 00:29:24.34ID:oROs5baw
>>177
神は神です
未来永劫不滅の存在ですね
0182132人目の素数さん垢版2017/12/26(火) 00:36:55.71ID:oROs5baw
>>180
できてますよね
0184132人目の素数さん垢版2017/12/26(火) 00:38:54.73ID:oROs5baw
>>183
未来永劫不滅の存在です
また全知全能です
0185132人目の素数さん垢版2017/12/26(火) 00:41:10.62ID:F2YLCYJx
>>177
神はそれ自身が神を表現していることを認知している
全は神によって作られたが、神は全によって作られないことも認知している

とはいえ理想化された全の要素は神に近い概念を表現することが可能
0186132人目の素数さん垢版2017/12/26(火) 00:41:24.51ID:/B0PS8cj
>>184
未来永劫という概念は時間という概念が先だって必要なので、時間がないと神は存在できないのですね
0187132人目の素数さん垢版2017/12/26(火) 00:42:07.07ID:oROs5baw
>>186
時間や宇宙が存在する前から、神はずーっといるんです
0191132人目の素数さん垢版2017/12/26(火) 00:45:18.44ID:oROs5baw
>>188
どこが矛盾してるんですか?
0192132人目の素数さん垢版2017/12/26(火) 00:46:46.69ID:/B0PS8cj
>>191
未来という言葉は時間を必要とするので、定義により、時間がなければ神は存在できません
0194132人目の素数さん垢版2017/12/26(火) 00:47:15.63ID:oROs5baw
>>192
未来永劫とは、例えですね
とにかく、神は常に存在してるわけです
0195132人目の素数さん垢版2017/12/26(火) 00:49:00.21ID:/B0PS8cj
>>193
端的にお願いします

>>194
常という言葉も時間の概念が必要ですね
どうやら神よりも時間の方が上らしいですね
0196132人目の素数さん垢版2017/12/26(火) 00:52:24.16ID:F2YLCYJx
>>195
つまり、神を産み出したのは
「時間、空間を含む神の産み出したあらゆるものが無い世界」である前者
0197132人目の素数さん垢版2017/12/26(火) 01:00:24.99ID:/B0PS8cj
>>196
それが神よりも偉いんですね


ID:oROs5baw さんは脱落したので、神を語る資格はもうありませんね
0200132人目の素数さん垢版2017/12/26(火) 01:03:16.75ID:oROs5baw
>>195
言葉というのは基本的にトートロジーですから仕方ないことです
ですが、言葉で表せないからといって、それが間違いだとは限りませんよね?
神はそこにあるわけてす
これはわかりますよね?
0202132人目の素数さん垢版2017/12/26(火) 01:07:43.41ID:oROs5baw
>>201
時間や宇宙は神が作ったものです
このことからも、神は時間に依存することなく存在することがわかりますね
0204132人目の素数さん垢版2017/12/26(火) 01:09:16.94ID:F2YLCYJx
>>201
ありがとうございます
概念が作り出される順番を上位から下位で表現し、より上位であることを偉いと表現していたんですね
0205132人目の素数さん垢版2017/12/26(火) 01:11:48.78ID:oROs5baw
>>203
まず現象があって、その現象に対して名前をつける
これが科学的な方法論ですね
まず神がいる、それを神と呼ぶ
どこがわからないんですか?
0207132人目の素数さん垢版2017/12/26(火) 01:15:16.83ID:oROs5baw
>>206
時間がないと神は存在できないとどこに書かれているんですか?
0209132人目の素数さん垢版2017/12/26(火) 01:16:35.56ID:F2YLCYJx
>>205
神が存在すれば確かにそれを神と呼ぶことができる
しかし神の存在はこの世界において観測不可能だ
だから存在の仮定に意味があると考えるのは難しい
0210132人目の素数さん垢版2017/12/26(火) 01:17:27.17ID:oROs5baw
>>208
時間がなければ神を定義することはできない、ならなんとなくわかります
神が存在できないのはなぜですか?
0211132人目の素数さん垢版2017/12/26(火) 01:21:10.82ID:/B0PS8cj
>>210
時間がなければ未来永劫不滅なものは存在できないからです
時間がなくても未来永劫不滅なものが存在できると考えるのは何故ですか?
0212132人目の素数さん垢版2017/12/26(火) 01:22:36.32ID:oROs5baw
>>211
未来永劫というのは例えだと言いましたね
確かに時間の概念がないと、未来永劫という言葉は意味をなしません
でも、とにかく神はずっといるんです
宇宙や時間が存在する前から
0213132人目の素数さん垢版2017/12/26(火) 01:23:28.52ID:/B0PS8cj
>>212
ずっとという言葉も時間の概念が必要ですね
どうやら神よりも時間の方が上らしいですね
0214132人目の素数さん垢版2017/12/26(火) 01:24:08.10ID:oROs5baw
>>213
ずっとも比喩ですよー
0216132人目の素数さん垢版2017/12/26(火) 01:27:04.41ID:oROs5baw
>>215
定義する、という言葉を、定義や自己言及や比喩や例えを用いずに説明することはできますか?
0218132人目の素数さん垢版2017/12/26(火) 01:28:30.76ID:oROs5baw
>>217
知ってるでしょうね
なんでも知ってますから
0219132人目の素数さん垢版2017/12/26(火) 01:28:46.63ID:F2YLCYJx
>>210
時間が無く空間が存在する世界に神が存在するとき、この神は時間しか作ることができない
つまり、時間の無い世界において神は全を作ることができない
よって時間の無い世界で神は存在できない
0221132人目の素数さん垢版2017/12/26(火) 01:36:07.53ID:oROs5baw
>>220
私は神ではないですからね
私はあなたに質問したのですけど?
0222132人目の素数さん垢版2017/12/26(火) 01:42:11.07ID:fqF7Lnuj
谷山志村予想を日本では谷山予想と呼ぶ人が多い。
外国で谷山志村予想と呼ばれている。 志村の業績は大きい。
あの当時では勇気のいる予想であり、志村の努力と実績が大きい。
(谷山が自殺したのは、残念である。 志村の本をみると二人の友情は数学のロマンでもある。)

志村は日本の数学界、教育界、インテリの偽善ぶりを痛烈に批判している。
こういうことも日本の数学界が志村を無視する一因でもあるのだろう。
歴史の経過を現時点でみれば、志村が正しいのは明白である。
0223132人目の素数さん垢版2017/12/26(火) 01:45:57.45ID:/B0PS8cj
>>221
そもそもあなたに神を定義しろとは言ってません
神はなんでもできるらしいので>>215もできると思うのですが、どうですか?
0224132人目の素数さん垢版2017/12/26(火) 01:47:05.60ID:oROs5baw
>>223
私は神でないから答えられませんが、神はできますよ?
0225132人目の素数さん垢版2017/12/26(火) 01:47:43.29ID:oROs5baw
てか、あなたに聞いてるんですけどw
0226132人目の素数さん垢版2017/12/26(火) 01:52:21.28ID:/B0PS8cj
草が生え始めましたね

>>224
神による神の定義において、使われている概念のひとつをAとします
このとき、定義より、Aがなければ神は存在できないので、Aは神より上となります
0227132人目の素数さん垢版2017/12/26(火) 01:55:37.32ID:oROs5baw
>>226
定義とは言葉を用います
言葉とは本質的にトートロジーです

あなたが定義をトートロジーを用いずに説明できないように、言葉とはそういう面もあるんです
0229132人目の素数さん垢版2017/12/26(火) 01:59:09.92ID:oROs5baw
自己言及を避けることはできないということです
0231132人目の素数さん垢版2017/12/26(火) 02:03:01.59ID:oROs5baw
人間の言葉ですからね、仕方ないです
0232132人目の素数さん垢版2017/12/26(火) 07:26:19.70ID:/B0PS8cj
>>231
神は神の言葉を使うのではないですか?
また、神は他にどんなことができないのですか?
0233BLACKXスマホ ◆jPpg5.obl6 垢版2017/12/26(火) 09:28:59.04ID:65XIxb/4
神は人の風習や習わしや集団行動における危険学習の成果
人間の中に神は居る、しかし時代とともに神の概念が変わって来て
昔の人の間の神は受け入れ難くなってるから疑問が生じる
0235132人目の素数さん垢版2017/12/26(火) 12:10:36.12ID:fqF7Lnuj
志村を無視する数学者は国際的には2流である。
0237132人目の素数さん垢版2017/12/26(火) 13:21:34.65ID:V83tGg3y
この問題と回答について質問です
https://m.chiebukuro.yahoo.co.jp/detail/q1019052030

点pをAB上、点qをBC上に取る理由
pとqの範囲が、ともに≦ではなく<をとる理由
「面積の条件から2pq=a」はなぜそうなるのでしょうか?
あとPQ^2=〜の式にある≦は相加平均相乗平均であってますか?

以上4つお願いします!
0238132人目の素数さん垢版2017/12/26(火) 13:29:12.78ID:bh2BICch
>>236
c=a+bがabの約数ね
ab=k(a+b)としたら
(a-k)(b-k)=kk
うーん
0239132人目の素数さん垢版2017/12/26(火) 13:43:14.15ID:ZMM98NSr
>>83
a^2+ab+b^2=(a+b)^2ーab=c^2ーab
これがcの倍数ならabがcの倍数
a=b=2,c=4が条件を満たす。このときd(c)=1d(c)<1となる正整数cは1のみだが、条件a+b=cを満たす正整数a,bは存在しない
よってd(c)の最小値は1
0241132人目の素数さん垢版2017/12/26(火) 14:21:05.19ID:ZMM98NSr
>>239
d(c)=1なのでcは素数の冪p^k(k≧1)であるが、ab=a(cーa)=acーaaがcの倍数なのでaaもc=p^kの倍数であり、aはpを素因数にもつ。
同様にbはpを素因数にもつ。
c=a+b≧2p>pである。よってc=p^kとしたときk≧2でなければならない
逆にc=p^k(k≧2)に対して条件を満たすa=p^(kー1),b=(pー1)aが必ず存在する
よってd(c)の最小値を与えるcの条件は、c=p^k(p:素数かつk≧2)
0242132人目の素数さん垢版2017/12/26(火) 15:46:19.61ID:KANJOZGi
>>123

u(t)=∫[0,t]{-1/s +(1/ss)e^(ss/2)∫[0,s]e^(-rr/2)dr}ds + c1∫[0,t](1/ss)e^(ss/2)ds + c2

=(tt/6) 2F2(1,1;2,5/2;tt/2)+ c1{∫[0,t] e^(ss/2)ds -(1/t)e^(tt/2)}+ c2,

う〜ん
0243132人目の素数さん垢版2017/12/26(火) 20:03:34.98ID:vASqnmuP
>>241
素晴らしい証明に感動しました。
a+b=cから条件p<2p<p^2を引き出すところが分からず、p^1=pを解に入れてしまっていました。
ありがとうございました
0244132人目の素数さん垢版2017/12/26(火) 21:26:33.36ID:vASqnmuP
>>237
まずp=0やq=0だと、そもそも三角形が二つに分かれないですよね。これは問題の意図に反する。
だから範囲の両端は≦ではないんです。

次に2pq=aについて。
△OEFで、OE=e、OF=fとします。
OE上に点Sと点Tを、OS=s、OT=tとなるようにとるとき、面積比は
△OST:△OEF=st:ef…(ア)
これは中学の相似で習います。

本題に戻ると、△BPQ:△BAC=1:2にしたいのでした(二等分するのだから、△BPQの面積は全体の半分)。
(ア)を使うと、△BPQ:△BAC=pq:a
。(a=1*aに注意)
よって1:2=pq:aで、2pq=aとなります。
0245132人目の素数さん垢版2017/12/26(火) 21:30:04.96ID:vASqnmuP
>>237
相加相乗平均で合っています。
当たり前ですが、pもqも正で、かつ独立に動くので、今回は相加相乗平均を使えます。
理系なら微分でもいいですが面倒ですね
0246132人目の素数さん垢版2017/12/26(火) 21:44:13.98ID:V83tGg3y
>>245
しかしこれ、頂点上に考えてないのは少しまずいですよね?
0248132人目の素数さん垢版2017/12/26(火) 21:52:31.39ID:V83tGg3y
点Pと点Qが三角形の頂点上にある場合を加味しなければならない、ということです

その場合を追加(具体的に求まる)して最後に大小比較すれば消えるのかな?とは思いますが。
0249132人目の素数さん垢版2017/12/26(火) 22:16:33.97ID:vASqnmuP
>>248
B以外の頂点にある場合は考慮する必要がありますね、それでOKだと思います
多分知恵袋の人もサッと解いたので、その辺の厳密さに目が向かなかったんじゃないでしょうか、本番なら減点だとは思います
0250132人目の素数さん垢版2017/12/27(水) 02:13:21.22ID:W92ufYpH
とあるクジAが当たる確率は9/20で一度引くのに80円かかります。そして当たったクジAを10枚または外れクジA'を20枚使うと別のクジBが引けて当たりクジBが当たる確率は3/10です。

問.当たりクジBを8枚と引き換えに景品がほしい時最低いくらクジ屋へお金を払えば良いですか?


こんな感じの確率が入り組んだ時ってどうやって計算したらいいの?
0251132人目の素数さん垢版2017/12/27(水) 02:26:46.99ID:paTdLx+Q
>当たりクジBを8枚と引き換えに景品がほしい時

の日本語がなんかよくわからないので、詳しくお願いします
0253132人目の素数さん垢版2017/12/27(水) 03:10:43.39ID:X1v+IZZt
ぽよ?
0254132人目の素数さん垢版2017/12/27(水) 05:54:38.82ID:uyGelgwQ
最低、と言うと確率の問題じゃなくなるね
クジBの当たりを8枚引くまでに掛かるクジAの代金の期待値が知りたいんじゃない?
0255132人目の素数さん垢版2017/12/27(水) 07:37:50.71ID:ls/ynmV7
凸関数の性質についての質問です。
『凸関数ならば、任意のx₁、x₂、0≦λ≦1に対して
λ(x₁)+(1−λ)f(x₂)≥f(λx₁+(1−λ)x₂) ー@を満たす。』
と書いてあったのですが、高校数学でイェンゼンの不等式を帰納法で証明する時とか、@はいちいち証明せず性質として用いていいのでしょうか?
(f"(x)≧0が与えられているとして)
というか@自体がイェンゼンの不等式(n=2)からきてるんじゃないかと思うとわけがわからない...
https://mathtrain.jp/jensen_proof


https://i.imgur.com/hSzmI1c.jpg
0257132人目の素数さん垢版2017/12/27(水) 11:29:51.91ID:0lbb/ymp
行列の階数を行列の基本変形で計算する方法についてですが、

「行列の階数の計算のためには、階数定理に無関係で行列式の
計算を要求しないもう1つの方法が存在する。しかしそれは、階数
そのものだけを知りたいので、いったいどういう行(または列)が
極大な1次独立系をなすのかを問題にしない場合にのみ適用される
ものである。」

と書いてあります。これって間違っていますよね?

オリジナルの行列のどの行が極大な1次独立系であるかもどのような
基本変形を行ったかを覚えておけば、分かりますよね。
0260132人目の素数さん垢版2017/12/27(水) 15:36:29.91ID:ls/ynmV7
Can an arc of a parabola inside a circle of radius 1 have a length greater than 4 ?

という問題ってどう和訳すればいい?
0261132人目の素数さん垢版2017/12/27(水) 15:39:45.95ID:Kn+TgMEa
半径1の円内に収まる放物線の孤でその長さを4より大きくすることは可能か
0262132人目の素数さん垢版2017/12/27(水) 17:08:17.47ID:uyGelgwQ
>>250
近似解でよければ:
クジAを引く行動をxy平面上で点を動かす行動に置き換えます。つまり、
原点(0,0)から始めて、クジAを一度引くたびに、点(x,y)を確率9/20で点(x+1/10,y)に、確率11/20で点(x,y+1/20)に移す行動を繰り返します。
このようにすると、aを越えない最大の整数を[a]で表すとして、[x]+[y]が1増えるたびにクジBが1回引けることとなります。
クジAをn回引いたとき点は直線y=n/20ーx/2の上にあります。k回目のクジBを引くとき、点はこの直線と、曲線[x]+[y]=kの交点にあることになりますが、
[x]+[y]=kが多数の線分からなる折れ線となるため、この曲線上で確率分布を考えると計算が複雑になります。
そこで、[x]+[y]=kを、各線分の中点を通る直線x+y=k+1/2で近似します。
x+yはクジAを一度引くたびに、確率9/20で1/10、確率11/20で1/20増加しますので、平均では9/20×1/10+11/20×1/20=29/400増加します。
まず、クジBを8回当てるまでに引く回数の期待値は8÷3/10=80/3回ですので、クジAを引く回数の期待値は概算で、
(80/3+1/2)÷29/400=32600/87≒374.71回
近似したことによる誤差が±10回
金額はこれに80円をかけて約29977±800円
0263132人目の素数さん垢版2017/12/27(水) 21:08:18.45ID:ls/ynmV7
>>260
この問題の解答がこれなんですが、わたしには正直よくわかりません。

どなたか日本語で噛み砕いて、あわよくば和訳して説明していただけますか?
もちろん単純に解いていただいても構いません...


https://i.imgur.com/VF3y70x.jpg
0264132人目の素数さん垢版2017/12/27(水) 21:10:28.50ID:ls/ynmV7
あ、私が考えたのは単純に、2本の直線が直径を通ったら丁度4だけど放物線ってちょっと曲がってるし
どれだけ狭くなっても1直線にはならないんじゃないかな?
って感じです

でも違うんですよね...
0265132人目の素数さん垢版2017/12/27(水) 21:16:01.45ID:s7lJvydj
解答1行目から派手にやらかしてますね
0266132人目の素数さん垢版2017/12/27(水) 21:39:44.14ID:Kn+TgMEa
>>264の直線にはならないんじゃないかなって解釈はおかしくね
直線ではなくて弧長を4より大にすればいいんでしょ
0267132人目の素数さん垢版2017/12/27(水) 22:36:46.64ID:ls/ynmV7
ゴリゴリ積分計算するだけですかね?
0268132人目の素数さん垢版2017/12/27(水) 23:02:31.67ID:BgwkAQ1+
円の半径をsとし、s=aθと表すことはできますか?
また、ds=adθとなるならば、

ds=adθ=dx/cosθ=dy/sinθ

となりますが、この微分方程式より、s=x/sinθ=-y/cosθ となります。

これが s=x/cosθ=y/sinθと矛盾する気がするのですが、どのように解釈すれば良いのでしょうか。

また、原点は(0,-a)だそうですが、これもよくわからないので、教えてください。
0269132人目の素数さん垢版2017/12/28(木) 01:23:00.10ID:09kEM2xk
>>260 >>263
細かいところは結構雑に書いてあるので、行間を埋めたり修正したりすると、
大体以下のような感じ。

放物線y=kx^2の、x^2+(y-1)^2≦1に含まれる部分の長さLは、
十分大きいkについては、T=2√(2k-1)として
L=(1/k)∫[0〜T]√(1+t^2)dt
と表される。
ここで、t≧1で√(1+t^2)>t+0.4/tとなることは容易に示せる
(例えばf(t)=t√(1+t^2)-t^2-0.4は単調増加であることを利用)ので、
A = ∫[0〜1]√(1+t^2)dtとして、

L > (1/k)(A + ∫[1〜T]√(t+0.4/t)dt)
= (1/k)(A + T^2/2 + 0.4logT - 1/2)
= (1/k)(A + 4k - 5/2 + 0.4logT)

となり、kを十分大きくとるとA - 5/2 + 0.4logTを正にすることができるので
L > 4となる。

(もちろん、これはlim[k→∞]L=4であることと矛盾しない。)

なお、本筋には関係ないが、正確に計算すると
A = (log(√2+1)+√2)/2
0270132人目の素数さん垢版2017/12/28(木) 01:51:53.08ID:Gj2H71EO
>>268
元の問題および回答を省略せずに書き写してください
0271132人目の素数さん垢版2017/12/28(木) 02:02:22.18ID:LMhCdvra
一平面上の曲線Γは、その上の各点Pまでの(定点OからΓに沿って測った)長さsとその点での接線の方向θの関数s=s(θ)を与えることによって記述される。これを曲線の弧座標表示という。
 s=aθはどのような曲線を表すのか答えよ。
0272132人目の素数さん垢版2017/12/28(木) 02:07:59.15ID:CJQxizSD
半径1の円と放物線はそれぞれx^2+(y-1)^2=1、y=kx^2としても一般性を失わないのでこれらで考える。
放物線と円は、原点と(±X, Y)の3点で交わる。ただしX=√(2k-1)/k, Y=(2k-1)/kである。
このとき円の内部の弧の長さLはL=2∫_{0}^{X}√(1+4k^2x^2)dxで与えられる。
ここで、t=2kxと置換すればx(0→X);t(0→2√(2k-1)), dt=2kdxという対応関係が得られる。
さらに、T=2√(2k-1)とすればL=1/k∫_{0}^{T}√(1+t^2)dtとなる。

kが小さいとき弧の長さは0に近づく。k→∞のとき放物線は退化して(開きが狭くなって)y軸に沿った二本の直線となり、このとき長さは4となる。

√(1+t^2)の積分は行えるが1/2t√(1+t^2)+1/2ln(t+√(1+t^2))となり複雑である。
弧の長さの評価に苦労しそうなので近似を試みる。最も単純な近似は√(1+t^2)>tだが、
1/k∫_{0}^{T}√(1+t^2)dt>1/k∫_{0}^{T}tdt=4-2/kとなり、4より大きいという評価が得られないのでまだ不十分。

√(1+t^2)を展開すればt+1/(2t)-1/(8t^3)+...が得られる。0の近傍では成立しないので、取り敢えずt≧1の範囲で近似を考えてみる。
L=1/k∫_{0}^{1}√(1+t^2)dt+1/k∫_{1}^{T}(t+0.4/t)dtについて、1/k∫_{1}^{T}(t+0.4/t)dtの部分を計算すると、
4-2/k+0.4/klogTが得られ、T→∞(k→∞)のとき、(0.4/klogT)/(2/k)→∞から、0.4klogT > 2/kが成り立つ。
また、1/k∫_{0}^{1}√(1+t^2)dt = 1/2(√2+log(1+√2))>0であるので、L > 4であることが分かった。
0273132人目の素数さん垢版2017/12/28(木) 03:40:16.76ID:qdiVZepn
>>263

単純に解けば…

交点を(a,b)=({√(2k-1)}/k,(2k-1)/k)として
(a,b)−(a/2,b/4)−(a/4,b/16)−(a/8,b/64)−(a/16,b/256)−(a/32,b/1024)−(0,0)
の折れ線の長さをExcelで計算して2倍したら、
k〜45 ぐらいで4を超えました。
L = 4.000658…
0274132人目の素数さん垢版2017/12/28(木) 04:23:43.57ID:qdiVZepn
t = sinhθとか置いて

∫[0,T]√(1+tt)dt ={T・√(1+TT)+ log(T+√(1+TT))}/2
≧{TT + log(2T)}/2
= 4k -2 + log{16(2k-1)}/4
> 4k    (k > 100)
0275132人目の素数さん垢版2017/12/28(木) 05:27:32.98ID:Gj2H71EO
あー殺したい
殺したい
殺したい
殺したい
殺したい
0276132人目の素数さん垢版2017/12/28(木) 07:08:29.42ID:qNA/GRr/
>>268
極座標表示のs=aθは円ではなく螺旋となりますね
「アルキメデスの螺旋」という名前がついています
もしxとyが(x,y)=(s・cosθ,s・sinθ)を表しているなら、
x=s・cosθ=aθcosθから
dx/dθ=a(cosθ+θsinθ)となります
adθ=dx/cosθはdx/dθ=acosθを表していると思われますが、これは誤りです
導出過程の考え方に問題があると思われます。少しやりなおしてみましょう
0279132人目の素数さん垢版2017/12/28(木) 11:29:57.67ID:4AgiLVkd
等式
{(2b-a)/c}={(2c+b)/a}={(ka-c)/b}
を満たす自然数の組(a,b,c)が存在するような自然数kを全て求めよ。
無数に存在する場合はkがどのような自然数であるかを述べよ。
0281132人目の素数さん垢版2017/12/28(木) 14:53:24.93ID:jRuqely8
宇宙飛行士とロスチャイルドはどっちの方が凄いですか?
0283132人目の素数さん垢版2017/12/28(木) 20:03:46.87ID:SOnWdl0y
天皇とダライ・ラマはどっちの方が偉いですか?
0284132人目の素数さん垢版2017/12/28(木) 20:08:52.61ID:5Z1xqBCp
デカ•マラの方が偉いです
0285132人目の素数さん垢版2017/12/28(木) 20:15:33.72ID:SOnWdl0y
アル=アズハル大学総長とコンスタンディヌーポリ総主教は宗教権威としてどっちの方が格上ですか?
0287132人目の素数さん垢版2017/12/29(金) 01:31:08.67ID:Fcad1R4A
>>260 >>263

L ={8/(4+TT)}{T・√(1+TT)+ log(T + √(1+TT)}/2,   >>274
より、最大となるTは
4√(1+TT) - T・log(T+√(1+TT))= 0,

k = 94.091281195985373635077640
T = 27.362935689868566971379010
L = 4.0026702976799552884212295(最大)
0289132人目の素数さん垢版2017/12/29(金) 02:50:51.04ID:i7XTPUMn
AB=48, BC=52, AC=20である三角形ABCの内部に2つの円O,Pがある.
円O, Pは同じ長さの半径をもち, 互いに外接している.
また, 円Oは辺AB, BCとそれぞれ点D, Eで接し, 円Pは辺BC, ACとそれぞれ点F, Gで接している.
BE:CFおよび円Oの半径を求めよ.

全然分からないです……
0290132人目の素数さん垢版2017/12/29(金) 03:14:51.10ID:i7XTPUMn
>>289
円Oの半径は面積を使って求められるんですが、BE:CFを求めさせる意図が分からないです
0291132人目の素数さん垢版2017/12/29(金) 11:07:43.94ID:y8mFmvrz
A を n 次正方行列とし、 det(A) = 0 とする。

このとき、 A の余因子行列を A* とすると、

det(A*) = 0
rank(A*) ≦ 1

が成り立つことを証明せよ。
0292132人目の素数さん垢版2017/12/29(金) 11:18:15.82ID:y8mFmvrz
det(A*) ≠ 0 と仮定する。

det(A) = 0 だから

A*(A*) = O

O*(A*)^(-1) = A*(A*)*(A*)^(-1) = A

A* = O* = O

det(A*) = det(O) = 0

これは矛盾である。
0293132人目の素数さん垢版2017/12/29(金) 11:19:22.48ID:y8mFmvrz
rank(A*) ≦ 1

を証明してください。
0294132人目の素数さん垢版2017/12/29(金) 11:42:40.95ID:y8mFmvrz
det(A) = 0 だから

rank(A) < n

である。

rank(A) ≦ n - 2

ならば、

すべての余因子は明らかに 0 である。

よって、 A* = O

rank(A*) = 0
0295132人目の素数さん垢版2017/12/29(金) 11:48:51.48ID:y8mFmvrz
rank(A) = n - 1

ならば、

0 でない余因子が存在する。

よって、

rank(A*) ≧ 1

である。

det(A) = 0 だから、

A * (A*) = O

である。

任意の n 次元ベクトル x に対して、

A * (A*) * x = 0

である。よって、

Im(A*) ⊂ Ker(A)

である。

dim(Ker(A)) + dim(Im(A)) = dim(Ker(A)) + rank(A) = dim(Ker(A)) + (n - 1) = n

であるから、

dim(Ker(A)) = 1

である。

よって、

rank(A*) = dim(Im(A*)) ≦ 1

である。

以上より、

rank(A*) = 1

である。
0296132人目の素数さん垢版2017/12/29(金) 11:49:15.82ID:y8mFmvrz
>>294-295

より、

rank(A*) ≦ 1

である。
0297132人目の素数さん垢版2017/12/29(金) 12:52:27.74ID:LwE4Dgtx
>>290
意図っていうか、この設問で間接的にBEとCFの長さがわかる(もしくは求める必要がある)
つまり、『円の面積だけじゃなくて位置もちゃんと把握できてるよな?』を確認したいというのが意図じゃない?
0298132人目の素数さん垢版2017/12/29(金) 14:06:04.52ID:FZBCcLtg
√2が実数であることを示してください
0300132人目の素数さん垢版2017/12/29(金) 16:57:50.94ID:BxAgcdJH
>>298
√2は文字列なので実数ではありません
0302132人目の素数さん垢版2017/12/29(金) 22:25:48.04ID:NkuzGyy/
感動する!数学って本持ってる人このスレでID付きでうpしてくれ
今日中なら大丈夫

【年末年始暇な奴来い】安価で指定されたものを全力で探してうpするスレ
http://hebi.5ch.net/test/read.cgi/news4vip/1514548120/
0303132人目の素数さん垢版2017/12/30(土) 01:31:23.05ID:9B6mLulH
円の方程式は、中心は点なので、0+0=r^2になるのですか?
0304132人目の素数さん垢版2017/12/30(土) 01:49:16.06ID:wkDZuUnP
ジョルダン標準形の作り方教えてくれませんか
参考書読んでもよくわからないです
0305303徴発吉川晃司垢版2017/12/30(土) 03:27:13.61ID:9B6mLulH
お前ら、0+0=0^2、0+0=0
つまり、点だということを理解できるか?一度でも考えたことがあるか?
0306132人目の素数さん垢版2017/12/30(土) 03:48:08.10ID:JB6RAHIi
弥勒(僧)とニールス・アーベルはどっちの方が凄いですか?
0307132人目の素数さん垢版2017/12/30(土) 04:04:55.63ID:t8/AOqf4
>>305
あなたがトンデモだということは考えたことがありますね
0308132人目の素数さん垢版2017/12/30(土) 08:08:05.98ID:9B6mLulH
円の方程式という式を作ること、そしてそれが成立することがわからない。
0309132人目の素数さん垢版2017/12/30(土) 08:44:49.01ID:/EcKcOjl
>>305
つまり、自らの心を観察するとは限りない分析、分裂、分別を招くだけ。

なら、どうするか?
観察するものが観察されるものと識るのみである。

どうやって識るのか?

大いなる ものにいだかれ あることを
けさふくかぜの すずしさにしる

此があれば彼があり、此がなければ彼がない。
此が生ずれば彼が生じ、此が滅すれば彼が滅す。
0312132人目の素数さん垢版2017/12/30(土) 09:31:09.02ID:Eg/yPu8D
>>311
公理だよ
0315糞凡愚垢版2017/12/30(土) 09:59:30.17ID:/EcKcOjl
>>314
まぁ根拠を示しなよw

2回目
0316132人目の素数さん垢版2017/12/30(土) 10:51:43.86ID:jCgTb9dO
誰がトンデモとも言ってないのに盛大にファビョってますな
正に屁は元から騒ぎだす
0317132人目の素数さん垢版2017/12/30(土) 10:53:21.49ID:n9Bt5X4R
バカの集まるスレというのはここですか?
0318132人目の素数さん垢版2017/12/30(土) 10:55:49.75ID:Kph6jccf
バカはバカを蚋
0319132人目の素数さん垢版2017/12/30(土) 10:58:00.95ID:KJ2eFTPY
冬休みだなあ
0320132人目の素数さん垢版2017/12/30(土) 11:15:21.26ID:ULLlXxdf
伊理正夫さんは偉い学者なのでしょうか?


----------------------------------------------------------------------
現在我が国では(海外でも)「行列」,「行列式」,「線形代数」,等の言葉を本の表題
あるいは一部の章の表題に含む本は数え切れないほどある.そこに新たに一つを
付け加えることにどれだけの意味があるか,疑問に思われる方も少なくないかも
しれない.しかし,私には,現存のそれらの本はどれも大同小異にみえる。
やや暴言をお許し頂ければ,怠惰な学生と広範囲の応用の経験に乏しいのに
応用系の学生を教えなければならない教師とに阿(おもね)るかのごとくに書かれた
“分かりやすくて通り一遍の”教科書か,著者が“自分がどこまで抽象的にしかも
厳密に理解しているかをひけらかす”ような数学者のための数学専門書かの
どちらかで,数学の利用者,消費者をほとんど無視したものばかりのようである.
私は,長年多くの応用分野で線形代数に関連した方法を利用し,また不足している
ところは必要に応じて自前で補ったりしながら数理工学的な研究を続けてきたもので
あるが,現在遍在している上記のような本の著者達とは思い切って立場を変えて,
利用者の観点からおよそ何かの役に立ちそうなものを体系的に整理して一冊の本に
纏めてみるのも無駄ではなかろうと常々考えていた.
----------------------------------------------------------------------
0321糞凡愚垢版2017/12/30(土) 11:32:37.85ID:ZdsKgfpW
汾州の無業は 、かつて仏教の学問のすべてをつくしたが 、心そのものがそのまま仏であるという道理がわからなかった 。

かれが 、そのことを馬祖にただしたとき 、馬祖の答えはつぎのようであった 。

「君がわからぬといっている心そのものがそれであり 、けっして別のものはないのだ 。そのことがわからぬのが迷いであり 、それがわかれば悟りだ 。あたかも 、手が拳となり 、拳が手となるように 」
0323132人目の素数さん垢版2017/12/30(土) 12:29:12.98ID:ULLlXxdf
伊理正夫著『線形計画法』を読んでいます。

「要するに、一日寝っころがって読んでいれば“線形計画法”についての一通りの偽物でない
知識と技能が身につくような、そのような本にしてみたいということである。」

などと書かれていますが、一日で読めるとは思えません。
0324132人目の素数さん垢版2017/12/30(土) 12:33:48.74ID:WZ69HtKQ
誰に知識と技能が身につくかが曖昧ですが
要は伊理せんせーは一日で読める、ということではないでしょうか
0325132人目の素数さん垢版2017/12/30(土) 12:41:51.35ID:ULLlXxdf
伊理正夫著『線形計画法』を読んでいます。

まえがきに以下のように書いてあります。

「いわゆる“入門書”には、“わかったつもり”にはならせるが、その実、不正確・不十分な
記述が多いこと。(もっとも、面倒なことを避けてわかった気にさせて貰えるというのは、
たいへんありがたいことなのではあるが。)」

「いわゆる“専門的大著”には、必要以上に高踏的な論法を用いたり本質的には差異の
ないことがらをことさら分類・区別したりするなど、読者に過度の負担をかける傾向が
あること。(もっとも、これは“勉強になって”よいことでもあるが。)」
0328菩提達磨垢版2017/12/30(土) 13:29:47.84ID:Y+pKeTIk
法学と航空宇宙工学ってどっちの方がムズイ?
0332132人目の素数さん垢版2017/12/30(土) 15:40:54.41ID:ULLlXxdf
>>329

具体的に挙げてください。
0333132人目の素数さん垢版2017/12/30(土) 15:41:38.94ID:ULLlXxdf
伊理正夫著『線形計画法』を読んでいます。

行と列のindexを自然数の有限集合に限定せず、任意の有限集合としています。

確かに、合理的ですね。

Philip N. Klein著『Coding the matrix』と同じですね。
0334132人目の素数さん垢版2017/12/30(土) 15:49:37.18ID:ULLlXxdf
ロシア系といえば

GantmacherのThe Theory of Matricesってすごい本らしいですね。
0335132人目の素数さん垢版2017/12/30(土) 15:54:42.81ID:ULLlXxdf
Gilbert Strangの線形代数の本の良さが分かりません。
0336132人目の素数さん垢版2017/12/30(土) 15:57:26.55ID:ULLlXxdf
凸解析のいい本を教えてください。

非線形解析のいい本を教えてください。

非線形解析って実際に役に立っているのでしょうか?
0337132人目の素数さん垢版2017/12/30(土) 15:59:03.36ID:ULLlXxdf
伊理正夫さんと甘利俊一さんはどっちのほうが数学が得意な工学者でしょうか?
0339132人目の素数さん垢版2017/12/30(土) 16:44:31.56ID:ULLlXxdf
松坂和夫著『解析入門4』を読んでいます。

線形写像のノルムについて書いてありますが、他にそれについて書いてある
微分積分の本ってありますか?
0342132人目の素数さん垢版2017/12/31(日) 01:29:43.23ID:bwFalEhI
ばかはしななきゃなおらなあい
0345132人目の素数さん垢版2017/12/31(日) 17:55:56.74ID:t2s3ffvn
ここ簡単すぎるから見る気にならないよな
なみんなもそーだろ?な?
0347132人目の素数さん垢版2017/12/31(日) 18:35:57.51ID:ky9WzhjK
x=sin1/xのグラフてなんでこのになるのか、という出題に対し

そもそもx=0で定義できないからx=0では不連続なのは明らか
微分可能かということ以前にx≠0では1/xが連続であって、sinも連続だからその合成関数sin(1/x)は連続、それに連続関数xをかけたものも連続という程度の話だ
という回答をしたのですが

過程ではなくて連続性を調べろなのでそれではだめだ、題意とずれていると言われました

この回答は間違っているのか、そうなら正しい回答を教えてください

https://i.imgur.com/IItzRZb.jpg
0348132人目の素数さん垢版2017/12/31(日) 18:39:40.02ID:vRYXSQrZ
>>347
それ関数何??
0349132人目の素数さん垢版2017/12/31(日) 18:39:49.15ID:qB3hGKZg
いろいろともうダメだろ
0350132人目の素数さん垢版2017/12/31(日) 18:44:32.13ID:ky9WzhjK
xsin1/xです
0351132人目の素数さん垢版2017/12/31(日) 18:45:46.09ID:ywPl/uaQ
困難しかおらんの?
0352132人目の素数さん垢版2017/12/31(日) 18:45:58.84ID:OyfTQ5Xf
>>350
x=0のときf(x)=0と書かれていませんか?
0353132人目の素数さん垢版2017/12/31(日) 18:48:29.19ID:ky9WzhjK
>>352
この画像で質問されたので、この画像から読み取れることまでしかわかりません
0354132人目の素数さん垢版2017/12/31(日) 18:50:53.54ID:OyfTQ5Xf
>>353
え、あなたもどっかのスレッドから持ってきたんですかw?

あなたの回答もおかしいですし、その質問者もずれてますよ
自分がわからないことには回答しない方が良いでしょうね
0355132人目の素数さん垢版2017/12/31(日) 18:58:34.47ID:ky9WzhjK
>>354
ではどう回答すべきですか?
0356132人目の素数さん垢版2017/12/31(日) 19:02:21.94ID:OyfTQ5Xf
>>355
ごめんなさい、問題を勘違いしてました
x=0での連続性を調べろではなくて、ただ連続性を調べろで、x≠0で連続、が回答なんですね

それで良いでしょう
細かいことを言えば、定義域の外では連続性は定義できませんから、x=0では連続でも不連続でもありません
y=xsin1/xは全ての定義域において連続です
0357132人目の素数さん垢版2017/12/31(日) 19:04:49.62ID:xQfsN1Dd
杉浦光夫著『解析入門2』は重積分のところが厳密じゃないそうですが、
松坂和夫著『解析入門6』はどうですか?
0359132人目の素数さん垢版2017/12/31(日) 19:10:22.59ID:42Vx907K
>>356
(イ)の回答はあれだけなのでグラフが回答のメインだと思います。この問題集の他の問題も回答として書かなければならないところは最低限書いてあるので多分グラフメインの回答だとおもいます

ただこの後のページに「微分可能→連続」という公式があったのでそれ使ってもいけるかなておもいまして。
ちなみにグラフがなぜこうなるのかも教えてほしいです
0360132人目の素数さん垢版2017/12/31(日) 19:18:15.27ID:OyfTQ5Xf
>>359
微分可能であるかどうかは積の微分を使うのだと思いますが、それを使うためにはxやsinや1/xが微分可能であることがわかってないといけないので、それは結局連続性がわかっているということになり遠回りですね
微分可能性は連続性よりも強いもしくは狭い概念ですから、連続性だけで議論する方が好ましいでしょう

xが大きいとき、xsin1/x→1ですからあんな感じですね
xが0に近いとき、1/xはどんどん大きくなり続けます
xが0に近づくほど、xの増加に対する1/xの増加は増え続けます
つまり、sin1/xで考えればsinの中身が原点に近づくほど加速度的に増えていくので、振動数が大きくなっていくわけです
さらに、xsin1/x→0ですから、振幅は小さくなっていきます
0361132人目の素数さん垢版2017/12/31(日) 20:44:17.94ID:JK9wza+7
シャーンタラクシタと油井亀美也氏はどっちの方が頭が良いですか?
0362132人目の素数さん垢版2017/12/31(日) 23:21:21.05ID:LoQiGhY7
多変数関数の微分についてですが

とある本に
f(x,y):=a log(x^2+y^2)+b+cx+dy+o(√x^2+y^2) ((x,y)->(0,0))
に対して、
F(x,y):=(x^2+y^2)f(x,y)/{1+(x^2+y^2)f(x,y)^2}
は、a≠0のとき、(0,0)の近傍でC^1級ではあるが、C^2級ではない。

とサラリと書いてあるのですが、簡単に分かるものなのでしょうか?
定義どおり、F_xx、F_xy、F_yyなどを計算するのはすごく大変なような気がするのですが
0363132人目の素数さん垢版2017/12/31(日) 23:28:11.39ID:qB3hGKZg
その程度ですごくと言うなら体力なさ杉
0365132人目の素数さん垢版2017/12/31(日) 23:35:02.55ID:VJq97eKw
なんか人生飽きた。
自殺しようか迷う。
でも、自殺して無になれなかったら嫌だなぁ・・・・・。
無になれないどころか、無間地獄に落ちて永遠に苦を味あわされたりしたら超最悪だしなぁ・・・・・。
自殺するか迷う。
0366132人目の素数さん垢版2017/12/31(日) 23:35:38.11ID:LoQiGhY7
>>363
>>364
ありがとうございます。
やっぱり素直に計算するしかないんですね・・・

(x^2+y^2)log(x^2+y^2)
がC^1級だから、全体もC^1級か?みたいなことを考えていました。
0367132人目の素数さん垢版2018/01/01(月) 11:17:45.22ID:Iz81pdHr
>>362
わりと簡単に分かる
その関数は(x^2+y^2)log(x^2+y^2)の性質で決まってるが、
これはx^2log(x^2)と同じ
微分はx^2だけの微分で決まる
0368132人目の素数さん垢版2018/01/01(月) 12:23:32.85ID:C5IEESt5
馬鹿な質問で悪いんですけど
<と≦の違いって
“はっきりさ”と形容しても正しいですか?
“含む含まない”が一番正しいですかね?
0369132人目の素数さん垢版2018/01/01(月) 12:27:18.97ID:1gKohQS0
因数分解の問題です。
a^2b+ab^2+a+b-ab-1
答えは、(a+b-1)(ab+1)らしいのですが、(ab+1)の+1がどこから湧いてくるのでしょうか?
0371miss type垢版2018/01/01(月) 12:59:21.09ID:ZKO4FBlC
a^2b+ab^2+a+b-ab-1=ab(a+b-1)+a+b-1=(a+b-1)(ab+1)
0372132人目の素数さん垢版2018/01/01(月) 13:27:43.93ID:MtjEXQ7g
A

=

[+a, +b, +c, +d]
[-b, +a, -d, +c]
[-c, +d, +a, -b]
[-d, -c, +b, +a]

の行列式を求めよ。

A * transpose(A) = (a^2 + b^2 + c^2 + d^2) * I

det(A)^2 = (a^2 + b^2 + c^2 + d^2)^4

det(A) = ±(a^2 + b^2 + c^2 + d^2)^2

det(A) の a^4 の係数は 1 であるから、

det(A) = +(a^2 + b^2 + c^2 + d^2)^2
0373132人目の素数さん垢版2018/01/01(月) 13:31:09.42ID:MtjEXQ7g
>>372

のように解答には書いてあります。

言いたいことは分かりますが、この解答は厳密なのでしょうか?

ある場合には、

det(A) = +(a^2 + b^2 + c^2 + d^2)^2

であり、

ある場合には、

det(A) = -(a^2 + b^2 + c^2 + d^2)^2

であるということが起こり得ないことを証明しきれているでしょうか?
0374132人目の素数さん垢版2018/01/01(月) 13:32:29.25ID:MtjEXQ7g
>>372

det(A) を行列式の定義にしたがって計算すれば、
a, b, c, d に関する多項式が得られるからということ
でしょうが。
0375132人目の素数さん垢版2018/01/01(月) 13:34:42.49ID:MtjEXQ7g
a, b, c, d がある条件を満たすときには、
[a, b, c, d に関する多項式] = +(a^2 + b^2 + c^2 + d^2)^2

となる。

a, b, c, d がある条件を満たすときには、
[a, b, c, d に関する多項式] = -(a^2 + b^2 + c^2 + d^2)^2

となる。


↑このようなことが起こらないことを証明していないように思われます。 👀
Rock54: Caution(BBR-MD5:0be15ced7fbdb9fdb4d0ce1929c1b82f)
0376132人目の素数さん垢版2018/01/01(月) 13:35:32.74ID:lOWgLKKm
高校生の問題だろうから…
1.普通に、aについて整理して、そのままたすき掛けで因数分解。
2.どうみても対称式なので、p=(a+b)、q=(ab)と置いて因数分解。

どこから湧いてくるかと言われても、-1 = 1 x (-1) だからとしか言いようがないかも。
0377132人目の素数さん垢版2018/01/01(月) 13:38:26.88ID:MtjEXQ7g
[a, b, c, d に関する多項式1] により定義される R^4 から R への関数 f1



[a, b, c, d に関する多項式2] により定義される R^4 から R への関数 f2

は、

[a, b, c, d に関する多項式1]



[a, b, c, d に関する多項式2]

であるとき、

f1 ≠ f2

であるということを証明しなければならないように思われます。
0379132人目の素数さん垢版2018/01/01(月) 13:48:26.67ID:MtjEXQ7g
det(A) を行列式の定義にしたがって計算した結果が、

+(a^2 + b^2 + c^2 + d^2)^2

とも

-(a^2 + b^2 + c^2 + d^2)^2

とも異なる多項式 p になる可能性を排除しなければならないように思われます。

そして、

a, b, c, d がある条件を満たすときには、
p(a, b, c, d) = +(a^2 + b^2 + c^2 + d^2)^2

となる。

a, b, c, d がある条件を満たすときには、
p(a, b, c, d) = -(a^2 + b^2 + c^2 + d^2)^2

となるということが起こらないことを証明しなければならないように思われます。
0380132人目の素数さん垢版2018/01/01(月) 13:54:12.99ID:OgKz+4d1
>>379
必要条件としてその解を得たわけですから、それら以外の解が出てくることはあり得ません
0381132人目の素数さん垢版2018/01/01(月) 14:02:09.63ID:MtjEXQ7g
>>380

det(A)^2 = (a^2 + b^2 + c^2 + d^2)^4

が得られただけだと思います。
0382132人目の素数さん垢版2018/01/01(月) 14:05:55.13ID:OgKz+4d1
>>381
>>374はダメなんですか?
0383132人目の素数さん垢版2018/01/01(月) 14:11:55.32ID:ZKO4FBlC
すごい考察力ですね。 あるかもしれませんね。
a^4 の係数の比較で終わりにしておけば簡単ですね。

det(A*Transpose(A))=det(A)det(Transpose(A))=(det(A))^2=(a^2+b^2+c^2+d^2)^4

det(A) = det(-A) ただし Aは4x4行列Det(-A])


det(A)= (a^2+b^2+c^2+d^2)^2
det(B)=-(a^2+b^2+c^2+d^2)^2

これ以上は先生に聞いてください。
0384132人目の素数さん垢版2018/01/01(月) 15:45:30.38ID:nv+P5/3t
>>367
>>362です。ご助言ありがとうございます。
ですが、理解できませんでした・・・

・”その関数”とは、fですか?Fですか?
・”(x^2+y^2)log(x^2+y^2)の性質で決まる”とは?
やはり、分子にC^1級関数が含まれていれば、その分数関数はC^1級関数になるのでしょうか?
・”これはx^2log(x^2)と同じ”とは?
t=√x^2+y^2とおいて、fをtの関数としてみるということでしょうか?
0385132人目の素数さん垢版2018/01/01(月) 16:08:55.51ID:aMNEVO1U
>>381
380のコメントで納得いかないなら
連続関数 det(A) : R^4→Rの零点は原点のみなので、原点以外の点では同符号になる。
というのはどう?
0386132人目の素数さん垢版2018/01/01(月) 17:20:45.83ID:C5IEESt5
2つの実数a,b(a<b)についてa<n<bを満たす整数nの個数は-[-b]-[a]-1個

[x]はガウス記号です。これって正しいですかね?あと証明するなら場合分けしかないんでしょうか?
0387132人目の素数さん垢版2018/01/01(月) 17:39:25.20ID:MtjEXQ7g
>>385

ありがとうございました。

p(a, b, c, d) := det(A)

p(1, 0, 0, 0) = 1 > 0

よって、

p(a, b, c, d) ≧ 0

よって、

p(a, b, c, d) = (a^2 + b^2 + c^2 + d^2)^2 for all (a, b, c, d)

となりますね。
0388132人目の素数さん垢版2018/01/01(月) 17:42:09.96ID:WRx3yiBV
>>386
[-b]≦-b<[-b]+1
-[-b]-1<b≦-[-b]
[a]≦a<[a]+1≦n≦-[-b]-1<b≦-[-b]
0389132人目の素数さん垢版2018/01/01(月) 17:42:42.35ID:MtjEXQ7g
>>385

>連続関数 det(A) : R^4→Rの零点は原点のみなので、原点以外の点では同符号になる。

n 変数の連続関数 f が1点においてのみ 0 となるならば

f(x) ≧ 0 for all x

または、

f(x) ≦ 0 for all x

が成り立つ。

というのは n = 1 のときには成り立ちませんが、 n ≧ 2 では成り立ちそうですね。

これはどうやって示すのでしょうか?
0390132人目の素数さん垢版2018/01/01(月) 17:53:37.13ID:ZKO4FBlC
連続関数 det(A) : C^4→Cの零点は原点のみなので、原点以外の点では同符号になる。
というのはどう?
だめね
0391132人目の素数さん垢版2018/01/01(月) 18:14:01.74ID:C5IEESt5
>>388
何ですかこれは?

p,qを整数として、
a<p≦n≦q<b
を満たすような最小のp,最大のqを考えるとする。

nの取り得る個数は、
(q-p+1)個。

pはひとまず[a+1]と表せそう。

qは、
bが整数の場合はb-1、
そうでなければ[b]と表せる。
ただし、bを負に裏返すと、いずれの場合も-[-(b-1)]でまとめられる。

∴q-p+1
= -[-(b-1)]-[a+1]+1
= -[-b]-[a]-1

これであってますかね?
0392132人目の素数さん垢版2018/01/01(月) 18:16:47.10ID:N4dp07d2
ガウス記号の定義[x]≔max{n∈ℤ|n≦x} (x∈ℝ)より、[a]≦a<[a]+1,[-b]≦-b<[-b]+1が従う
又前者は則ち-[-b]-1<b≦-[-b]であるが、此れは-[-b]-1=max{n∈ℤ|n<b}を意味している
同様に、[a]≦a<[a]+1は則ち、[a]+1=min{n∈ℤ|n>a}である
以上より、任意のn∈ℤに対し、a<n<b⇔[a]+1≦n≦-[-b]-1を得る
∴求むるべきは、(-[-b]-1)-([a]+1)+1=-[-b]-[a]-1

ほれ、此れで完璧に示したった
0393132人目の素数さん垢版2018/01/01(月) 18:53:42.37ID:MtjEXQ7g
連続関数 f(x, y) が点 (0, 0) でのみ 0 になると仮定する。

x0 ≠ 0
y0 ≠ 0

とする。

(x0, y0) ≠ (0, 0)

だから、

f(x0, y0) ≠ 0

である。

f(x0, y0) > 0 と仮定する。

y0 ≠ 0 だから、

f(x, y0) ≠ 0 for all x

もしも、 f(x, y0) < 0 となるような x が存在すると仮定すると、
中間値の定理により、 f(x, y0) = 0 となるような x が存在することになってしまい矛盾が起こる。

よって、

f(x, y0) > 0 for all x

同様にして、

f(x0, y) > 0 for all y
0394132人目の素数さん垢版2018/01/01(月) 18:54:18.00ID:MtjEXQ7g
任意の点 (x1, y1) ≠ (0, 0) を考える。

仮定により、

f(x1, y1) ≠ 0

である。

(A) x1 ≠ 0 の場合

f(x1, y0) > 0

であるが、もしも、

f(x1, y1) < 0

ならば、中間値の定理により、

f(x1, y) = 0

となる y が存在することになってしまい矛盾が起こる。

よって、

f(x1, y1) > 0

でなければならない。

(B) y1 ≠ 0 の場合

f(x0, y1) > 0

であるが、もしも、

f(x1, y1) < 0

ならば、中間値の定理により、

f(x, y1) = 0

となる x が存在することになってしまい矛盾が起こる。

よって、

f(x1, y1) > 0

でなければならない。

以上から、

任意の点 (x1, y1) ≠ (0, 0) に対して、

f(x1, y1) > 0

でなければならない。
0395132人目の素数さん垢版2018/01/01(月) 18:54:37.13ID:MtjEXQ7g
f(x0, y0) < 0 と仮定すると上と同様にして、

任意の点 (x1, y1) ≠ (0, 0) に対して、

f(x1, y1) < 0

でなければならないことが導かれる。
0396132人目の素数さん垢版2018/01/01(月) 19:30:51.81ID:MtjEXQ7g
連続関数 f(x, y) が全平面で 0 にならないと仮定する。
2変数連続関数の中間値の定理により、

f(x, y) > 0 for all (x, y)

または、

f(x, y) < 0 for all (x, y)

が成り立つ。
0397132人目の素数さん垢版2018/01/01(月) 19:35:36.85ID:MtjEXQ7g
以上より、連続関数 f(x, y) がたかだか原点でのみ 0 になると仮定すると、

f(x, y) ≧ 0 for all (x, y)

または、

f(x, y) ≦ 0 for all (x, y)

が成り立つ。
0398132人目の素数さん垢版2018/01/01(月) 20:40:26.75ID:MtjEXQ7g
連続関数 f(x1, x2, …, xn) が点 (0, 0, …, 0) でのみ 0 になると仮定する。

y1 ≠ 0
y2 ≠ 0

yn ≠ 0

とする。

(y1, y2, …, yn) ≠ (0, 0, …, 0) だから

f(y1, y2, …, yn) > 0 または f(y1, y2, …, yn) < 0 が成り立つ。

f(y1, y2, …, yn) > 0 と仮定する。

任意の点 (z1, z2, …, zn) ≠ (0, 0, …, 0) を考える。

z1 ≠ 0 と仮定しても一般性を失わない。

(z1, y2, …, yn) ≠ (0, 0, …, 0) だから
f(z1, y2, …, yn) ≠ 0 である。

もし、 f(z1, y2, …, yn) < 0 であると仮定すると、

1変数連続関数の中間値の定理より、

f(x, y2, …, yn) = 0

となるような x が存在することになってしまうが、
(x, y2, …, yn) ≠ (0, 0, …, 0) だから
これは矛盾である。

よって、

f(z1, y2, …, yn) > 0 である。

n-1 変数連続関数 f(z1, x1, …, x(n-1)) の中間値の定理により、

f(z1, z2, …, zn) > 0 である。



同様にして、

f(y1, y2, …, yn) < 0 と仮定すると、

任意の点 (z1, z2, …, zn) ≠ (0, 0, …, 0) に対して、

f(z1, z2, …, zn) < 0 である。
0399132人目の素数さん垢版2018/01/01(月) 21:55:43.83ID:MtjEXQ7g
>>372

は松坂和夫著『解析入門4』に書いてある解答です。

いいかげんな解答ですね。
0401132人目の素数さん垢版2018/01/01(月) 22:15:20.62ID:EAXKzlme
>>400
関数として等しいんでしょ?
0402132人目の素数さん垢版2018/01/01(月) 22:15:48.71ID:EAXKzlme
>>399
ごく普通の回答と思うけど
0403132人目の素数さん垢版2018/01/01(月) 22:47:11.74ID:C5IEESt5
背理法で示せって問題は無限降下法で示しても問題ないですよね??
0404132人目の素数さん垢版2018/01/01(月) 22:58:00.41ID:exES7dYS
>>401
f(x)のxが定義域上の点だとすると、区間Iの端がxになっているのが変に感じます。f(x)は微分可能な関数より区間Iは点にはならない、つまりx≠aだけど、普通にxにaを代入している。なぜ?
0405132人目の素数さん垢版2018/01/01(月) 23:50:13.15ID:0Z8o5sKM
2x=1-√7iのとき x^3 + 3x^2 - 8x + 5の値を求めなさいという問題で

2x=1-√7iよりx^2 - x +2=0より
x^3 + 3x^2 - 8x + 5をx^2 - x +2で割って余りを出して余りに代入すれば良いとあるのですが
ゼロで割り算は駄目という数学ルールがあると思いますが何故この場合は出来るのでしょうか?
0406132人目の素数さん垢版2018/01/02(火) 00:08:39.86ID:xgu4Hu0I
多項式を多項式で割っているからです
多項式を多項式の0では割れません
多項式の値が0になることがあっても、多項式自体が0でなければおっけーなんです
0407132人目の素数さん垢版2018/01/02(火) 00:11:27.42ID:0lWf9w73
y=x−1/xの漸近線を調べる過程を教えてください

y=f(x)とします
lim[x→+0]f(x)=-∞
よりx=0は漸近線である。
y軸に平行でない漸近線を考える。求める漸近線の方程式をy=ax+bとするとlim[x→∞](x-1/x-ax-b)=0となる実数a,bの組を求めればいい
lim[x→∞]{(1-a)x-1/x-b}=0
1-a>0のときは{}の中は無限大となり不適。同様に1-a<0の場合も不適。1-a=0のときlim[x→∞]b=0よりb=0よってy=xは漸近線となる。以上より求める漸近線はx=0,y=x

これってx→-∞も調べる必要ありますか?
まだ漸近線の定義くらいしかしてないのでもしかしたら求め方おかしいかもしれません...
0408132人目の素数さん垢版2018/01/02(火) 00:24:35.44ID:qVLvwhg6
n≧3, nを自然数 aを非負整数とするとき
n^aをn−1で割った余りが1であることを示せという問題についてなのですが
n^a≡1mod(n−1)
⇔n^a−1≡0mod(n−1)を示せばいい
n^a−1=(n−1){n^(a−1)+n^(n−2)+・・・1}
ゆえにn^aをn−1で割った余りは1である
これってあってますか?
同値としてもいいですか?
0409132人目の素数さん垢版2018/01/02(火) 00:30:43.64ID:xgu4Hu0I
良いですよ
0410132人目の素数さん垢版2018/01/02(火) 00:44:23.33ID:vRG+mbsI
>>406
有難うございます

最初は回答見て分からなくて考えてたのですが
多項式を多項式で割るって結局は因数分解?というか目的の多項式で括っただけといいますかそういう作業をしてるので
その後0を代入しても不具合ないようなものでしょうか
0411132人目の素数さん垢版2018/01/02(火) 00:51:00.07ID:xgu4Hu0I
>>410
これ結構ややこしくて、多分ここの回答者でもわかってない人いるんじゃないかって問題なんですね

あなたのいうように、多項式の割り算は因数分解に余りつけたもの、で納得できればそれで良いと思います

多項式の割り算と、普通の割り算は別物ってことなんですよ
(x^2+3)÷x=xあまり3
x=3代入してみると
12÷3=3あまり3
おかしいですよね

多項式の0で割るというのは、こんな感じです
(x^2+3)÷0
これはできませんね
0で割れないのは多項式の場合も同じなんですけど、あくまで多項式の0では割れない、ということです
0412132人目の素数さん垢版2018/01/02(火) 01:06:40.65ID:0lWf9w73
2つの多項式A(x),B(x)に対し
A(x)=B(x)Q(x)+R(x) ...★
(R(x)=0 又は (R(x)の次数)<(B(x)の次数))
を満たす多項式Q(x),R(x)が定まり、
Q(x)をA(x)をR(x)B(x)で割ったときの商、
R(x)をA(x)をB(x)で割ったときの余りと言う。
但しB(x)は定数0ではないとする。

という定義だが、★により商と余りが定義されている。
★は任意の数xについて従う恒等式であるが故に、★の両辺のxには任意の値を代入して良い。
0413132人目の素数さん垢版2018/01/02(火) 01:07:19.36ID:klrRfPqq
>>405
厳密にはゼロで割っているというわけてはなくて、
x^2 - x + 2 = 0 の関係を利用して計算を簡単にするために元の式を
x^3 + 3x^2 - 8x + 5 =(x^2 - x + 2)(x + 4) - 6x - 3
のように変形したいわけですが、これを求めるために割り算の筆算に似た手法を使うというだけのことに過ぎません。
上の式は x^2 - x + 2 = 0 のときも含めて成立する恒等式になっているのがわかるでしょう。
0414132人目の素数さん垢版2018/01/02(火) 01:08:45.40ID:xgu4Hu0I
↑こういうのがわかってない人の典型例ですね
0415132人目の素数さん垢版2018/01/02(火) 01:13:46.28ID:zyzKWjOo
>>407
y=x−1/xの漸近線を調べる過程を教えてください
y=f(x)とします
lim[x→+0]f(x)=-∞
よりx=0は漸近線

感覚的には正しいです、記述はもう少し書いたほうがいいかなと
x=0とy=f(x)が交わらないのというのが漸近線の定義の一つなので、増減表を適当に書いておいたほうが良いですね

高校範囲で厳密にやりたいなら、
「x=0とy=f(x)の、y=tとの交点をP,Qとする。tが十分小さいとき、PQの長さは単調減少で、かつその長さは限りなく0に近づく。よって漸近線である」くらいでしょうか
0416132人目の素数さん垢版2018/01/02(火) 01:16:21.97ID:zyzKWjOo
>>407
追記
交わらないことが必要な理由ですが、
例えば
y=e^(-x)sinxは
x→+∞でy→0
ではy=0は漸近線かというと違いますよね
0418132人目の素数さん垢版2018/01/02(火) 01:20:55.58ID:0lWf9w73
−∞も調べた方がいいですかね
今回は±∞でまとめても問題ないのかな?


漸近線とは曲線が無限遠方へ続く時、曲線との距離が限りなく近づいていく直線であるという定義を意識した解き方をやってみましたが大丈夫ですか?


漸近線の方程式をax+by+c=0・・・@とおくと、曲線上の点(t,t-1/t)と@との距離は
|at+bt−b/t+c|/(a^2+b^2)
=|(a+b)t-b/t+c|/(a^2+b^2)・・・A

1、
t→±0の時、b=0、c=0でA=0となるので漸近線の方程式は
ax=0
x=0

2、
t→±∞の時、a+b=1、c=0でA=0となるので、
ax+by=0
ax−ay=0
y=x

よって1、2より漸近線の方程式は
x=0、y=x
0421132人目の素数さん垢版2018/01/02(火) 01:44:52.60ID:klrRfPqq
わかってるとは思うけど、
x^3 + 3x^2 - 8x + 5 =(x^2 - x + 2)(x + 4) - 6x - 3 のようにするのは剰余 - 6x - 3 を求めるため
この演算は整式の剰余算であって除算とは似て非なるもの
除算はこう
(x^3 + 3x^2 - 8x + 5)÷(x^2 - x + 2)=x + 4 +(- 6x - 3)/(x^2 - x + 2)
これは x^2 - x + 2 = 0 じゃ定義されません
この2つを同じ「割り算」って言葉で表現するから混乱を招く
0422132人目の素数さん垢版2018/01/02(火) 02:06:49.73ID:pzZPkAmk
>>411で触れられている多項式の割り算のようなものがいかに見当外れかよく分かる解説だと思いました
0423132人目の素数さん垢版2018/01/02(火) 02:13:37.09ID:xgu4Hu0I
>>419
多項式の割り算というのはちゃんと定義できてるんですよ
多項式イデアルですね

>>420
なんでですか?
0424132人目の素数さん垢版2018/01/02(火) 02:22:19.24ID:kNe9BAcz
y'=exp(-2xy)
y→a(x→∞)
a>0 を、近似的にとく方法を教えてください
0426132人目の素数さん垢版2018/01/02(火) 02:54:42.16ID:xgu4Hu0I
>>425
割り算ではない、という説明は不適切だということです
多項式の割り算ですから
0427132人目の素数さん垢版2018/01/02(火) 03:11:48.12ID:klrRfPqq
>>426
「厳密には」割り算ではない、と言いました。
なぜゼロで割っても問題ないか、という質問でしたから、ここでおこなっているものが除算とは異なる演算であることを説明する必要があるでしょう
0428132人目の素数さん垢版2018/01/02(火) 03:24:35.76ID:xgu4Hu0I
>>427
>これを求めるために割り算の筆算に似た手法を使うというだけのことに過ぎません。


割り算の筆算、というのはイデアルそのものですね
割り算の筆算に似た、ではなく、割り算なんです
0430132人目の素数さん垢版2018/01/02(火) 03:49:34.86ID:xgu4Hu0I
>>429
どこがダメなんですか?
0431132人目の素数さん垢版2018/01/02(火) 04:01:42.78ID:klrRfPqq
ま、もうややこしいので「割り算」の語は避けますね

一般的にゼロで割れない「除算」という演算は、数A,Bに対して
A=BQとなるような商Qを求める演算です。
B=0のとき、右辺は恒等的に0ですから、Qが一意に定まりません。そのため、除算においては0で割ることを禁止しています。

対して、整式の「剰余算」は、>>412で説明していただいている通り、整式A(x)と一次式以上の整式B(x)に対して
A(x)=B(x)Q(x)+R(x)となるような商整式Q(x),とB(x)より次数の低い剰余整式R(x)の2つを求める演算です
この演算はR(x)の項があることによって、B(x)=0となるようなxがあっても、恒等式となるようにQ(x)とR(x)を機械的に(=代数的に)一意に選ぶことができます
これが、剰余算において、除算とは異なり、除数が0になる場合があっても問題ない理由です。

これでいいですかね?
0432132人目の素数さん垢版2018/01/02(火) 04:11:15.39ID:xgu4Hu0I
任意のxでQ(x)=0のときは、演算は定義できないんですよ
割り算ですよね

多項式のゼロ元は、Q(x)=0ただ一つで、xの値により0になる部分があるだけではゼロ元にならないんですよ
0436132人目の素数さん垢版2018/01/02(火) 10:40:12.45ID:okX91MtS
>>411
>(x^2+3)÷x=xあまり3
>x=3代入してみると
>12÷3=3あまり3
これに代入する人初めて見た
割り算は
x^2+3=x*x+3
x=3代入して
12=3*3+3
問題なし
0437132人目の素数さん垢版2018/01/02(火) 10:46:32.54ID:okX91MtS
>>435
-∞かもしれないしな
0438132人目の素数さん垢版2018/01/02(火) 11:47:07.60ID:0lWf9w73
ありがとうございます

二つ質問です
@左下のマーカー部分についてです。このような発想はどうすればできるのでしょうか…(1)の不等式を用いるんだろうなあ、というのはわかるのですがその使い方がいつも思いつきません。

A右上のマーカー部分についてです
n×log(1/n)肺
となるのではないかと思ったのですが、nは掛けなくていいんでしょうか?


https://i.imgur.com/7bVX7ZD.jpg

https://i.imgur.com/wggcT2v.jpg
0439132人目の素数さん垢版2018/01/02(火) 12:56:59.31ID:CJz2ifze
(1)
x(logx-logy)>=x-y
Let t= x/y
We can prove
log(t) >= t-1 <== t-log(t) <=1

(2)
Let y=1/n in (1)
xi log(xi)>= xi-1/n+xi log(1/n)
Sum[xi logxi]>= Sum[xi]-1/n Sum[1]+ log(1/n)Sum[xi]=ki(1/n)
0440132人目の素数さん垢版2018/01/02(火) 13:33:14.23ID:kThC8xvh
>>438
頭の中で考えられたらいいことは
・x(log(x)-log(y))=xlog(x)-xlog(y)だからx=x[i]とすればよさそう
・Σ[i=1,n]x[i]=1だからiを動かして足せばよさそう→不等式の右辺にx単体があるからできそう
・あとは愚直にy=1/nとすれば・・・

(2)は定数a=log(1/n)に対して
 Σ[i=1,n]x[i]*a=aΣ[i=1,n]x[i]
というふうにaをΣの外に出してるだけ
0441132人目の素数さん垢版2018/01/02(火) 13:53:52.26ID:xgu4Hu0I
>>436
小学校の割り算で何を習って来たんでしょうね
0442132人目の素数さん垢版2018/01/02(火) 13:58:05.74ID:okX91MtS
>>441
ここは小学校の割り算を云々するところですか?
0443132人目の素数さん垢版2018/01/02(火) 14:06:33.06ID:xgu4Hu0I
>>442
整数のユークリッド環としての性質を論じる場所ですね
0444132人目の素数さん垢版2018/01/02(火) 14:07:49.10ID:okX91MtS
>>443
違うのでは?多項式の割り算の話だったでしょう?
0445132人目の素数さん垢版2018/01/02(火) 14:11:53.60ID:xgu4Hu0I
>>444
多項式もユークリッド環ですからね
0446132人目の素数さん垢版2018/01/02(火) 14:13:14.45ID:PpYNA0rh
>>438
(2)はエントロピーの式ですね。信号理論等で使われます。
(2)を解くのにどういう発想をするか、ということですが、(1) の式の x と y を一気に置き換えようとしなくてもよいように思います、
(2)の左辺は Σ(x_i・log x_i)ですが、(1) の式を x・log x≧x-y+x・log y と変形するとこの左辺が (2) の左辺に適用できそうです。
まずx=x_iを代入して辺々足すと
Σ(x_i・log x_i)≧Σ(x_i-y+x_i・log y)=Σ(x_i)-Σy+Σ(x_i・log y)=Σ(x_i)-ny+Σ(x_i)・log y…@です。
Σ(x_i)=1 を使うと、
Σ(x_i・log x_i)≧1-ny+log y
となります。@のlog y の係数がΣ(x_i)であったため n が掛かっていないことに注意してください。
ここで、1-ny の項を消すために y=1/n を代入します。すると
Σ(x_i・log x_i)≧log(1/n)となります。
一気に考えると混乱する場合でも、このように段階的に考えればわかりやすいと思います。
0447132人目の素数さん垢版2018/01/02(火) 14:16:32.81ID:okX91MtS
>>445
別のユークリッド環ですね
0448132人目の素数さん垢版2018/01/02(火) 14:18:38.36ID:xgu4Hu0I
そーですね
それらが別物であるということを示すための説明が、>>411ですね
0449132人目の素数さん垢版2018/01/02(火) 14:23:34.00ID:okX91MtS
p_a:R[x]->Rを
p_a(f(x))=f(a)とすると
環準同形になりますが
ユークリッド環としての性質は保存されません
あなたが
>>411
>多項式の割り算と、普通の割り算は別物ってことなんですよ
>(x^2+3)÷x=xあまり3
>x=3代入してみると
>12÷3=3あまり3
>おかしいですよね
と書いたのもそのことを指摘したのだと思いますが
p_aを使う方が普通だと思いますので
>>436
>これに代入する人初めて見た
と書いたわけです
0450132人目の素数さん垢版2018/01/02(火) 14:27:10.45ID:xgu4Hu0I
>>449
普通だろうがなんだろうが、結局同じことですよね?
意味的には
異なることを示す例としてはわかりやすいかと思ったんですよ
元の式に戻すと、因数分解した形に意味があるというような誤解が生じる可能性がありますからね
構造そのものが異なっているんですから
0451132人目の素数さん垢版2018/01/02(火) 14:36:20.97ID:PpYNA0rh
問題の要点を外れて徒に厳格な定義を持ち出して問題を複雑化することは、
数学嫌いの学生を増やすのではないかということが心配です。
0453132人目の素数さん垢版2018/01/02(火) 14:42:07.68ID:m3zRkaWc
ここは数学教育スレではありません
0454132人目の素数さん垢版2018/01/02(火) 14:43:30.67ID:xgu4Hu0I
>>453
まーた、わからない問題を書くだけで質問スレではない、ってやつですか?
恥ずかしくないんでしょうかね
0455132人目の素数さん垢版2018/01/02(火) 15:37:31.52ID:PpYNA0rh
>>452
等号が成立する条件の考え方は元の画像にも書いてあるとおりです。
(1)で x(log x-log y)≧x-y であることを利用しましたので、
この式に x=x_i, y=1/n を代入したとき、どれか一つでも両辺が等しくないものがあると、
総和をとったときに等号が成立しません。
また、(1)の式の等号は x=y のときのみです。
以上のことより(2)の等号はすべての x=x_i, y=1/n について x=y の場合であり、
すなわち、すべての x_i について x_i=1/n のとき、ということになります。
0456132人目の素数さん垢版2018/01/02(火) 15:38:12.30ID:0lWf9w73
納得しました!!


左側中段のここで、S-T=h(t)とおくとってとこからなんですけど、
t>0において、t=√2/4でh(t)が最大なのはわかるんですけど、その後の|S-T|=√2/24のときの計算で
S-T=-√2/24のときの因数分解がわかりません。
右側中段の点Aのt座標について、の下の式はたてれたんですけど、この式を因数分解するにはt=-√2/4が解の一つであると言えないとできないと思うんです…
なんでt=-√2/4が解の一つになると言えるのか教えて欲しいです

「a^2-b^2=(a+b)(a-b)
を利用すれば可能だけど、式が複雑でわかりづらい場合は次のように考えると良い

h'(t)=0を満たすtを求めたい
-2x^2+1/4=0
2x^2=1/4
x^2=1/8

このような変形をすればh'(t)=0の解が1/8の平方根であることは明らか」

ということだそうですが
h'(t)=0の解が1/8の平方根であるのはわかるんですけど、±で2つ解がでてくるじゃないですか?
なんで"-"のほうってわかるんですか?


https://i.imgur.com/VUkOX5w.jpg
0458132人目の素数さん垢版2018/01/03(水) 00:34:17.23ID:bmR7MaiG
>>457
意図が分かりづらいので、
点AとかAB=3,AC=4とか、直角三角形とか、円上に点Eがとか
記述してもらえると助かります

考察したいのは3枚目の画像の図形ということですか?
0459132人目の素数さん垢版2018/01/03(水) 00:58:20.96ID:ZwC7n7D7
>>458
そうです
点の名前は回答者様が自由に設定した方がいいかと...
お願いします
0461132人目の素数さん垢版2018/01/03(水) 01:13:41.55ID:ZwC7n7D7
直角三角形の3点にあるピン3つと鉛筆を、糸で作った輪っかの内部に引っ掛けて、鉛筆を動かして作図します。
ただし糸はたるませないです。

そうして出来上がった円と三角形の位置関係は最後の写真ようになります。
0462132人目の素数さん垢版2018/01/03(水) 01:40:15.82ID:AjRm2w/O
糸を2点に引っ掛けて楕円を書くやつの、3点版ってことであってる?
糸の長さは6πってこと?
0463132人目の素数さん垢版2018/01/03(水) 01:49:29.46ID:ZwC7n7D7
>>462
はい
0464132人目の素数さん垢版2018/01/03(水) 02:02:44.43ID:oE7rIqyx
>>463
いやいや、なんで糸の長さ6πとかいう設定にしたかなあ…
せめて糸の長さ18とかならまだやる気が起きるが。
ちなみに、できる曲線は6つの楕円の弧をつなぎ合わせたものになるのはわかるよね?
0465132人目の素数さん垢版2018/01/03(水) 02:07:31.60ID:3ye3FlM+
宇宙飛行士と飽和潜水士はどっちの方が凄いですか?
0466132人目の素数さん垢版2018/01/03(水) 02:07:54.64ID:QH4adFi9
神の方がすごいですね
0467132人目の素数さん垢版2018/01/03(水) 02:09:03.45ID:ZwC7n7D7
糸の長さは半径3の円周なので6π。
三角形の斜辺は三平方の定理より5。
書かれる線を動点Pの軌跡とします。
これをグラフ上に表すと添付写真のようにまとめられます。

左上にPがあるとき、糸はBからCを経由しAに至っており、AとBからPまでは糸が張られた状態です。
よってAP+BP=6π-BC-AC=6π-7

次にその右上の、直線ACと直線ABに囲まれた象限にPがあるとき、糸はBとCから張られておりAには触れていない状態です。
よってBP+CP=6π-BC=6π-4

以下同様に6つのエリアすべてを式にすることができます。

あとはそれぞれのエリアについて点Pの軌跡を式で表し、積分して面積を出す

って方針ですよね
0468132人目の素数さん垢版2018/01/03(水) 02:12:11.21ID:3ye3FlM+
>>466
神でも無には勝てませんよね?
0469132人目の素数さん垢版2018/01/03(水) 02:15:45.47ID:QH4adFi9
>>468
いいえ、神が最強ですね
0470132人目の素数さん垢版2018/01/03(水) 02:19:16.10ID:3ye3FlM+
>>469
理由を教えてください。
0471132人目の素数さん垢版2018/01/03(水) 02:22:04.88ID:QH4adFi9
>>470
神だからです
0472132人目の素数さん垢版2018/01/03(水) 02:24:54.71ID:3ye3FlM+
>>471
もう少し詳しく教えてください。
0474132人目の素数さん垢版2018/01/03(水) 02:40:08.87ID:QH4adFi9
>>472
神は最強なんですね

>>473
東大は今日は休みなんですか?
0475132人目の素数さん垢版2018/01/03(水) 02:46:18.22ID:tNteplmz
https://i.imgur.com/SiAZZut.jpg

すみません。画像のように計算したんですけど、答えは「-arctan(3x)」になってました。

どこかを間違えてると思うのですが、どこを間違えてるのか分かりません。解説お願いします。
0476132人目の素数さん垢版2018/01/03(水) 02:59:49.80ID:oE7rIqyx
>>475
なんでarctanとか知ってるのに、高校レベルの積分の基本を理解してないのだろう。

f(x)の原始関数の一つがF(x)のとき
∫f(ax+b)dx = (1/a)F(ax+b)+C (Cは積分定数)
となるって話の1/aが抜けてるだけ。

高校数学を勉強し直すべき。
0477132人目の素数さん垢版2018/01/03(水) 03:07:14.55ID:tNteplmz
>>476
あー。そーでしたそーでした。
ありがとうございます。
0478132人目の素数さん垢版2018/01/03(水) 03:29:12.49ID:aB1KIcQu
>>474
なぜ神は最強なのかを教えてください。
0479132人目の素数さん垢版2018/01/03(水) 08:54:13.41ID:HBQ6kBGH
水素水がすごいん
0480132人目の素数さん垢版2018/01/03(水) 08:57:43.15ID:f4nthuXW
>>474
答えられないんですね(笑)
神云々で荒らすのはもうちょっと知能をつけてきてからでお願いします
0481132人目の素数さん垢版2018/01/03(水) 11:18:33.06ID:P/ExIS0m
このスレ次立てる人、スレタイは「分からない数学の問題は…」とかにして欲しいかな
0483132人目の素数さん垢版2018/01/03(水) 11:57:11.20ID:Ylmj5M1i
>>467
2定点からの距離の和が一定となるような点の軌跡は、
ひとまず楕円になるw

全体の軌跡は6パターンの楕円の一部を切り取ったものの集合体になるw
なので6パターンの楕円の一部の面積をそれぞれ求めて足し合わせれば
いけるはずだが、角度がラジアンに直しにくいので非常にしんどいw
0484132人目の素数さん垢版2018/01/03(水) 17:42:57.69ID:P/ExIS0m
>>483
6つの領域の境界は三角形の各辺を含む直線であって、糸の長さで決まるものじゃないから、糸の長さが無理数であることと、問題の複雑さはあまり関係ないようにも思う
0485132人目の素数さん垢版2018/01/03(水) 21:44:35.92ID:oE7rIqyx
複雑さには関係なくても、面積求めるにはどう考えても逆三角関数が必要な問題で、
その逆三角関数に突っ込む値にπが含まれてるとか、無意味にごちゃごちゃ
させてるだけでしょうに。
0486132人目の素数さん垢版2018/01/03(水) 22:15:09.20ID:Z2VJpcac
俺は電卓じゃないから計算しないけど、文字を適当に置きながらやれば、出来るかなぁ。
正直面倒なだけの問題にしか見えないけど、なんかいい方法ありそう?
0487132人目の素数さん垢版2018/01/03(水) 23:26:54.48ID:ZwC7n7D7
ちなみに…高校数学の知識のみで解けますかね…?
0488132人目の素数さん垢版2018/01/03(水) 23:40:28.79ID:Z2VJpcac
逆三角関数だけ使えれば。
例えば、「αはcosα=3/5 sinα=4/5 を満たす角。」とか。
これを使えば高校レベルで多分いけるけど、自分の見積もりでは半端じゃない面倒くささ。

先の方針で解けるはずだけど、なんかクールな方法があるそうな雰囲気もある。
ので、この話題に付き合ってる。
0489132人目の素数さん垢版2018/01/04(木) 10:31:30.45ID:CxAihJ0+
考えてみたが、とりあえず糸の長さを変数として一般式で求めておいて具体的な値は後で代入すればよい
求める6つの領域の面積は各々楕円の扇形に、三角形の部分を2つずつ足したり引いたりして得られる
このうち三角形の部分は底辺と高さが同一のものが正負2つ組で存在するため合計するとすべてキャンセルされて無くなる
結果としては6つの楕円の扇形の面積を合計すればよい
扇形の楕円はそれぞれ焦点が元の三角形の頂点であり、糸の長さと三角形の周の差から離心率が求められるので、
真円に引き伸ばした時の中心角がわかれば求められる
中心角は三角関数と逆三角関数で求めればよさそう
と、ここまでわかった
0491132人目の素数さん垢版2018/01/04(木) 11:43:59.73ID:p4vxFKTx
極座標についての質問です。点P(x,y)の位置ベクトルrをとるとき、なぜ、

r=xe_x+ye_y

になるのでしょうか。
また、

e_r=cosθe_x+sinθe_y

となる理由が純粋にわからなくなりました。

教えてください。
0492132人目の素数さん垢版2018/01/04(木) 11:50:29.98ID:Hj9rtC2t
A(1.3)B(2.4)の直線媒介定数を求めよ。
これってBを基準にしても正解ですか?
またその下に書いてる問題ではどちらを基準にしても良いのでしょうか?

媒介定数t〜のくだりが無かったらxyの式で表すという認識で良いのかな?


https://i.imgur.com/vP8vmZG.jpg

https://i.imgur.com/Daasgcf.jpg
0493132人目の素数さん垢版2018/01/04(木) 11:58:31.74ID:HLyTTIMA
>>491
まずベクトルを勉強してから来て
0495132人目の素数さん垢版2018/01/04(木) 13:05:17.06ID:C7qqGfS/
>>489
こうかな
元の三角形の頂点をA,B,C、辺BC,CA,ABの中点をそれぞれO,P,Qとし、
辺の延長と問題の図形との交点を、
直線BC上でBに近い側をB1、Cに近い側をC1、
直線CA上でCに近い側をC2、Aに近い側をA2、
直線AB上でAに近い側をA3、Bに近い側をB3とする
求める面積のうち、三角形ABCの外側の面積S_outerは、辺を含む直線上の線分と楕円弧を使って以下のように表される
S_outer = BB3⌒C2C + CC2⌒C1 + CC1⌒A3A + AA3⌒A2 + AA2⌒B1B + BB1⌒B3
図形BB3⌒C2Cは、扇形OB3⌒C2と三角形BOB3,COC2の3つに分割できる。
図形CC2⌒C1は、扇形QC2⌒C1から三角形CQC1,CQC2を除いたものである。
同様にして以下の式を得る
S_outer = BOB3 + OB3⌒C2 + COC2 - CQC2 + QC2⌒C1 - CQC1 + CPC1 + PC1⌒A3 + APA3 - AOA3 + OA3⌒A2 - AOA2 + AQA2 + QA2⌒B1B + BQB1 - BPB1 + PB1⌒B3 - BPB3
これらのうち、三角形COC2とCQC2は、底辺CC2が共通で高さが等しいため面積が等しい
同様に面積が等しい三角形を相殺すると以下の式を得る
S_outer = OB3⌒C2 + QC2⌒C1 + PC1⌒A3 + OA3⌒A2 + QA2⌒B1B + PB1⌒B3
求める面積は、この楕円扇形6つの合計であるS_outerと三角形ABCの面積の和である
0496132人目の素数さん垢版2018/01/04(木) 13:45:49.60ID:ss/HswBA
>>492
ここにはこんな難しい質問に答えられる人はいないよw
0498132人目の素数さん垢版2018/01/04(木) 19:30:59.94ID:ZgRpzAUK
>>492
媒介変数表示といっても、ただ一つしかないわけではなく色々あります
基準を変えれば答えが変わるということも当然あるでしょうね
0502132人目の素数さん垢版2018/01/05(金) 03:07:07.82ID:RDCuTZmX
空間ベクトルの問題でかなり難しいもの、解ける方いたりしますか?
自分では全くわかりません…


https://i.imgur.com/eoOlvCL.jpg
0503132人目の素数さん垢版2018/01/05(金) 04:25:04.09ID:hW9ZHYt/
>>499
(a-1)^2+(b-1)^2=?
0504132人目の素数さん垢版2018/01/05(金) 04:59:40.11ID:cwrLWsZu
アルティンガロア理論の訳本p46定理15に「EをKの正規拡大体でその自己同型群をGとする。するとEはKの分離拡大体である。」と意味がわからないことが書かれているのですがどういうことでしょうか?
またネットで読める英語版と日本語訳版では色々と違うのですが日本語版は何を元にしているのでしょうか?
0506132人目の素数さん垢版2018/01/05(金) 05:16:29.48ID:jYU6KwYb
すべての x ≧ 0 に対して、x^3 - 3x ≧ k (3x^2 - 12x - 4) が成り立つ定数 k の値の範囲を求めよ。(慶応大)
ていう問題がある本にのっていて、答えは途中経過なしでただ 1/4 ≦ k ≦ (3+√13)/2 と書かれていたのですが、これ問題か答えのどちらか間違っていないですかね?
私の勘違いであれば、誰か解説していただけると非常にありがたいです。
0508132人目の素数さん垢版2018/01/05(金) 07:41:56.60ID:hOwI1AnF
>>506
何でてめえのために5分10分かけて解答確かめなきゃいかんの?
お前の書いた解答見せてから物を言え
バーカ
0509132人目の素数さん垢版2018/01/05(金) 07:56:57.44ID:hOwI1AnF
ID:jYU6KwYbはNG推奨、他のスレで荒らしているのを確認しました

ところで、abc予想に関する傑作問題を解いてくれませんか?

正の整数a,b,cはa+b=cを満たし、a,b,cのどの2つの整数も互いに素である。
また、d=ab+bc+caとする。
dが素数となるようなa,b,cの組は無数に存在するか。
0513132人目の素数さん垢版2018/01/05(金) 10:29:22.94ID:z4TBWLRD
カンタベリー大主教とアラブの石油王はどっちの方が格上ですか?
0514132人目の素数さん垢版2018/01/05(金) 10:44:07.67ID:z4TBWLRD
六法全書とか日本国語大辞典とか広辞苑とか現行日本法規全巻とか現行法規総覧全巻などを全て丸暗記したらどんなご利益がありますか?
あと、前になんかのスレッドで、記憶力が良い人は理解力も高いみたいな書き込みがあったのですが、それは本当ですか?
本当だとしたらなぜそうなのかを詳しく教えてください。
0515132人目の素数さん垢版2018/01/05(金) 11:19:29.38ID:2i0MaZDm
フェルマーテストで
2^(n-1) (mod n)
でnが合成数の時2^(n-1) (mod n)が1になる確率は
1/nくらいですか?
0516132人目の素数さん垢版2018/01/05(金) 12:11:15.15ID:7ZNncoy1
凸多角形Pの各辺bに対して、bを1つの辺とする三角形であってPに含まれるものの面積の最大値を割り当てる。この凸多角形Pの各辺に割り当てられた面積の和は、Pの面積の2倍以上であることを示せ。

教えてください
0517132人目の素数さん垢版2018/01/05(金) 12:31:47.08ID:Gp//NrYm
>>515

ちゃうとおもうよ
n=1000 で17%だから

でもa^(n-1)==1 mod n,n=2..100,でa を2から30ぐらいまで動かすと
0.27近辺になるね。
0519132人目の素数さん垢版2018/01/05(金) 12:38:20.69ID:Gp//NrYm

n=2..5000にすると0.14になる。

整数論で確立というのは、考え方によるね
現実的確率と思考実験の確率がかんがえられるね。
0520132人目の素数さん垢版2018/01/05(金) 13:02:22.31ID:i0ZGVGdI
>>515
その場合は確率というより、式を満たすか満たさないか、ではないの?
もしくは、2だけでなく任意のa(=1,2,3,...,n-1)を選んだとき、a^(n-1)≡1を満たすaの個数/(n-1)がどういった値に近づくか、みたいな感じの。

nが偶数のとき
2^(n-1)=2·2^(n-2)≡1(mod n)
2^(n-2)は2の逆元だが、2とnは互いに素ではないのでこの式は成り立たない
0521132人目の素数さん垢版2018/01/05(金) 13:13:53.25ID:jYU6KwYb
>>507
どうもありがとうございました。問題を直したら書いてあった答えが出せました。もうひとつ、
(a^2 - b^2)^2 - 2(a^2 + c^2) + c^4 を因数分解せよ
という問題があってわからないのですが、これも問題が間違っていますか?こちらは解答がのってないんです。
どちらの問題も、「コンピュータ時代の入試数学」という、入試数学についての本に応用問題の例としてのっていたので、問題集からじゃないんです。
>>509
ウソを書かないでください。数学板への書き込み自体初めてです。実際にIDを調べてもらえればすぐわかります。
0522132人目の素数さん垢版2018/01/05(金) 13:36:33.98ID:jYU6KwYb
>>516
「凸n角形Pの任意の一辺を除いた計n-1個の各辺について、その辺を一辺とする三角形でPに含まれるものの面積の最大値の合計がPの面積の2倍以上である」ことを示せば十分。これをnに関する帰納法で示します。
n = 3の場合、Pの各辺を一辺とするPに含まれる最大の三角形はP自身。これを2つの辺について合計すればちょうどPの面積の2倍。
次に、証明したいことが3 ≦ n ≦ kとなるすべてのnで成立するとします。凸(k+1)角形は任意の対角線を引くと2つの凸(k以下)角形に分けられます。この2つの凸(k以下)角形に対して、今引いた対角線以外の辺を考えて帰納法の過程を使えば良いです。
0523132人目の素数さん垢版2018/01/05(金) 13:45:07.71ID:jYU6KwYb
書いたばっかですみませんが、これ間違ってました。1辺を除いてしまうのはダメですね。
0525132人目の素数さん垢版2018/01/05(金) 13:59:20.01ID:R3Rbplq0
>>504
お願いします
0526132人目の素数さん垢版2018/01/05(金) 14:12:26.77ID:jYU6KwYb
>>524
どうもありがとうございました。やっぱり問題が間違ってたみたいですね。こんな便利なサイトがあったとは!
0528132人目の素数さん垢版2018/01/05(金) 17:30:07.71ID:F9/Q1Eod
2変数関数の極値計算で、H=(fxx)(fyy)-(fxy)^2と0との大小を確認し、H=0になった時の極値かどうかの判断ってどういった方法がありますか?
0529132人目の素数さん垢版2018/01/05(金) 19:29:29.58ID:E5LBzWbF
阿原一志著『計算で身につくトポロジー』を読んでいます。

蛇の補題を5ページも使って説明しています。

本を見ずに証明を試みたところ、非常に簡単に証明できました。

その後、本の証明を見たところ、こんな記述がありました:

「ここで、 v - v' を考えるのがコツである。とはいえ、なぜ v - v' を考えつかなければいけないかは
茫漠としているが、以後の長い議論を経て、この取り方が唯一の解決であることが後でわかる。
(一流の数学者はこのくらいの証明はひと目で読みきれる、ということのようだ。)」

阿原さんの感覚は大丈夫なのでしょうか?
0530132人目の素数さん垢版2018/01/05(金) 22:59:01.24ID:/nh60RMv
史上最大の素数を発見。50番目となるメルセンヌ素数は、原稿用紙5万8000枚分
1/5(金) 18:08配信 ハフポスト日本版
https://headlines.yahoo.co.jp/hl?a=20180105-00010002-huffpost-int

史上最大の素数が発見された。なんと2324万9425ケタだ。400字詰め原稿用紙に書き起こすと、5万8000枚近く必要になる。

探査プロジェクト「GIMPS」は1月3日、史上最大の素数を発見したと公式サイトで発表した。

今回発見された数字のコードネームは「M77232917」。2×2×2×2×2...といった具合に、2を7723万2917回掛け合わせた数から1を引いた数に当たる。数式にすると以下の通り。

これは、2016年1月に発見された素数よりも、ほぼ100万ケタ大きいという。

史上最大の素数を発見。50番目となるメルセンヌ素数は、原稿用紙5万8000枚分
Huffpost Japan
GIMPSとは?
「GIMPS」は、インターネットを介した分散コンピューティングによって、史上最大の素数を探すプロジェクトで、1996年に設立された。

GIMPSが探す素数は、メルセンヌ素数と呼ばれるものだ。2×2×2×2×2...といった2を掛け合わせた数字から1を引くことで表せる。
世界中のメンバーがコンピューターを使って探している。今回発見されたのは50番目のメルセンヌ素数だ。

発見者は数学ファンのジョナサン・ペースさん。アメリカテネシー州に住む51歳の電気技術者で、
フェデックスに勤務している。これまで14年にわたってGIMPSプロジェクトにボランティアとして参加していたという。ペース氏には、GIMPS研究発見賞として3000ドル(約34万円)が贈られるという。

実際に数字を並べてみると......。
今回見つかった「M77232917」の正確な数字は、GIMPSの公式サイトがZIPファイルで公開している。


リーマン予想は他の数学の問題とは格が違うんですか? 👀
Rock54: Caution(BBR-MD5:0be15ced7fbdb9fdb4d0ce1929c1b82f)
0532132人目の素数さん垢版2018/01/06(土) 00:26:15.62ID:0Kt01Ob4
レポート丸投げの季節か
0533132人目の素数さん垢版2018/01/06(土) 01:06:03.53ID:ioRU9Isi
>> 521
そのタイトルの本にそういう問題が載っていることは調べたら出てきた
>(a^2 - b^2)^2 - 2(a^2 + c^2) + c^4 を因数分解せよ
斉次じゃないのがちょっと気持ち悪い
(a^2 - b^2)^2 - 2(a^2 + b^2)c^2 + c^4 を因数分解せよ
だったらそんなに難しくなくて結果も綺麗
あと、その下の x^3+y^3-3xy+1 なんかは斉次じゃないけど綺麗
0534132人目の素数さん垢版2018/01/06(土) 01:12:44.50ID:e/dk8iN6
コンピュータ時代の入試数学ということなら、ネット情報を活用して問題を解くのも
個人のスキルの一つだから、wolframalphaあたりで計算してもらうのもありじゃないのか。
0535132人目の素数さん垢版2018/01/06(土) 01:15:07.09ID:0H7ls88C
将来自殺している未来しか思い浮かべることができないんですけどどうすれば良いですか?
今死んだほうがいいんでしょうか?
0536132人目の素数さん垢版2018/01/06(土) 01:28:26.19ID:Cd7HZ2B3
>>533
x^3+y^3-3xy+1 は斉次式 x^3+y^3+z^3-3xyz の特殊形
ωを使って3つに分解するか、そうせずに2次式を残すか
高校レベルなら後者かな
0537132人目の素数さん垢版2018/01/06(土) 03:55:31.35ID:DiOH1yHL
αを複素数とするとき、
積分(-α^2)sinα dα を求めよ。
ただし、原点を中心とする1と-1を直径とする半円(虚部が非負のもの)にこの直径を連結させた閉曲線をCとする。
積分路はCである。
0538132人目の素数さん垢版2018/01/06(土) 04:27:41.86ID:nlggsK6z
世界一の数学の天才になりたい。
0539132人目の素数さん垢版2018/01/06(土) 08:56:42.58ID:sXQdObAU
ちょっと聞きたいんすけど0と1の間は存在しますか?
0540132人目の素数さん垢版2018/01/06(土) 09:18:53.09ID:Cd7HZ2B3
>>537
(-α^2)sinα に特異点が存在しない以上、積分路によらずコーシーの定理で0になる
この問題はどこから持ってきたの?

>>539
対象とする集合による
整数で考えたら間はない
実数や有理数で考えたら間がある
それ以前に順序関係が定義されていなければ間という概念そのものがない
0541132人目の素数さん垢版2018/01/06(土) 09:38:25.50ID:sXQdObAU
>>540
回答ありがとうございます。
0542132人目の素数さん垢版2018/01/06(土) 10:26:37.34ID:A29tepm3
tanx=√2のとき、2cos^2x-sin^2x=ア
詳しく解説お願い致しますm(__)m解説みてもわかりませんでした
0543132人目の素数さん垢版2018/01/06(土) 10:45:33.35ID:ioRU9Isi
>>542
いつもこの方法が使えるわけではないが
この問題はtan x=sin x/cos x の関係を使えば簡単に解ける
0545132人目の素数さん垢版2018/01/06(土) 11:42:42.87ID:4W4NFvTb
tanが分かってるんだからせめてcosで割ることくらいしろよと思ったが、解説見ても分からないってそれ三角関数の定義が分かってないんじゃない
教科書読め
0546132人目の素数さん垢版2018/01/06(土) 11:47:24.02ID:RbumRdWp
>>516
これ解けないんだが誰か解けたやつおる?
0547132人目の素数さん垢版2018/01/06(土) 13:21:59.29ID:Cd7HZ2B3
>>543
tan x がわかっていればその式と (sin x)^2+(cos x)^2=1 から (sin x)^2 と (cos x)^2 の値が一意に定まる
あとは代入するだけだから差がゼロでなくてもその方針でいい
0548132人目の素数さん垢版2018/01/06(土) 13:37:15.30ID:hzr+D9ld
>>533
>(a^2 - b^2)^2 - 2(a^2 + b^2)c^2 + c^4 を因数分解せよ
なるほど、きっとそれの書き間違いだったのでしょうね!
この本けっこうつっこみどころが多くて、「教科書の最上級レベル」の例として
(a + b + c)(ab + bc + ca) - abc
があって、それよりひとつ上のレベルの「受験用学習書の中級レベル」の例として
a^2(b + c) + b^2(c + a) + c^2(a + b) + 2abc
があるけど、展開するとどっちも同じだし、むしろ上の方が展開が面倒なぶん難しく感じる
0552132人目の素数さん垢版2018/01/06(土) 20:22:23.53ID:A29tepm3
>>547意味がわかりませんすみません
Sin^2+cos^2=1

tan=sin/cosなのはわかります
0554132人目の素数さん垢版2018/01/06(土) 20:59:59.10ID:Cd7HZ2B3
>>547のヒントで考えてほしかったんだけどね
tan x=√2なのでsin x/cos x=√2
よってsin x=√2・cos x
これを(sin x)^2+(cos x)^2=1に代入して(√2・cos x)^2+(cos x)^2=1
よって2(cos x)^2+(cos x)^2=1さらに整理して3(cos x)^2=1
ここから(cos x)^2=1/3
これを(sin x)^2+(cos x)^2=1に代入して(sin x)^2+1/3=1
ここから(sin x)^2=2/3
あとはこれらを元の式に代入してくださいな
0555132人目の素数さん垢版2018/01/07(日) 09:42:44.60ID:DCkGrmwK
二次関数のグラフが次の3つの(x.y)座標点をとおるとき
0556132人目の素数さん垢版2018/01/07(日) 09:44:19.44ID:DCkGrmwK
二次関数のグラフが次の3つの(x.y)座標点をとおるときの二次関数は次のうちどれか
(-2.16)(1.1)(3.21)
0557132人目の素数さん垢版2018/01/07(日) 09:44:57.74ID:DCkGrmwK
>>556
教えて下さい
0561132人目の素数さん垢版2018/01/07(日) 13:45:33.94ID:5ZRynvi7
ボケの方が偉い
0563132人目の素数さん垢版2018/01/07(日) 17:38:56.55ID:RLnz3oMJ
解析的に解ける確率微分方程式(幾何ブラウン運動など)のよい解き方あるいは解き方をまとめたサイトがあれば教えてください
普通の常微分方程式は定数変化法や変数分離法とか線形ならいろいろ解き方がまとまって見つかるのですが
確率微分方程式だとごくわずかなものしか見つからず、この条件を満たす方程式ならこのやり方で解けるみたいのが知りたいです
0565132人目の素数さん垢版2018/01/07(日) 17:58:31.61ID:5ZRynvi7
stochastic differential equations をarxivでggする
0566還暦FX垢版2018/01/08(月) 00:05:21.00ID:c93nVah2
2以上の整数を2以上の整数の和で表す方法
2=2で1通り
3=3で1通り
4=2+2=4で2通り
5=2+3=3+2=5で3通り
6=2+2+2=2+4=4+2=3+3=6で5通り
フィボナッチ数列になりそうだが、その理由は何でしょうか?
0570還暦FX垢版2018/01/08(月) 00:40:16.81ID:c93nVah2
>567  4から先でもいいです。
0571還暦FX垢版2018/01/08(月) 00:42:44.03ID:c93nVah2
>568 反例は何でしょうか?
0572132人目の素数さん垢版2018/01/08(月) 01:08:30.92ID:AWKuaxkV
>>569
どゆう意味ですか?上のひとの解説も???でした
0573132人目の素数さん垢版2018/01/08(月) 01:34:48.63ID:6yE9/RTC
>>566 >>570

nをk個の自然数の和で表わす方法を q_k(n)とおく。
求めるものは
Σ[k=1,[n/2]]q_k(n-k)

これは漸化式
q_k(n)= q_{k-1}(n-1)+ q_k(n-k),q_1(n)= q_n(n) = 1
を満足して、生成関数
Σ[n=k,∞]q_k(n)x^n =(x^k)/{(1-x)(1-xx)…(1-x^k)},
をもつ。

数セミ増刊「数学100の問題」日本評論社(1984) p.58
0576132人目の素数さん垢版2018/01/08(月) 04:18:30.36ID:V+Ra263X
次の問題が分からないので教えてください。

実数a,bに対して、次の条件を満たす二項演算#を考える。
・実数a,bに対し、a#b=(a#a)+(b#b)
・実数cに対し、c#1=1#c=c-1
・任意の実数tに対しd'>dのときd'#t>d#t、任意の実数uに対しe'>eのときu#e'>u#e
このとき、1でない正の実数xに対し、x#xの値はただ一通りに定まるか。
0577132人目の素数さん垢版2018/01/08(月) 07:00:07.79ID:xaiDjCmX
>>566
最初の項が2なら最初の項を除いた物に対応させる。
最初の項が2より大きいなら最初の項が1小さい物に対応させる。
0578還暦FX垢版2018/01/08(月) 08:58:01.27ID:c93nVah2
>573  ヒントのつもりでしょうが、かなり遠い。
>577  もっと詳しく具体的に言ってもらわないと・・・
0579132人目の素数さん垢版2018/01/08(月) 09:00:13.33ID:xaiDjCmX
a#b
a#a=(a#a)+(a#a)。
a#a=0。
a#b=(a#a)+(b#b)=0。
0580132人目の素数さん垢版2018/01/08(月) 10:56:16.59ID:AWKuaxkV
<<575
(-2.16)(1.1)(3.21)

(3x-26)^2 +(3y-31)^2 = 1313,
なってますが
16.21.3,-2,1これらの数字をどう活用したのかわかりません
なにかの公式にあてはめたのですか?
0581132人目の素数さん垢版2018/01/08(月) 12:23:27.30ID:QCjUXnP+
>>580
二次関数のグラフってのは
y=ax^2+bx+c…@
の形のものでいい?
これを指定しないと頭の柔らかいひとは>>569>>574のような別解を思い付いたりする
@なら@でいいけどこれに点の座標を「あてはめ」れば方程式が3つできる
それらを連立させて解きなさい、というのが>>562のひとの説明
0582132人目の素数さん垢版2018/01/08(月) 12:33:34.06ID:AWKuaxkV
>>554
理解できました!!ありがとうございましたm(__)m
0583132人目の素数さん垢版2018/01/08(月) 12:45:52.09ID:AWKuaxkV
二次関数のグラフが次の3つの(x.y)座標点をとおるときの二次関数は次のうちどれか
(-2.16)(1.1)(3.21) y=ax^2+bx+c
16=4a-2b+C
1=a+b+C
21=9a+3b+C
を解いてy=ax^2+bx+cに代入すれば終わりということですね!
0584132人目の素数さん垢版2018/01/08(月) 14:50:17.06ID:D9asrVCn
多変数関数の微分
(x,y)=(0,0)で、0/0になる分数関数で、C^∞級関数になる例って何かありますか。

例えば
f(x,y):=x^3*y/(x^2+y^2) (x,y)≠(0,0)
f(0,0):=0
この関数fは(0,0)でC^1級であるが、C^2級ではない。
0585132人目の素数さん垢版2018/01/08(月) 15:36:59.89ID:spLj1lRb
計算機科学者と宇宙飛行士はどっちの方が頭が良いですか?
0586132人目の素数さん垢版2018/01/08(月) 16:05:57.95ID:AWKuaxkV
https://i.imgur.com/gmHbry4.png

傾きが-1/2の直線lが放物線y=1/2x^2と二点A.Bで交わっている。また、放物線上に点pがあるとする。点Aのx座標が-3であるとき
1)直線lの方程式をもとめよ
2)△AOBの面積を求めよ
どうときますか?教えて下さいお願い致します
0587132人目の素数さん垢版2018/01/08(月) 16:13:11.24ID:WA++1Lt3
縦ベクトル、横ベクトルなどといって区別するのは何か意味があるのでしょうか?
0588132人目の素数さん垢版2018/01/08(月) 16:53:55.12ID:ymFiPURZ
宇宙にはたてもよこもない。区別に意味はない
0591132人目の素数さん垢版2018/01/08(月) 16:58:13.41ID:SEA8GVJZ
>>586
直線の係数行列はヴァンデルモンド行列なので一意に係数が定まる。
面積はAとBのx座標間で幾つかの関数たちを積分してやればよい。
0593132人目の素数さん垢版2018/01/08(月) 18:06:20.15ID:AWKuaxkV
>>591
これって数1じゃないの?拾ってきた問だからわからんけど
積分…ならってないや
0596132人目の素数さん垢版2018/01/08(月) 19:05:27.66ID:VWO23o6Q
GL(2;C)の部分群 G = { {{Exp[I t],0},{0,Exp[I t a]}} | t in R, a 無理数}}

G_={{Exp[It},0},{0,Exp[I s]}}| t,sin R}

はGのクロージャであることをしめせ
0597132人目の素数さん垢版2018/01/08(月) 22:18:31.44ID:V+Ra263X
二項演算#は、任意の正の実数a,bに対し定義され、以下の性質を持つ。

・a#a=a
・a#(a+1)=a(a+1)
・a#x=f(x)とおくと、fはxについて連続
・a<bなら、a#a<a#b
・ab-(1/10^ab)<a#b<ab+(1/10^ab)がすべてのa≠bであるa,bに対し成り立つ

このとき、a#b=abであることを証明せよ。
0598132人目の素数さん垢版2018/01/08(月) 22:21:40.01ID:wOk7ob+W
>>595
>方程式のグラフ
0599132人目の素数さん垢版2018/01/08(月) 22:22:24.01ID:wOk7ob+W
>>597
>・a#a=a
a#b=ab?
0600132人目の素数さん垢版2018/01/08(月) 23:04:39.20ID:6yE9/RTC
>>590

辺BAおよび辺CDを延長し、その交点をEとする。
傳CE ∽ 僊DE は直角2等辺三角形

◇ABCD = 傳CE - 僊DE
=(BC^2 - AD^2)/2
={(AC^2 -AB^2)-(BD^2 -AB^2)}/2
=(AC^2 - BD^2)/2
= 68
0601132人目の素数さん垢版2018/01/09(火) 03:05:32.33ID:aCFpul36
566>>
整数nを、k個の1以上の整数で表す方法をA(n,k)と表すと、(n個のものを一直線に並べ、
n-1カ所の境目からk-1箇所を選んで仕切りをいれるアナロジーから)
A(n,k)=C[n-1,k-1] であることが判る。
「2以上の整数nを2以上の整数の和で表す方法 」をB(n)と表すこととすると、
(各和因子から1づつ減じた時のA(n,k)との関係から)
B(n)=A(n-1,1)+A(n-2,2)+A(n-3,3)+A(n-4,4)+...+A(n/2,n/2) または、A((n+1)/2,(n-1)/2)
(終項は、nの偶奇に依る)となることが判る。

B(2n+0)=C[2n-2,0]+C[2n-3,1]+C[2n-4,2]+...+C[n+0,n-2]+C[n-1,n-1]
B(2n+1)=C[2n-1,0]+C[2n-2,1]+C[2n-3,2]+...+C[n+1,n-2]+C[n,n-1]
B(2n+2)=C[2n-0,0]+C[2n-1,1]+C[2n-2,2]+...+C[n+2,n-2]+C[n+1,n-1]+C[n,n]
B(2n+3)=C[2n+1,0]+C[2n-0,1]+C[2n-1,2]+...+C[n+3,n-2]+C[n+2,n-1]+C[n+1,n]

C[n,k]=C[n-1,k-1]+C[n-1,k]やC[2n-1,0]=C[2n-0,0],C[n-1,n-1]=C[n,n]、あるいは、C[2n+1,0]=C[n,n]の関係から、
B(2n+2)=B(2n+1)+B(2n+0)
B(2n+3)=B(2n+2)+B(2n+1) が得られる。
0603132人目の素数さん垢版2018/01/09(火) 04:59:44.07ID:6f/jWShU
適当な整数を当て嵌めるだけでは解決せず、まるで方針が見えません。方針を教えてください。

p,qを互いに素な自然数とするとき、
abs{(p/q)-√5}<(1/q^4)
を満たすp,qを一組求めよ。
0605132人目の素数さん垢版2018/01/09(火) 07:24:31.00ID:6f/jWShU
nを自然数とする。
ただし以下の2題に直接の関係があるとは限らない。

(1)極限 lim[n→∞] ln(n^n)/ln(n!) を求めよ。
(2)次の極限は収束するか。
lim[n→∞] n^n/n!
0606132人目の素数さん垢版2018/01/09(火) 07:37:11.63ID:6f/jWShU
f(x)=1/x、g(x)はxの整式とする。
さらにh(x)を、
h(x)=e^(g(x))・f(x) (x≠0)
h(0)=lim[x→0] e^(g(x))・f(x) (x=0)
と定義する。なお、
lim[x→+0] h(x) = lim[x→-0] h(x)
が成り立つ場合のみh(0)を定義することに注意せよ。

h(x)がすべての実数に対し定義され、かつ、常に有限の実数値をとるようなg(x)の例を一組与えよ。
また、その例が確かに条件を満たしていることを説明せよ。
0607132人目の素数さん垢版2018/01/09(火) 09:18:36.39ID:aoqRTJub
2つの集合A={ x|x≧3}、B={x|2x-1≧a}についてA⊃Bが成り立つ
とき、 aの値の範囲は

どうとくのですか?
0608132人目の素数さん垢版2018/01/09(火) 11:32:15.34ID:aoqRTJub
放物線C:y=x∧2+6x+5kはx軸と2点A,Bで交わる。ただし、kは自然数とする。

線分ABの長さは、AB=□である。
x=−1,−5
点A,Bの座標は(-5.-1),(-1,0←x座標はわかるんだけどなんでyがこの数値なの?上に代入してもならんです
0609132人目の素数さん垢版2018/01/09(火) 12:51:09.97ID:dgOIUcU6
>>605

(1)
ln(n!)= Σ[k=2,n]ln(k)≒ ∫[3/2,n+1/2]log(x)dx =[ x・ln(x)-x ](3/2→n+1/2)
=(n+1/2)ln(n+1/2)-n +0.3918
≒(n+1/2)ln(n)-n +0.8918
1に収束する。

(2)
n!≒C n^(n+1/2)/(e^n)
収束しない。

>>607

 2x-1≧a ⇒ x≧3 ⇔ 2x-1≧5
∴ a≧5

>>608

ミスプリントです。(-5,0)(-1,0)
0610132人目の素数さん垢版2018/01/09(火) 13:47:29.85ID:aoqRTJub
>>609
2)は

y=-2(x+1)(x+5)
その頂点は-1と-5の真ん中のx=-3となあるのですが真ん中なぜ-3なのでふか?
0611132人目の素数さん垢版2018/01/09(火) 13:47:51.89ID:aoqRTJub
>>609みすでしたかありがとうございました
0612132人目の素数さん垢版2018/01/09(火) 14:28:38.25ID:aoqRTJub
放物線C:y=x∧2+6x+5kはx軸と2点A,Bで交わる。ただし、kは自然数とする。

(3)点A,BおよびC(0,-10)を通る放物線の頂点をPとすると、Pの
座標は(-□,□)である。

(4)点Pを通りX軸に平行な直線と放物線Cの交点D,Eとすると、線分DEの長さは、DE=□√□である。
こたえ
3)(-3,8)
4)4√3でいいですか?ミスプリみたいなので怪しいです
0613132人目の素数さん垢版2018/01/09(火) 14:29:13.55ID:aoqRTJub
>>610
すみませんわかりました
0614132人目の素数さん垢版2018/01/09(火) 15:55:10.87ID:k41au2Xd
ABを直径とする半円のなかにある円があり、円弧ABとTで接し、直径ABとUで接したます
角BTUが45度になることはどう示せばよいですうか。
0615132人目の素数さん垢版2018/01/09(火) 16:14:58.36ID:WWAbK6mK
>>614
半円を完全な円にする その中心を O とする
小円の中心を Q とする
TU と円 O の交点で T ではないほうを V とする
△QTU ∽ △OTV が容易にわかる
よって QU ‖ OV で, ∠BUQ = ∠BOV = 90° がわかる
0616132人目の素数さん垢版2018/01/09(火) 16:19:06.47ID:aoqRTJub
<<0607

2x-1≧aを解くと
x≧(a+1)/2までわかったのですが
A⊃Bが成り立つ とき3≦(a+1)/2

なんで≦向きなんですか?
0617132人目の素数さん垢版2018/01/09(火) 18:44:38.68ID:oqIt5eQp
GL(2;C)の部分群 G = { {{Exp[I t],0},{0,Exp[I t a]}} | t in R, a 無理数}}

G_={{Exp[It},0},{0,Exp[I s]}}| t,s in R}

はGのクロージャであることをしめせ
0618132人目の素数さん垢版2018/01/09(火) 23:05:52.35ID:6f/jWShU
以下の問題が分かりません。2^k>1+2+…+2^(k-1)を使うのだろうということは分かりますが、2個の余分な玉の処理の仕方が分かりません。

n個の箱H1,H2,...,Hnが左からこの順に並んでいる。
この箱に、n+2個の玉を投げ入れる。球がどの箱に入るかは同様に確からしいとする。

球が入っている箱の中で、最も右側にあるものをHkとする。このとき、Fiを

・Hiの中にm個の球が入っているとき、Fi=2^m
・Hiの中に球が入っていないとき、Fi=0
・Hkの中にm個の球が入っているとき、Fk=-2^m

と定める。
各Fiの和について
Σ[i=1,n] Fi >0 
が成り立つ確率を求めよ。
0619614垢版2018/01/10(水) 00:21:05.04ID:iwwPN5Uq
>>615 さま。ありがとうございます。
当方頭悪いみたいで

>△QTU ∽ △OTV が容易にわかる

が容易に分かりません(´;ω;`)
宜しくお願いします
0621132人目の素数さん垢版2018/01/10(水) 01:11:21.39ID:iwwPN5Uq
そうか!二等辺三角形だった!
わかっちゃいました。ありがおとうごじました。
0624132人目の素数さん垢版2018/01/10(水) 08:09:24.78ID:XJpuOhA2
2x-1≧aを解くと
x≧(a+1)/2までわかったのですが
A⊃Bが成り立つ とき3≦(a+1)/2

なんで≦向きなんですか?
0626132人目の素数さん垢版2018/01/10(水) 09:56:34.28ID:dSClGAkM
>>624
Aに含まれるxは3≦xを満たし、
Bに含まれるxは(a+1)/2≦xを満たす
A⊃Bというのは、Bの範囲がAの範囲をはみ出さないことを表す
さて、3と(a+1)/2のどちらが大きければ良いでしょう?というのが問題
0628132人目の素数さん垢版2018/01/10(水) 11:39:48.47ID:9NYY/5Sm
>>627
x軸に3と(a+1)/2を記入
0629132人目の素数さん垢版2018/01/10(水) 11:41:04.13ID:FVLABp+/
宇宙飛行士と飽和潜水士はどっちの方が頭が良いですか?
0632132人目の素数さん垢版2018/01/10(水) 14:04:51.58ID:yyayDtEj
>> ID:bRNIXSXz
566の問題は、和因子が2以上に限っているため、分割数とは異なるのは明白だが、
その限定を無くしても、

>>566
>>5=2+3=3+2=5で3通り
>>6=2+2+2=2+4=4+2=3+3=6で5通り
という例からも判るように、566の問題と、分割数の問題とは異なる。
分割数では、例えば6は、
1+1+1+1+1+1=2+1+1+1+1=2+2+1+1=2+2+2=3+1+1+1=3+2+1=3+3=4+1+1=4+2=5+1=6
と考え、11通りの表現方法があるため、p(6)=11となる。
566の例のように、2+3と3+2、あるいは、2+4と4+2を別物とは考えない。
0633132人目の素数さん垢版2018/01/10(水) 14:33:28.83ID:dSClGAkM
>>632
分割数と異なるのはわかったとして、元の問題が「2以上の整数からなり、かつ総和がnの有限数列」の個数を求める問題と読み替えれば良いだけと違う?
0634132人目の素数さん垢版2018/01/10(水) 15:08:04.29ID:sU8Z6M9O
中学数学やりなおしてる。
底面が8センチ縦が6センチの円柱の体積はいくつか
って問題で答が96π立方センチとあったんだけど 301.44立方センチじゃ不正解なの?
0635132人目の素数さん垢版2018/01/10(水) 15:58:59.29ID:Pyy2bIO0
>>634
円周率をどのように扱うか、そのときの学習段階でのお約束による
受験問題ならどのように扱うのか明記されているはずなので心配無用というのもお約束

それ以前に
> 底面が8センチ縦が6センチの円柱
これではわけがわからん
0636132人目の素数さん垢版2018/01/10(水) 16:00:49.93ID:dSClGAkM
>>634
円周率をどうしなさいと書いてあったかが問題
πのままにしてよい、とあればそうすれば良いし
3.14としなさいとか、3にしなさいとか書いてあればそれに従えばよい
指定がなければ教科書としては不備

>底面が8センチ
底面の円の直径のことでよい?
0637132人目の素数さん垢版2018/01/10(水) 16:12:23.88ID:XJpuOhA2
>>628
意味わかりませんすみません
0641132人目の素数さん垢版2018/01/10(水) 16:54:00.01ID:oaWP3F32
>>637
阿保?
0644132人目の素数さん垢版2018/01/10(水) 19:03:11.98ID:MMGnlsA5
杉原厚吉著『トポロジー』を読んでいます。

「皆さんの中には、ロープに関する私たちの常識はかなり確固としたものであって、
トポロジーなどという学問で補強されなくても大丈夫だと思っている人がいるかも
しれない。しかし常識というのはあいまいなものである。常識が必ずしも通用しない
ことを実感できる一つの例が、図1.2に示すパズルである。」

などといって、子供だましなパズルを登場させています。

パッと見、普通のロープに関する常識で不可能なパズルか可能なパズルかを
瞬間的に判断できます。

恥ずかしい人です。


(a) を (b) にできないのは明らかです。二つの玉が口を挟んで反対側にあるからです。

(a) を (c) にできるのも明らかです。ひもを口から外して、玉の位置を調整して、
ひもを口に入れればいいからです。
0646132人目の素数さん垢版2018/01/10(水) 19:04:14.48ID:MMGnlsA5
https://imgur.com/LUUhV9C.jpg

杉原厚吉著『トポロジー』を読んでいます。

「皆さんの中には、ロープに関する私たちの常識はかなり確固としたものであって、
トポロジーなどという学問で補強されなくても大丈夫だと思っている人がいるかも
しれない。しかし常識というのはあいまいなものである。常識が必ずしも通用しない
ことを実感できる一つの例が、図1.2に示すパズルである。」

などといって、子供だましなパズルを登場させています。

パッと見、普通のロープに関する常識で不可能なパズルか可能なパズルかを
瞬間的に判断できます。

恥ずかしい人です。


(a) を (b) にできないのは明らかです。二つの玉が口を挟んで反対側にあるからです。

(a) を (c) にできるのも明らかです。ひもを口から外して、玉の位置を調整して、
ひもを口に入れればいいからです。
0647132人目の素数さん垢版2018/01/10(水) 19:24:29.65ID:oaWP3F32
>>646
どう外すの?
0648132人目の素数さん垢版2018/01/10(水) 20:10:00.53ID:MMGnlsA5
>>646

あ、これって素直に口にひもを入れていないんですね。
絵が下手なので気づきませんでした。

でも、口から出ているひもを伸ばして、顔の下から顔の裏へ持って行って
顔の上から前へ持ってくればいいですよね。

子供だましですね。
0649132人目の素数さん垢版2018/01/10(水) 20:10:57.00ID:MMGnlsA5
>>646

あ、これって素直に口にひもを入れていないんですね。
絵が下手なので気づきませんでした。

でも、口に入っているひもを伸ばして、顔の下から顔の裏へ持って行って
顔の上から前へ持ってくればいいですよね。

子供だましですね。
0651132人目の素数さん垢版2018/01/10(水) 20:17:37.65ID:MMGnlsA5
>>649

あ、なんかおかしいですね。
0652132人目の素数さん垢版2018/01/10(水) 20:21:10.44ID:dSClGAkM
頭の悪い人は、頭のいい人が理解できることを理解できない
ちょっと頭のいい人は、頭の悪い人がなぜ理解できないか理解できない
もっと頭のいい人は、頭の悪い人がなせ理解できないか理解できる
そういうことですよ
0653132人目の素数さん垢版2018/01/10(水) 20:41:15.42ID:GzoDTMM5
GL(2;C)の部分群 G = { {{Exp[I t],0},{0,Exp[I t a]}} | t in R, a 無理数}}

G_={{Exp[It},0},{0,Exp[I s]}}| t,s in R}

はGのクロージャであることをしめせ
0654132人目の素数さん垢版2018/01/10(水) 21:02:40.18ID:9NYY/5Sm
>>651
>ID:MMGnlsA5
バカだということが分かって何より
0655132人目の素数さん垢版2018/01/10(水) 21:23:25.02ID:/abNLWqR
直径1の円に内接するn角形のうち、その周長が最もπに近くなるのは正n角形ですか?
0656還暦FX垢版2018/01/10(水) 21:30:58.07ID:q2GGfkXh
>566 の問題について、皆さんいろいろ考えてもらって恐縮です。
解答は >601 でいいのだろうと思います。ただし
(各和因子から1づつ減じた時のA(n,k)との関係から)
B(n)=A(n-1,1)+A(n-2,2)+A(n-3,3)+A(n-4,4)+...+A(n/2,n/2) または、A((n+1)/2,(n-1)/2)
(終項は、nの偶奇に依る)となることが判る。

これを何により説明したかが気になります。
確かに、いくつかの例を調べればこうなっているのは正しいのですが、
いくつかの例からこの関係があるというのなら、初めから、フィボナッチの関係
がみられるから、フィボナッチ数列だというのと同じだと思います。そこがどうも・・・。
0658132人目の素数さん垢版2018/01/10(水) 21:59:33.65ID:FTjEJJ/x
頭が良くなりたいのに全然良くならないのですが、自殺するしかないですか?
0659132人目の素数さん垢版2018/01/10(水) 22:16:49.38ID:yyayDtEj
>>656 例えば、B(10)を考えてみましょう。
1項から成るもの:10
2項から成るもの:2+8,3+7,4+6,5+5,6+4,7+3,8+2
3項から成るもの:2+2+6,2+3+5,2+4+4,2+5+3,2+6+2,3+2+5,3+3+4,3+4+3,3+5+2,4+2+4,4+3+3,4+4+2,5+2+3,5+3+2,6+2+2
4項から成るもの:2+2+2+4,2+2+3+3,2+2+4+2,2+3+2+3,2+3+3+2,2+4+2+2,3+2+2+3,3+2+3+2,3+3+2+2,4+2+2+2
5項から成るもの:2+2+2+2+2
つまり、1+7+15+10+1=34となります。
ここに現れる数字は、当然ながら全て2以上です。1を引いても、なお、1以上が残ります。では、全ての数字から1を引いてみましょう。
1項から成るもの:9
2項から成るもの:1+7,2+6,3+5,4+4,5+3,6+2,7+1
3項から成るもの:1+1+5,1+2+4,1+3+3,1+4+2,1+5+1,2+1+4,2+2+3,2+3+2,2+4+1,3+1+3,3+2+2,3+3+1,4+1+2,4+2+1,5+1+1
4項から成るもの:1+1+1+3,1+1+2+2,1+1+3+1,1+2+1+2,1+2+2+1,1+3+1+1,2+1+1+2,2+1+2+1,2+2+1+1,3+1+1+1
5項から成るもの:1+1+1+1+1
これらは、将に、9を1分割したもの、8を2分割したもの、7を三分割したもの、6を四分割したもの、5を五分割したものです。
だから、それぞれ、C[8,0]、C[7,1]、C[6,2]、C[5,3]、C[4,4]に対応できるのです。
最初の、C[8,0]だけは、他の内容から、こじつけた感じはありますが、これらについては、>>601で、
「(各和因子から1づつ減じた時のA(n,k)との関係から) 」と書いた通りで、結果からこじつけたものではありません。
たまたま、各項がコンビネーションで書け、さらに、偶数と奇数で、微妙な差があったこと、さらに、
コンビネーションの加法定理が、フィボナッチ数列の漸化式に対応できたため、完全に一致しただけです。回り道をして、結果的に偶然にも一致したのです。
一致しているという事実から、おそらく対応関係を使って直接的な説明もあるのだと思いますが、今のところ思い浮かびません。
0661還暦FX垢版2018/01/10(水) 22:35:16.67ID:q2GGfkXh
例なら示せますが
B(n)を考えよう
k項からなるもの ・・・・
と示さなければ証明にならないのでは? という意味です。
0663132人目の素数さん垢版2018/01/10(水) 22:50:55.25ID:/abNLWqR
初等関数の和差積商で表される関数fについて
その不定積分Fが初等関数の和差積商で表されるかどうかを判定する方法ってありますか?
0664132人目の素数さん垢版2018/01/10(水) 22:53:50.03ID:yyayDtEj
>>660
なるほど、そう考えれば、直接的ですね。この説明で十分。秀逸ですね。

>>661
601では、直接B(n)を求めて、表式を示しています。
そしてそのB(n)が前二項の和でもあるということを、計算で示しているのです。
初期値などについては省略していますが、証明の形式として問題ないと思いますが。
0665132人目の素数さん垢版2018/01/10(水) 23:04:15.43ID:XJpuOhA2
>>640すみませんそれがなんで
なんで≦向きなんですか?
0666132人目の素数さん垢版2018/01/10(水) 23:07:50.24ID:MMGnlsA5
杉原厚吉著『トポロジー』を読んでいます。

------------------------------------------------------------------
R^n の任意の部分集合 X と、 X の中の任意の2点間に定義されている
ユークリッド距離 D との対 (X, D) を、距離空間(metric space)という。

X を図形とし、 U ⊂ X とする。任意の点 P ∈ U に対して、 N(P, ε) ⊂ U
を満たす ε が存在するとき、 U を開集合(open set)という。

性質1.2 (X, D) が距離空間で、 ε が任意の正の定数のとき、任意の P ∈ X
の ε 近傍 N(P, ε) は開集合である。
------------------------------------------------------------------

などと書いてありますが、性質1.2の N(P, ε) は X に含まれるとは限りませんよね。
おかしいですね。
0667132人目の素数さん垢版2018/01/10(水) 23:09:05.70ID:MMGnlsA5
大体、工学系の人が専門でもないトポロジーの本を書くというのが図々しいですよね。
0669132人目の素数さん垢版2018/01/10(水) 23:16:12.62ID:iygiki0Q
>>665
 640に書いた (1+x)/2 は (1+a)/2 の誤記ということはわかっているものとして

A⊃B の意味は解っているの?

で、解っているものとして

(1+a)/2 ∈B ということは分る?
A⊃B だから (1+a)/2∈A となることは分る?
(1+a)/2∈A なら A の定義から 3≦(1+a)/2 となることは分る?
0670132人目の素数さん垢版2018/01/10(水) 23:48:36.04ID:FOVMfCwz
>>566
F(k)を、k≧2のとき、各項が2以上の整数でかつ和がkとなる項数1以上の多項式とする。
多項式は、項数と同じ順番の項が等しいときのみ等しいとする。すなわち、2+3と3+2は異なる多項式である。
B(k)を、F(k)の要素数とする。(k≧2)
これから、B(k+2)=B(k+1)+B(k)であることを示そう。

s(k)をkの後者とする。(s(k)はk+1と同じ値を持つが、紛らわしいので多項式の中ではこのように記載する)
集合F1(k)を{s(x1)+x2+...+xm|x1+x2+...+xm(m≧1)∈F(k)}と定義すると、F1(k)⊂F(k+1)である。
よってF1(k+1)⊂F(k+2)である。
集合F2(k)を{2+x1+x2+...+xm|x1+x2+...+xm(m≧1)∈F(k)}と定義すると、F2(k)⊂F(k+2)である。
F1(k+1)とF2(k)は、ともにF(k+2)の部分集合であるが、(つまりF1(k+1)∪F2(k)⊂F(k+2))
F1(k+1)の要素は、初項が3以上であるため、F2(k)と共通の要素はない。(つまりF1(k+1)∩F2(k)=φ)
F1(k),F2(k)の要素数はF(k)の要素数B(k)と等しいことから、B(k+2)≧B(k+1)+B(k)…@
次に、F(k)のうち、初項が2のものからなる集合をFA(k)、それ以外の要素からなる集合をFB(k)とする。
集合Fa(k)を{x1+x2+...+xm|2+x1+x2+...+xm(m≧1)∈FA(k+2)}と定義すると、Fa(k)⊂F(k)である。
集合Fb(k)を{x1+x2+...+xm|s(x1)+x2+...+xm(m≧1)∈FB(k+1)}と定義すると、Fb(k+1)⊂F(k+1)である。
Fa(k)の要素数とFA(k+2)の要素数は等しく、
Fb(k)の要素数とFB(k+1)の要素数は等しく、
かつ、FA(k+2)の要素数とFB(k+2)の要素数の和はB(k+2)に等しいことから、B(k+2)≦B(k+1)+B(k)…A
@とAより、B(k+2)=B(k+1)+B(k) □
0673132人目の素数さん垢版2018/01/11(木) 10:34:31.47ID:eU7svxbo
GL(2;C)の部分群 G = { {{Exp[I t],0},{0,Exp[I t a]}} | t in R, a 無理数}}

G_={{Exp[It},0},{0,Exp[I s]}}| t,s in R}

はGのクロージャであることをしめせ
0674132人目の素数さん垢版2018/01/11(木) 10:39:31.96ID:HhrLSwk0
一辺の長さが有理数である正n角形のうち、面積も有理数となるものをすべて求めよ。
無数に存在する場合は、どのようなnが条件を満たすかを述べよ。
解答にあたり、sin(π/k)が有理数となるのはk=1,2,6のみであることを用いて良い。

正方形だけだと思うのですが、ヒントの使い方がうまく行きません。
s=1/2bcsinθを使うのだと思うのですが、周長が有理数という条件をどう結びつけるか分かりません。
よろしくお願いします。
0675132人目の素数さん垢版2018/01/11(木) 12:26:04.03ID:k88se1oG
>>672
≧じゃないのかとか根拠しれないと納得できないてす
0676132人目の素数さん垢版2018/01/11(木) 12:55:41.04ID:d+fpg1rB
>>674
三角形の面積は、(1/2)×底辺×高さで考えればよい。
1辺をaとすると正n角形の面積はna^2/(4tan(π/n))となるので、
これが有理数となる条件はtan(π/n)が有理数となること。
あとは、sin2θ=2tanθ/(1+(tanθ)^2)より、
tanθが有理数⇒sin2θが有理数
を使えばいい。
0677132人目の素数さん垢版2018/01/11(木) 13:04:46.03ID:gFnXPNe4
ムハンマドとエウクレイデスはどっちの方が凄いですか?
0679132人目の素数さん垢版2018/01/11(木) 13:08:57.75ID:HhrLSwk0
>>676
sinとtanが分数式の形でつながるので、tanが有理数→sinが有理数なんですね
ここに気が付きませんでした、なるほどと思いました
ありがとうございました
0680132人目の素数さん垢版2018/01/11(木) 13:56:17.79ID:d+fpg1rB
>>679
と思ったが、これだけだとnが奇数のときtan(π/n)が無理数になることが
まだ言えないことに気づいた。

tanθが有理数⇒sin2θもcos2θも有理数なので、kが奇数のとき
sin(2π/k)とcos(2π/k)が有理数 ⇒ sin(π/k)が有理数
を言う必要がある。
このためには、
cos(nθ)=f(cosθ)
sin(nθ)=sinθ*g(cosθ) (f(x),g(x)はxの整数係数の多項式)
と表されることを数学的帰納法で示し、
sin(π/(2n-1)) = -sin(π+π/(2n-1)) = -sin(n*2π/(2n-1))
を使えばよい。
0681132人目の素数さん垢版2018/01/11(木) 15:12:49.86ID:DF+KMzcz
>>675
問題が
「2つの集合A={ x|x≦3}、B={x|2x-1≦a}についてA⊃Bが成り立つ
とき、 aの値の範囲は」

なら 3≧(1+a)/2 となるね。

元の問題では
⊃ と ≧ が同じ向きにならないのが気持ち悪いんだろ   
0683132人目の素数さん垢版2018/01/11(木) 16:08:49.07ID:JMEE9epQ
https://i.imgur.com/0i2QYOn.jpg
画質悪くてごめんなさい。

明日テストなので、教えて下さい。
0685132人目の素数さん垢版2018/01/11(木) 17:23:04.26ID:JMEE9epQ
>>684
分母分子をn^2乗で割るのですね。
0686132人目の素数さん垢版2018/01/11(木) 19:45:39.37ID:ROuvx2W4
Aのツボは99個の青い球と1個の赤い球が詰まっている

Bのツボは99個の赤い球と1個の青い球が詰まっている

このとき、自分の目の前のツボから1個球を
取り出してみたら赤い球であった

目の前のツボはAのツボだろうか、Bのツボだろうか
0689132人目の素数さん垢版2018/01/11(木) 21:36:14.15ID:XVqDnWxl
>>681
気持ち悪いってのが気持ち悪い
0690132人目の素数さん垢版2018/01/11(木) 23:20:11.92ID:d+fpg1rB
>>688
f(x)の原始関数の1つをg(x)とおく。すなわち、g'(x) = f(x)
F(x) = ∫[-sinx,sinx]f(t)dt = g(sinx)-g(-sinx)
F'(x) = g'(sinx)*(sinx)' - g'(-sinx)*(-sinx)'
= (f(sinx)+f(-sinx))cosx
0691132人目の素数さん垢版2018/01/12(金) 01:17:07.66ID:9YqB0pR4
>>686
Aは青っぽいツボ
Bは赤っぽいツボと思えば、
赤球が出てきたツボはBっぽい気がする。
それを正当化する議論は
ベイズ流で得られる。

ツボがAであるという条件下に
赤球を取り出す確率は P(赤|A)=1/100、
青球を取り出す確率は P(青|A)=99/100。
ツボがBであるという条件下に
赤球を取り出す確率は P(赤|B)=99/100、
青球を取り出す確率は P(青|B)=1/100。
ベイズの定理を使うと、
赤球を取り出したという条件下に
ツボがAである確率 P(A|赤) は、
赤球を取り出す以前に
ツボがAであると思われる確率P(A)を使って
P(赤|A)p(A)=P(A|赤)P(赤),
P(赤)=P(赤|A)P(A)+P(赤|B)P(B),
P(A)+P(B)=1 より、
P(A|赤)=P(赤|A)p(A)/{P(赤|A)P(A)+P(赤|B)P(B)}
=(1/100)P(A)/{(1/100)P(A)+(99/100)(1-P(A))}
=1/{99/P(A)-1}。

0≦P(A)≦1 より P(A|赤)≦1/98,
P(B|赤)≧97/98 となるから、
ツボはBっぽいと考えるのが妥当だろう。
0693132人目の素数さん垢版2018/01/12(金) 02:21:30.93ID:j6rdYvqK
a,p,qは実数とする。数列a(n)は
a(1)=a
a(n+1)=pa(n)+q
を満たす。
また、数列a(n)のある項a(k)で
a(k-1)>a(k)<a(k+1)またはa(k-1)<a(k)>a(k+1)
となるものがm個あるとき(ただしm≧1,k≧2)、数列a(n)はm個の極値をもつという。またこのときのkを特異値という。
以下の問に答えよ。

(1)a(n)が極値をもつとき、a,p,qが満たすべき条件を求めよ。
(2)(1)において、a(n)は何個の極値を持つか。
(3)Nが特異値であるとき、a,p,qが満たすべき条件を求めよ。
0696132人目の素数さん垢版2018/01/12(金) 06:04:34.45ID:j6rdYvqK
>>694
工学系の大学院入試ですか?
解析学の教科書に書いてあることも多いので、それを参考にしてください
大学の図書館探せば容易に解決します
0700132人目の素数さん垢版2018/01/12(金) 10:04:39.63ID:sbnqsFKD
x ∈ R^n

とする。

x に一番近い集合 [a_1, b_1] × … × [a_n, b_n] 上の点を求めよ。
0701132人目の素数さん垢版2018/01/12(金) 10:47:39.99ID:iCAhw7uf
中学校の数学から始めるための、良い参考書を教えて下さい。
0703132人目の素数さん垢版2018/01/12(金) 11:04:52.63ID:8cm1ve4d
ユークリッドの原論
0704132人目の素数さん垢版2018/01/12(金) 12:29:01.76ID:wSNmfFyf
>>693
p≠1 のとき、b(n)=a(n)+q/(p-1) とすると、
b(n+1)=a(n+1)+q/(p-1)=pa(n)+q+q/(p-1)=pa(n)+(q(p-1)+q)/(p-1)=pa(n)+pq/(p-1)=p(a(n)+q/(p-1))=pb(n)なので
b(n)は初項a+q/(p-1)、公比pの等比数列となる。
公比p≧0または初項a+q/(p-1)=0のときb(k)は単調増加、単調減少、k≧2において不変のいずれかであり、特異値は存在しない。
公比p<0かつ初項a+q/(p-1)≠0のときk≧2においてb(k-1)<0<b(k)>0>b(k+1)またはb(k-1)>0>b(k)<0<b(k+1)となるので、
全てのk≧2が特異値となる。
q/(p-1)が定数なのでa(k)の特異値はb(k)と一致する。
p=1のとき、a(n+1)=a(n)+qより、a(n)は初項a公差qの等差数列となる。
この場合もa(n)は単調増加、単調減少、不変のいずれかであり、特異値は存在しない。
以上から、a(k)の極値は、p<0かつa+q/(p-1)≠0の条件で存在し、k≧2となる全てのkが特異値となる。
0705132人目の素数さん垢版2018/01/12(金) 13:57:41.04ID:BrURCJm0
古代エジプトの数学者と現代イギリスの数学者はどっちの方が頭が良いですか?
0706132人目の素数さん垢版2018/01/12(金) 15:40:51.61ID:jxiFg+zg
>>703
ヒルベルトにより厳密性に欠けるとされている
0709132人目の素数さん垢版2018/01/12(金) 19:00:55.11ID:sbnqsFKD
R^n の開集合系 O = O(R^n) の部分集合 B が O の基底であるためには、
R^n の任意の開集合 O1(≠ 空集合)と O1 の任意の点 a に対し、

a ∈ U, U ⊂ O1

となる B の元 U が存在することが必要十分であることを示せ。

必要性:

O1 は B の元の和集合で表せる。

任意の a ∈ O1 はその和集合の元だから、

a ∈ B1 となるようなその和集合を構成している B1 ∈ B が存在する。

B1 ⊂ その和集合 = O1

十分性:

O1 を R^n の任意の開集合(≠ 空集合)とする。
O1 に含まれるような B の元全体の集合を C とする。

a を O1 の任意の元とする。仮定により、
a ∈ U, U ⊂ O1 となるような B の元 U が存在する。

この U は C に含まれている。

よって、 C の元の和集合 = O1 である。
0710132人目の素数さん垢版2018/01/12(金) 19:01:41.59ID:sbnqsFKD
>>709

この解答はあっていますか?

記号的にすっきりとした解答を書いてください↓
0711132人目の素数さん垢版2018/01/12(金) 19:02:39.80ID:sbnqsFKD
>>700

これは簡単でしたね。
0712132人目の素数さん垢版2018/01/12(金) 19:20:59.42ID:sbnqsFKD
(1)
R^n の開集合系 O = O(R^n) の部分集合 B が O の基底であるためには、
R^n の任意の開集合 O1(≠ 空集合)と O1 の任意の点 a に対し、

a ∈ U, U ⊂ O1

となる B の元 U が存在することが必要十分であることを示せ。

(2)
またこの条件は、 R^n の任意の点 a と任意の正数 ε とに対し、

a ∈ U, U ⊂ B(a ; ε)

となる B の元 U が存在することとも同等であることを示せ。
0713132人目の素数さん垢版2018/01/12(金) 19:29:13.28ID:sbnqsFKD
(2)ですが、以下の解答であっていますか?

(1) ⇒ (2):

O1 := B(a ; ε) とする。
a ∈B(a ; ε) だから、

a ∈ U, U ⊂ B(a ; ε)

となる B の元 U が存在する。


(2) ⇒ (1):

O1 を R^n の任意の開集合、 a を O1 の任意の点とする。
B(a ; ε) ⊂ O1 となるような正の実数 ε が存在する。

仮定により、 a ∈ U, U ⊂ B(a ; ε) となるような B の元 U が存在する。

U ⊂ B(a ; ε) ⊂ O1

だから、

U ⊂ O1

である。
0714132人目の素数さん垢版2018/01/12(金) 19:57:38.83ID:Up95xFG/
すみません、自殺したいという衝動を抑えることができません
人を殺したら抑えることができますか?
0715132人目の素数さん垢版2018/01/12(金) 20:15:57.55ID:yXSU2VlT
宝くじを買っていて、当たるかハズレるかの二分の一の筈なのに全く当たりません
何故ですか?
買っているのはロト6です
よろしくお願いします
0717132人目の素数さん垢版2018/01/13(土) 00:27:28.29ID:l9Qjr5Ul
>>694
こちらの問題ですが図書館で調べたところ(4)(5)以外は解決出来ましたので
どなたか(4)(5)の解き方を教えて頂けると助かります
0718132人目の素数さん垢版2018/01/13(土) 00:27:33.58ID:p9jSzu0M
なぜ?
数学が出来ない人間ほど
数学の間違いにキビシイのな
自分が間違いを指摘されまくっているからかな
それとも
正しく試行錯誤をした経験が少なくて
試行錯誤というものの価値を知らないのかな
0721132人目の素数さん垢版2018/01/13(土) 00:58:42.65ID:PkZjn6P7
>>720
どういうことですか?
0723132人目の素数さん垢版2018/01/13(土) 06:36:51.60ID:D8QyO0Jv
>>719
いくらなんでも基本的すぎる
ここで聞く前に図書館か、自分で持ってる教科書か、講義ノート見れば
それで十分
0724132人目の素数さん垢版2018/01/13(土) 09:38:10.24ID:PkZjn6P7
>>723
それが、手元に今資料とか教科書とか何一つ持ってないんです
どうかお願いします
0725132人目の素数さん垢版2018/01/13(土) 09:45:21.95ID:D8QyO0Jv
>>724
図書館行け
こんな基礎の基礎も出来ないなんてどこの低学歴だか知らんが、どんな大学にも図書館はあるからな
0727132人目の素数さん垢版2018/01/13(土) 10:51:23.61ID:safIkP1j
初っ端からいきなりまず使ってる教科書をエスパーしろとはとんだ難問だぜHAHAHA
0729132人目の素数さん垢版2018/01/13(土) 20:11:45.44ID:cJ/rLr4Z
The number of topologies on the set {1, 2, 3} is 29.

The all topologies on the set {1, 2, 3} are:
[[], [1, 2, 3]]
[[], [1], [1, 2, 3]]
[[], [2], [1, 2, 3]]
[[], [1, 2], [1, 2, 3]]
[[], [1], [1, 2], [1, 2, 3]]
[[], [2], [1, 2], [1, 2, 3]]
[[], [1], [2], [1, 2], [1, 2, 3]]
[[], [3], [1, 2, 3]]
[[], [1, 2], [3], [1, 2, 3]]
[[], [1, 3], [1, 2, 3]]
[[], [1], [1, 3], [1, 2, 3]]
[[], [2], [1, 3], [1, 2, 3]]
[[], [1], [1, 2], [1, 3], [1, 2, 3]]
[[], [1], [2], [1, 2], [1, 3], [1, 2, 3]]
[[], [3], [1, 3], [1, 2, 3]]
[[], [1], [3], [1, 3], [1, 2, 3]]
[[], [1], [1, 2], [3], [1, 3], [1, 2, 3]]
[[], [2, 3], [1, 2, 3]]
[[], [1], [2, 3], [1, 2, 3]]
[[], [2], [2, 3], [1, 2, 3]]
[[], [2], [1, 2], [2, 3], [1, 2, 3]]
[[], [1], [2], [1, 2], [2, 3], [1, 2, 3]]
[[], [3], [2, 3], [1, 2, 3]]
[[], [2], [3], [2, 3], [1, 2, 3]]
[[], [2], [1, 2], [3], [2, 3], [1, 2, 3]]
[[], [3], [1, 3], [2, 3], [1, 2, 3]]
[[], [1], [3], [1, 3], [2, 3], [1, 2, 3]]
[[], [2], [3], [1, 3], [2, 3], [1, 2, 3]]
[[], [1], [2], [1, 2], [3], [1, 3], [2, 3], [1, 2, 3]]
0730132人目の素数さん垢版2018/01/13(土) 20:15:10.78ID:cJ/rLr4Z
たとえば、

[[], [1], [3], [1, 3], [2, 3], [1, 2, 3]]

の元の任意個の和集合がまたこの集合に含まれることはどうやって証明するのでしょうか?

[[], [1], [3], [1, 3], [2, 3], [1, 2, 3]]

の任意の2つの元の和集合はまたこの集合に含まれるため、

[[], [1], [3], [1, 3], [2, 3], [1, 2, 3]]

の元の可算個の和集合がまたこの集合に含まれることは容易に証明できます。

[[], [1], [3], [1, 3], [2, 3], [1, 2, 3]]

の元の非可算個の和集合がまたこの集合に含まれることはどうやって示すのでしょうか?
0731132人目の素数さん垢版2018/01/13(土) 21:17:56.49ID:p9jSzu0M
>>730
元をチェックするだけよ
0732132人目の素数さん垢版2018/01/13(土) 21:53:53.20ID:cJ/rLr4Z
>>731

証明を書いてください。
0734132人目の素数さん垢版2018/01/13(土) 22:27:46.97ID:juTmgpJ0
殺す殺す殺す殺す殺す
0736132人目の素数さん垢版2018/01/14(日) 00:11:22.15ID:yNhhPVL1
良スレおつ
0737132人目の素数さん垢版2018/01/14(日) 00:39:48.11ID:7iI/H+u5
>>732
非可算個でも何でも有限種類しかないのでね
A_α(αで区別)は習合{A_α}にしたら有限個になっちゃうのよ
0739132人目の素数さん垢版2018/01/14(日) 13:20:00.93ID:pRmBgtM0
何で?
0740132人目の素数さん垢版2018/01/14(日) 16:11:20.41ID:/iIQ3OWQ
方程式ax^2-x-1=0の異なる2つの解α、βがともに-1と1の間にあるための条件を求めよ
解説お願いします
0741132人目の素数さん垢版2018/01/14(日) 16:16:36.79ID:pRmBgtM0
そんな難問、僕には溶けません。パス
0743132人目の素数さん垢版2018/01/14(日) 16:28:07.89ID:E6EeLnuk
>>740
数Vの範囲でいいなら、定数分離して増減調べるだけ
やり方はさっきの人が書いてくれた通り
0745132人目の素数さん垢版2018/01/14(日) 16:42:46.96ID:E6EeLnuk
>>740
しょうがないな数Tの解き方教えてやる
感謝しろよ

f(α)>0,f(β)>0を解いてaの不等式にしろ
次にf(α)<0,f(β)<0も同じように解け
この共通範囲が解だ
0747132人目の素数さん垢版2018/01/14(日) 17:06:39.11ID:E6EeLnuk
>>740
おまえオモチャにされてるぞ
いいのか?
今からでも遅くない、勢威ある書き込みをしろ
0748132人目の素数さん垢版2018/01/14(日) 17:26:30.07ID:09atsn3P
■ピタゴラスの定理の三辺の長さ

3 4 5

5 12 13

7 15 17

から何か規則性はありますか?
0749遊園地 ◆ExGQrDul2E 垢版2018/01/14(日) 17:28:59.68ID:vWf+Ng/c
組み合わせの問題
計算用数値(数字)(差分1ずつ増加)の減らし方

 例 1から7までの計算数字は  1,2、4で表せる

+1(合計1)
+2 -1(合計2)
+1(合計3)
+4 -2 -1(合計4)
+1(合計5)
+2 -1(合計6)
+1 (合計7)

この様に8の場合 や 9の場合 ……を求める場合の最小数字の組み合の求め方。

用語とか、有ったら教えて下さい。
0750132人目の素数さん垢版2018/01/14(日) 17:41:18.53ID:E6EeLnuk
>>749
すべての整数は4の倍数か、4で割ったあまりが1,2,3になる
3=1+2だから、すべての整数は4,1,2で表せる
必要な最小数も容易に求められる
0751遊園地 ◆ExGQrDul2E 垢版2018/01/14(日) 17:48:05.46ID:vWf+Ng/c
>>750
コメありり。

だけど、カードだと考えてくれ。
1が書かれたカード、2が書かれたカード、4が書かれたカード

 手札に無いカードは使えないのが条件なんだ。

渡すカードは+(プラス)  相手から貰うカードはー()マイナス
0752遊園地垢版2018/01/14(日) 17:50:54.99ID:vWf+Ng/c
カードの合計が種類数(異なる数字が書かれたカード)に関係なく、求める総数と等しい事には気づいた。
0754132人目の素数さん垢版2018/01/14(日) 18:21:34.68ID:pRmBgtM0
>>748
>から何か規則性はありますか?
あります、ありません、それが問題です。
0757132人目の素数さん垢版2018/01/14(日) 18:53:05.72ID:QZS2nyEG
>>748
7 15 17 だと直角三角形にならないからなあ
8 15 17 じゃないかなあ
n>m となる正整数nとmを使って
(n^2-m^2)^2+(2nm)^2=(n^2+m^2)^2となる
nとmを好きに選べば(n^2-m^2)と(2nm)と(n^2+m^2)でピタゴラス三角形が作れる
n=2,m=1が3 4 5
n=3,m=2が5 12 13
n=4,m=1が15 8 17
0758132人目の素数さん垢版2018/01/14(日) 18:54:28.82ID:09atsn3P
>>754
たとえば最初の3をxと置いたとき

5 12 13はx+2 4x 4x+1とか

7 15 17に含まれる7は3と4の和であるとか

いろいろと調べて
何かしらの関係性を導きたいのです
0759132人目の素数さん垢版2018/01/14(日) 19:50:57.68ID:LPeYtWU1
ピタゴラス数でググれ
0760132人目の素数さん垢版2018/01/14(日) 19:54:24.32ID:UoWoHnPk
>>748
(x,y,z)が、x^2+y^2=z^2を満たしているなら、
(X,Y,Z)=(z-y+t,z-x+t,2z-x-y+t) ただし、t= ±√(2*(z-x)(z-y)) or ±(x+y-z)   ・・・  (1)
で定まる(X,Y,Z)も、X^2+Y^2=Z^2を満たします。
例えば、(x,y,z)=(3,4,5) なら、t=±(3+4-5)=±2なので、
(X,Y,Z)=(5-4±2,5-3±2,10-3-4±2)=(3,4,5),(-1,0,1)らが、X^2+Y^2=Z^2を満たすことを主張しています。
これだけだと、あまり面白みがないかもしれないが、(x,y,z)が、x^2+y^2=z^2 を満たしているなら、
当然、(-x,y,z)もx^2+y^2=z^2を満たすので、(1)で x を -x に置き換えた
(X,Y,Z)=(z-y+t,z+x+t,2z+x-y+t) ただし、t=±(-x+y-z)  ・・・  (2)
も X^2+Y^2=Z^2 を満たすことが判ります。同様に、yの符号反転、x,yの符号反転したものから、
(X,Y,Z)=(z+y+t,z-x+t,2z-x+y+t) ただし、t=±(+x-y-z)  ・・・  (3)
(X,Y,Z)=(z+y+t,z+x+t,2z+x+y+t) ただし、t=±(-x-y-z)  ・・・  (4)
らが成立することも判ります。
(x,y,z)=(3,4,5)からスタートして(1)からは、(X,Y,Z)=(3,4,5),(-1,0,1)を見つけられましたが、
(2)からは、(X,Y,Z)=(5-4±4,5+3±4,10+3-4±4)=(5,12,13),(-3,4,5)
(3)からは、(X,Y,Z)=(5+4±6,5-3±6,10-3+4±6)=(15,8,17),(3,-4,5)
(4)からは、(X,Y,Z)=(5+4±12,5+3±12,10+3+4±12)=(21,20,29),(-3,-4,5)
らが、X^2+Y^2=Z^2を満たすことを見つけられます。正のピタゴラス数に限ると、
(3,4,5)から、(2)〜(4)の式を使って、(5,12,13),(15,8,17),(21,20,29)を見つけたことになります。
別の議論にはなりますが、この三つの式を繰り返し使うことによって、全てのピタゴラス数に到達することが知られています。
0761132人目の素数さん垢版2018/01/14(日) 20:29:18.08ID:1fovwpFR
{}
0762132人目の素数さん垢版2018/01/14(日) 20:29:38.37ID:1fovwpFR
{} -> {}
{1} -> {}
{2} -> {}
{3} -> {}
{1, 2} -> {}
{1, 3} -> {}
{2, 3} -> {}
{1, 2, 3} -> {1, 2, 3}
0763132人目の素数さん垢版2018/01/14(日) 20:30:11.04ID:1fovwpFR
{} -> {}
{1} -> {}
{2} -> {}
{3} -> {}
{1, 2} -> {}
{1, 3} -> {}
{2, 3} -> {2, 3}
{1, 2, 3} -> {1, 2, 3}

{} -> {}
{1} -> {}
{2} -> {}
{3} -> {}
{1, 2} -> {}
{1, 3} -> {1, 3}
{2, 3} -> {}
{1, 2, 3} -> {1, 2, 3}

{} -> {}
{1} -> {}
{2} -> {}
{3} -> {}
{1, 2} -> {1, 2}
{1, 3} -> {}
{2, 3} -> {}
{1, 2, 3} -> {1, 2, 3} 👀
Rock54: Caution(BBR-MD5:0be15ced7fbdb9fdb4d0ce1929c1b82f)
0764132人目の素数さん垢版2018/01/14(日) 20:30:49.70ID:1fovwpFR
{} -> {}
{1} -> {}
{2} -> {}
{3} -> {3}
{1, 2} -> {}
{1, 3} -> {3}
{2, 3} -> {3}
{1, 2, 3} -> {1, 2, 3}
0765132人目の素数さん垢版2018/01/14(日) 20:31:05.90ID:1fovwpFR
{} -> {}
{1} -> {}
{2} -> {}
{3} -> {3}
{1, 2} -> {}
{1, 3} -> {3}
{2, 3} -> {2, 3}
{1, 2, 3} -> {1, 2, 3}
0766132人目の素数さん垢版2018/01/14(日) 20:31:21.50ID:1fovwpFR
{} -> {}
{1} -> {}
{2} -> {}
{3} -> {3}
{1, 2} -> {}
{1, 3} -> {1, 3}
{2, 3} -> {3}
{1, 2, 3} -> {1, 2, 3}

{} -> {}
{1} -> {}
{2} -> {}
{3} -> {3}
{1, 2} -> {}
{1, 3} -> {1, 3}
{2, 3} -> {2, 3}
{1, 2, 3} -> {1, 2, 3}

{} -> {}
{1} -> {}
{2} -> {}
{3} -> {3}
{1, 2} -> {1, 2}
{1, 3} -> {3}
{2, 3} -> {3}
{1, 2, 3} -> {1, 2, 3} 👀
Rock54: Caution(BBR-MD5:0be15ced7fbdb9fdb4d0ce1929c1b82f)
0767132人目の素数さん垢版2018/01/14(日) 20:31:59.92ID:1fovwpFR
{} -> {}
{1} -> {}
{2} -> {2}
{3} -> {}
{1, 2} -> {2}
{1, 3} -> {}
{2, 3} -> {2}
{1, 2, 3} -> {1, 2, 3}

{} -> {}
{1} -> {}
{2} -> {2}
{3} -> {}
{1, 2} -> {2}
{1, 3} -> {}
{2, 3} -> {2, 3}
{1, 2, 3} -> {1, 2, 3} 👀
Rock54: Caution(BBR-MD5:0be15ced7fbdb9fdb4d0ce1929c1b82f)
0768132人目の素数さん垢版2018/01/14(日) 20:32:17.55ID:1fovwpFR
{} -> {}
{1} -> {}
{2} -> {2}
{3} -> {}
{1, 2} -> {2}
{1, 3} -> {1, 3}
{2, 3} -> {2}
{1, 2, 3} -> {1, 2, 3}
0769132人目の素数さん垢版2018/01/14(日) 20:32:56.37ID:1fovwpFR
{} -> {}
{1} -> {}
{2} -> {2}
{3} -> {}
{1, 2} -> {1, 2}
{1, 3} -> {}
{2, 3} -> {2}
{1, 2, 3} -> {1, 2, 3}
0770132人目の素数さん垢版2018/01/14(日) 20:33:40.69ID:1fovwpFR
{} -> {}
{1} -> {}
{2} -> {2}
{3} -> {}
{1, 2} -> {1, 2}
{1, 3} -> {}
{2, 3} -> {2, 3}
{1, 2, 3} -> {1, 2, 3}
0771132人目の素数さん垢版2018/01/14(日) 20:33:59.06ID:1fovwpFR
{} -> {}
{1} -> {}
{2} -> {2}
{3} -> {3}
{1, 2} -> {2}
{1, 3} -> {3}
{2, 3} -> {2, 3}
{1, 2, 3} -> {1, 2, 3}
0772132人目の素数さん垢版2018/01/14(日) 20:35:40.80ID:1fovwpFR
{} -> {}
{1} -> {}
{2} -> {2}
{3} -> {3}
{1, 2} -> {2}
{1, 3} -> {1, 3}
{2, 3} -> {2, 3}
{1, 2, 3} -> {1, 2, 3}
0773132人目の素数さん垢版2018/01/14(日) 20:36:11.99ID:UdDVE1LP
スクリプトですかこれ?
0774132人目の素数さん垢版2018/01/14(日) 20:47:54.71ID:09atsn3P
■ピタゴラスの定理の短い二辺の長さ

3 4 

5 12 

8 15 

だけから規則性を導きたいです

>>760
その式から(7,24,25)はどうやって導くのですか?
0776132人目の素数さん垢版2018/01/14(日) 21:02:06.53ID:/iIQ3OWQ
>>746
ax^2-x-1
=a(x^2-x/a)-1
=a(x-1/2a)^2-1/4a-1 まではわかりました
0778132人目の素数さん垢版2018/01/14(日) 21:16:25.33ID:pRmBgtM0
0/1 + 1/1 = 1/2
0/1 + 1/2 = 1/3  1/2 +1/1 = 2/3
0/1 + 1/3 = 1/4  1/3 + 1/2 = 2/5  1/2 + 2/3 = 3/5  2/3 + 1/1 = 3/4
0779132人目の素数さん垢版2018/01/14(日) 21:30:08.44ID:UoWoHnPk
>>774
与えられたピタゴラス数が、どのような経緯(=(2)〜(4)の式の適用順位)を辿ってできたかを考えるときは、(1)式が活躍します。
(7,24,25)を(1)により変換します
(25-24±6,25-7±6,50-7-24±6)=(7,24,25),(-5,12,13)
このうち、負の数が現れている(-5,12,13)に注目します。
x のところだけに負の数が現れているので、このピタゴラス数は直前に(2)式が適用されたと判断できます。
もし、y のところにだけ負の数が現れていたら(3)式が、xとy のところに負の数が現れていたら(4)式が
直前に使われたと判断できます。
実際、(5,12,13)に(2)を適用すると、(13-12±6,13+5±6,26+5-12±6)=(7,24,25),(-5,12,13)が得られます。
0780132人目の素数さん垢版2018/01/14(日) 21:53:17.47ID:UoWoHnPk
>>774
何らかの規則と言うことなので、次はどう?

c^2=a^2+b^2 のaとbが与えられて、a^2+b^2が平方数であることは当たり前ですが、
aが奇数、bが偶数の時
±b+√(a^2+b^2)=±b+c
2(±a+√(a^2+b^2))=2(±a+c)
らも平方数
0781132人目の素数さん垢版2018/01/14(日) 22:04:01.17ID:pRmBgtM0
>>778
>0/1 + 1/1 = 1/2 => (2^2-1^2)^2 +(2・1・2)^2 = (2^2+1^2)^2
>0/1 + 1/2 = 1/3  1/2 +1/1 = 2/3
>0/1 + 1/3 = 1/4  1/3 + 1/2 = 2/5  1/2 + 2/3 = 3/5  2/3 + 1/1 = 3/4
1/2 => (2^2-1^2)^2 +(2・2・1)^2 = (2^2+1^2)^2
1/3 => (3^2-1^2)^2 +(2・3・1)^2 = (3^2+1^2)^2
2/3 => (3^2-2^2)^2 +(2・3・2)^2 = (3^2+2^2)^2
1/4 => (4^2-1^2)^2 +(2・4・1)^2 = (4^2+1^2)^2
2/5 => (5^2-2^2)^2 +(2・5・2)^2 = (5^2+2^2)^2
3/5 => (5^2-3^2)^2 +(2・5・3)^2 = (5^2+3^2)^2
3/4 => (4^2-3^2)^2 +(2・4・3)^2 = (4^2+3^2)^2
0782132人目の素数さん垢版2018/01/15(月) 02:12:43.07ID:fdRl2/eX
問題というか質問なんだがlogの底に√3とかって入る?もちろん無理数eとかが入るのはわかるんだが、√とかが入って計算とかできるのかなと思って。高校生とかに聞かれたらどうすればいいのかなって。
0785132人目の素数さん垢版2018/01/15(月) 06:09:33.27ID:EVW63M4G
高校までの数の定義では表せない微分、積分、の最も簡単な具体例にはどんなものがありますか?
0786132人目の素数さん垢版2018/01/15(月) 10:18:38.24ID:/z+5bnHp
>>740
ax^2-x-1
=a(x^2-x/a)-1
=a(x-1/2a)^2-1/4a-1 まではわかりましたそれいこうが意味不明です
0787132人目の素数さん垢版2018/01/15(月) 11:41:29.99ID:UJNES4GC
>>776
それじゃ、続きだけ。

よって、xy-平面上で、2次関数 y=ax^2-x-1 のグラフCの頂点は A(1/2a、-1/4a-1) で、Aのy座標は、y=-1/4a-1。
1):a>0 のとき。グラフCは下に凸となる。
x=1 とすると、y=a-2 だから、Cはxy-平面上の点 (1,a-2) を通る。
x=-1 とすると、y=a だから、同様に考えると、Cは点 (-1,a) を通る。
ここで、2次方程式 ax^2-x-1=0 の2つの解α、βについて、-1<α,β<1 であり、α≠β。
また、2次多項式 ax^2、ax^2-x-1 について、x^2 の各項の単項式 ax^2 の各係数はaで同じだから、
2次関数 y=ax^2-x-1 のグラフは2次関数 y=ax^2 のグラフを或る方向に平行移動して重ね合わせることが出来る。
従って、中間値の定理を使って考えると、-1<α,β<1 なることと、a-2>0、a>0、-1/4a-1<0 がすべて成り立つこととは同値である。
故に、仮定から、aについて a-2>0、a>0、-1/4a-1<0。この連立不等式を解くと、aが存在し得る範囲は a>2。
これは a>0 としていることに反しない。故に、aの存在範囲は a>2。
2):a<0 のとき。グラフCは上に凸となるから、同様に考えると、
-1<α,β<1 なることと、a-2<0、a<0、-1/4a-1>0 がすべて成り立つこととは同値である。
故に、仮定からaについて a-2<0、a<0、-1/4a-1>0。この連立不等式を解くと、aが存在し得る範囲は a<-1/4。
これは a<0 としていることに反しない。故に、aの存在範囲は a<-1/4。
1)、2)から、-1<α,β<1 となる条件は、a>2 または a<-1/4。
0788132人目の素数さん垢版2018/01/15(月) 11:48:19.68ID:q/8nd93a
X4−3X2+9を因数分解せよって問題で詰まってしまったのですがどなたか教えて頂けないでしょうか
0789132人目の素数さん垢版2018/01/15(月) 11:49:15.11ID:UJNES4GC
>>786
>>776の方ですよね。
>>787>>746の意味分かりましたね。
数Tの範囲で直観的に考えるのであれば、必要十分条件のところで、
中間値の定理はいらないとは思います。
0791132人目の素数さん垢版2018/01/15(月) 12:00:55.93ID:DYKUkYXf
「位相的双対律」の証明を教えてください。
0792132人目の素数さん垢版2018/01/15(月) 12:08:52.53ID:UXcgXv6g
A,B,CをGの正規部分群とする
A⊂C ⇒ A(B∩C) = (AB)∩C
を示して下さい
0793132人目の素数さん垢版2018/01/15(月) 12:20:14.73ID:DYKUkYXf
双対性についてですが、なぜ、双対性が成り立つとだけ書いて、証明を書かない本ばかりなのでしょうか?
0795132人目の素数さん垢版2018/01/15(月) 12:52:29.82ID:SK6tvX5B
別人だけど…
>>794

この発想、別に自力で思いつけなくてもいい。
因数分解の問題のところに絶対類題があるから、そこでやり方を覚えればいい。

数学は暗記だとか絶対に思わないけど、毎回車輪を再発明しなくちゃと思うのも変すぎる。
0796132人目の素数さん垢版2018/01/15(月) 12:57:12.25ID:UJNES4GC
>>788
どういう風に因数分解するか問題だけど、複素数の範囲で。
xは実数として x=X^2 とおき、偏角の範囲は -π<θ≦π。
X^4−3X^2+9
=x^2−3x+9
=(x−3/2)^2−9/4+9
=(x−3/2)^2+27/4、
=(x−3/2)^2−(−27/4)
=(x−3/2)^2−(3i√3/2)^2
=(x−3/2+3i√3/2)(x−3/2−3i√3/2)
={ X^2−3(1/2−i√3/2) }{ X^2−3(1/2+i√3/2) }
={ X^2−3e^{-2πi/3} } { X^2−3e^{2πi/3} }
=( X+√3e^{-πi/3} )( X−√3e^{-πi/3} )・ ( X+√3i・e^{πi/3} )( X−√3i・e^{πi/3} )
=( X+√3e^{-πi/3} )( X−√3e^{-πi/3} ) ・( X+√3e^{πi/2}・e^{πi/3} )( X−√3e^{πi/2}・e^{πi/3} )
=( X+√3e^{-πi/3} )( X−√3e^{-πi/3} ) ・( X+√3e^{5πi/6} )( X−√3e^{5πi/6} )
={ X+√3(1/2−i√3/2) }{ X−√3(1/2−i√3/2) } ・{ X+√3(−√3/2+i/2) }{ X−√3(−√3/2+i/2) }
=( X+√3/2−3i/2) )( X−√3/2+3i/2 )( X−3/2+√3i/2 )( X+3/2−√3i/2 )。
0797132人目の素数さん垢版2018/01/15(月) 13:08:17.49ID:DYKUkYXf
松坂和夫著『集合・位相入門』のp.159定理7'の証明は以下であっていますか?

2^S ∋ M → M^a ∈ 2^S



(Ki)-(Kiv)を満たすとする。

写像 2^S ∋ M → M^(cac) ∈ 2^S はpp.154-155の(Ii)-(Iiv)を満たすことを以下で示す:

(Ii) S^(cac) = φ^(ac) = φ^c = S

(Iii) M^c ⊂ M^(ca), M ⊃ M^(cac)

(Iiii) (M ∩ N)^(cac) = (M^c ∪ N^c)^(ac) = (M^(ca) ∪ N^(ca))^c = M^(cac) ∩ N^(cac)

(Iiv) M^(caccac) = M^(caac) = M^(cac)
0798132人目の素数さん垢版2018/01/15(月) 13:12:24.25ID:DYKUkYXf
p.155定理7より

M^i1 = M^(cac) となるような位相空間 (S, O1) が存在する。

p.258(2.7)より、

M^i1 = M^(ca1c)

が成り立つ。

よって、

M^(cac) = M^(ca1c)
M^a = M^a1

が成り立つ。
0799132人目の素数さん垢版2018/01/15(月) 13:16:15.15ID:DYKUkYXf
一意性:

二つの位相空間 (S, O1), (S, O2) が存在して、

M^a1 = M^a
M^a2 = M^a

を満たすとすると

M^i1 = M^(ca1c) = M^(cac) = M^(ca2c) = M^i2

となる。

p.155定理7での一意性より、

(S, O1) = (S, O2)

でなければならない。
0802132人目の素数さん垢版2018/01/15(月) 14:43:51.69ID:Nr3Tqe6F
あ...補足ですが
下の画像が
元の問題のやつで、それのproblem11になります
0805132人目の素数さん垢版2018/01/15(月) 15:33:18.83ID:UJNES4GC
強引に因数分解した結果が複雑な式になるから、そういうのは関係ない。
X^4−3X^2+9=0 になるXの値も見つけるのは難しい。
因数分解は計算テクニックの1つでやって終わり。
0806132人目の素数さん垢版2018/01/15(月) 15:34:56.59ID:aQs2s/kN
>>801
ぱっと見、元のマルコフ連鎖の図からa+b=1とかp+q+r=1とかは条件として使うと思うんだけどね
0808132人目の素数さん垢版2018/01/15(月) 16:35:03.86ID:Nr3Tqe6F
>>807
ん、逆ですか?
チャップマン-コルモゴロフでHn,Snを解くとこの順序で合っていたのですが…
0809132人目の素数さん垢版2018/01/15(月) 17:36:53.94ID:Nr3Tqe6F
あと連投申し訳ないですが
>>670の主張が合っているのか教えてください
0810132人目の素数さん垢版2018/01/15(月) 17:55:17.25ID:nG9WfsmH
>>792
お願いします
0811132人目の素数さん垢版2018/01/15(月) 18:21:44.46ID:9ZNszaTp
>>787
これ数1じゃないのですか?中間値の定理なんてならってませんし試験の範囲でないです
拾ってきた問題だからレベル高いのかな
ax^2-x-1=0 の2つの解α、βについて、-1<α,β<1 なんで不等号の向きこうなりましたか?
0812132人目の素数さん垢版2018/01/15(月) 18:31:35.94ID:mZ+PQuFM
>>810
ab∈(AB)∩C(a∈A,b∈B)
aおよびabはCの元だからb∈C(∩B)
よってab∈A(B∩C)
0813132人目の素数さん垢版2018/01/15(月) 18:34:08.23ID:aQs2s/kN
>>801
ともあれ、この手の連立漸化式は正方行列とベクトルの積に置き換えて、正方行列の累乗を使って一般式を求めることができる
あとは一般式を級数に代入すればよい
0814132人目の素数さん垢版2018/01/15(月) 18:34:11.87ID:3kld9oW2
「正規」という条件は不要?
A(B∩C) ⊂ (AB)∩C
a∈A,b∈B∩Cとする。ab∈ABは明らかでA⊂Cだからab⊂Cも良い
(AB)∩C ⊂ A(B∩C)
a∈A,b∈B,ab∈Cとする。b∈Cをいえばよい
ab=c(∃c∈C)よりb=(a^-1)c∈C
0815132人目の素数さん垢版2018/01/15(月) 18:42:25.24ID:3kld9oW2
>>785
質問の意味が明瞭でないけど、
例えば 1/(x^2+1) の原始関数は高校じゃ習わないはず
(逆正接(アークタンジェント))
ほかにも 1/(logx) の原始関数とか無数にある
微分のほうはどういう意味だろう?「高校で習う微分可能な関数」の導関数は
必ず「高校で習う関数」になると思うけど
0816132人目の素数さん垢版2018/01/15(月) 18:54:27.63ID:wezANHdJ
>>813
なるほど、やってみます
0817132人目の素数さん垢版2018/01/15(月) 19:00:26.40ID:g92Xv0xu
■2つの封筒問題(two envelopes problem)

2種類の小切手があり、1つの小切手には
他方の4倍の金額が書き込まれています

中身が分からないように、それぞれ封筒に入れます

あなたは、どちらか1つの封筒を選ぶことができます

封筒を開けると10万円の小切手が入っていました

もし不満なら、残りの封筒と交換できます

あなたは交換しますか?しませんか?
0818132人目の素数さん垢版2018/01/15(月) 19:08:51.47ID:3kld9oW2
>>655
そう。中心角をθ1,...,θnとして周長をこれらの式で
表して、ラグランジュの未定乗数法つかうと
θ1=...=θn と全部等しくならないといけない
細かい論点ははしょった
0819132人目の素数さん垢版2018/01/15(月) 19:26:25.17ID:wezANHdJ
>>813
正則行列を対角化しようとすると固有値がとんでもないことに〓
0820132人目の素数さん垢版2018/01/15(月) 19:40:41.00ID:aQs2s/kN
>>819
固有値を式として求める必要はない
λ1、λ2 とでもして計算を進めて、計算の結果でλ1+λ2やλ1λ2が出てきたら固有多項式の係数に戻せばよい
0821132人目の素数さん垢版2018/01/15(月) 19:45:36.13ID:yIjavdoR
>>811
f(0) = -1 < 0より、a < 0のときはグラフとx軸は交わらないのでa > 0のときのみ考える。
a > 0のとき、グラフとx軸は必ず2点で交わるので、f(-1) = a > 0, f(1) = a-2 > 0の共通範囲を求めてa > 2。これはa > 0を満たす。
よって求める範囲はa > 2。

自分が解くべき問題はしっかり選んだ方がいいと思うよ。
0822132人目の素数さん垢版2018/01/15(月) 19:45:41.80ID:wezANHdJ
>>820
このAを対角化してから累乗するっていう方向性はわかりました
でも対角化できない(>_<)


https://i.imgur.com/9AkFlYV.jpg
0823132人目の素数さん垢版2018/01/15(月) 20:12:45.75ID:KdIP1Ead
>>822
なんで?
((λ1,0),(0,λ2))
に対角化されるよ?
0824132人目の素数さん垢版2018/01/15(月) 20:13:49.93ID:KdIP1Ead
対角化できないのは重根の時
重根で対角化できるのは元から対角行列(重根だからスカラ行列)の場合のみ
0825132人目の素数さん垢版2018/01/15(月) 20:57:58.20ID:wezANHdJ
>>824
あ、できないってのは能力的な話です
0826132人目の素数さん垢版2018/01/15(月) 21:01:48.72ID:6Rs5mrCs
五十そ生まれなおそう
0828132人目の素数さん垢版2018/01/15(月) 21:21:20.61ID:aQs2s/kN
>>825
能力が無いようには見えないんだけどね
前述の通り、固有値は変数のまま固有ベクトルを求めてそれらを束ねた正則行列とその逆行列を作ればよい
2×2行列ならそれほど手間ではなかろうかと思う
固有値が重根となる場合は別途考慮が必要だがこの場合の累乗を求めるのはもっと簡単
0829132人目の素数さん垢版2018/01/15(月) 21:25:14.55ID:hDdx4MhU
>>821セーター数1れべる?
0830132人目の素数さん垢版2018/01/15(月) 21:32:42.39ID:wezANHdJ
>>828
最近線形やってなかったので大分知識が抜け落ちてしまいまして…
ともあれ、色々とアドバイスありがとうございます
もう少しこねくり回してみます
0831132人目の素数さん垢版2018/01/15(月) 22:04:39.43ID:DUXDgO4Y
A={a,b,c}の3つの要素からなる集合とする。集合A上の関係は何種類あるか理由を付して答えよ。

関係Rは、Aの要素の組の部分集合によって表される。Aの要素の組は3^2=9通りあり、その部分集合は2^9通りである。

この問題で、9通りまであるのはわかりましたが、2の9乗通りあるというのがわかりません
なぜそれぞれに2通りあるのですか?
0833132人目の素数さん垢版2018/01/15(月) 22:20:36.37ID:yIjavdoR
>>829
すまん、訂正>>821
ax^2-x-1=0は異なる2つの実数解をもつので、a ≠ 0であり、判別式D > 0。
これより-1/4 < a < 0、0 < a - @
また、放物線の軸はx = 1/(2a)である。

[1] a > 0 のとき
D > 0, f(-1) > 0, f(1) > 0, -1 < 軸 < 1 が同時に成り立つ。
f(-1) > 0 より a > 0 - A, f(1) > 0 より a > 2 - B
軸は0 < x < 1の範囲にあるので、0 < 1/(2a) < 1。 2a > 0をかけてa > 2 - C
a > 0 のとき、@, A, B, Cを同時に満たすaの範囲は a > 2。
[2] a < 0 のとき
D > 0, f(-1) < 0, f(1) < 0, -1 < 軸 < 1 が同時に成り立つ。
f(-1) < 0 より a < 0 - D, f(1) < 0 より a < 2 - E
軸は-1 < x < 0の範囲にあるので、-1 < 1/(2a) < 0。 2a < 0をかけてa < -1/2 - F
a < 0 のとき、@, D, E, Fを同時に満たすaはない。

よって a > 2

個人的にはセンター試験より難しいと思うが。
0834132人目の素数さん垢版2018/01/15(月) 22:46:08.51ID:SK6tvX5B
>>740
記述式の答案を作るとして、一番簡潔そうなのは、
>>742のを少し弄って

x=0は解ではないので a=1/x + 1/x^2
1/x = tと置いて、t < -1、t >1 の範囲で、y=a と y=t+t^2 が二つの解をもつ条件を
求めればいいので、
y = t^2 + tのグラフを書いてグラフより
a > 2

(一応数1の範囲の解法…だとは思う。もしかしたら数2かも)
0835132人目の素数さん垢版2018/01/15(月) 22:52:37.27ID:FbciWArh
反比例のグラフは中学で習ってんだからそれ応用して描きゃええんじゃね
y=1/x (これは描けないなら義務教育からやり直し)
y=1/x^2 (x^2=t とでも置けば概形はわかるはず)
を描いといてそのy座標を合計したらいい
0837132人目の素数さん垢版2018/01/16(火) 00:17:40.55ID:ce41RVCQ
>>836

1/4 + y =(1/2 + 1/x)^2 ≧ 0,

等号は x=-2 のとき。

x→-∞ で y→0,

x<-1 で -1/4≦y<0,

x=-1 で y=0,

-1<x<0 で y>0,

x→0 で y→∞
0838132人目の素数さん垢版2018/01/16(火) 00:26:50.81ID:PWgP6/+/
a>0 A(a , loga) E(2a+1/a , −a^2+loga−1))のときAEの最小値を求めよって問題なんですけど
AE=√{(a+1/a)^2+(a^2+1)^2}ってなってそれぞれa=1のときに最小値になるからa=1でAE=2√2だと思ったのですが解答をみるとベクトルで表して大きさに直して微分するという方法でa=1/√2で3√3/2となってました
どこでミスしてますか?
0839132人目の素数さん垢版2018/01/16(火) 00:30:03.65ID:4Xmr8cR4
おとなしいA君は60分で大盛りカツ丼を完食できます。
普通のB君は20分で大盛りカツ丼完食できます。
では、2人で協力して大盛りカツ丼を食べた場合、何分で完食できるでしょう?
0840132人目の素数さん垢版2018/01/16(火) 00:35:35.13ID:xCUnWJat
何のためにどの程度のレベルの知識を仮定して問題を解いているのか考えたほうがいいと思うよ
0843132人目の素数さん垢版2018/01/16(火) 00:39:23.27ID:xCUnWJat
>>838
後半 (a^2+1)^2が最小になるのは、a=0のときだ
そのまま、AE^2を作って、微分するなりなんかよさそうな変形を思いつくなりするのがいいんじゃない?
0844132人目の素数さん垢版2018/01/16(火) 00:42:12.75ID:PWgP6/+/
あ、確かに…
aで括って相加平均で出したんですけどダメなんですか??
そのまま相加平均してもa=1になるような…
どこかおかしいですか?
0845132人目の素数さん垢版2018/01/16(火) 01:02:27.07ID:ss82hpah
>>844
正数である標本について相加平均≧相乗平均、等号はすべての数が等しいとき
なので、
相乗平均が一定という条件下では、この性質を使えば、等号が成立するとき相加平均が最小といってよい
しかし、
相乗平均が一定という条件がなければ、一概に等号が成立するときに相加平均が最小とは言えない
ってことです
0846132人目の素数さん垢版2018/01/16(火) 01:03:50.23ID:PWgP6/+/
括ったaがどうにもならないですね!

相加平均・相乗平均の関係式はただの不等式(値域ではない)で、
a²+1=a(a+a⁻¹)≧2a
この不等式は、a²+1が常に2a以上ってだけで役に立ちません
等号成立するときもa²+1=2aが成り立つだけで、このときにa²+1は最小になる保証はどこにも無いです

a+a⁻¹≧2
この不等式も、a+a⁻¹が常に2以上ってだけです
これとa+a⁻¹=2となるaがあることから、a+a⁻¹は最小値2を取るといえますが、
a+a⁻¹が2以上の全ての値をとることは示されていません

ということですか??
0847132人目の素数さん垢版2018/01/16(火) 07:46:32.96ID:eoZ+h/An
すみません、大学受験レベルの数学ってここで聞いてもよろしいでしょうか?皆さんの書いてらっしゃることがあまりにも難しいので気が引けて。
他スレでふさわしいスレありますか?一応探したのですが…
0848132人目の素数さん垢版2018/01/16(火) 09:45:02.35ID:xCUnWJat
>>847
他にもあるかもしれないけど、このスレでも大学受験レベルの質問は結構出てるよ
小難しいことを書いてる人は単なる荒らしの一種
0849132人目の素数さん垢版2018/01/16(火) 09:54:46.28ID:eoZ+h/An
レスありがとうございます。
では、恥ずかしながら質問させて頂きます。
三角形ABCは、AB=4、AC=6、角BAC=60度である。
ABCの外接円をOとする。辺BCを4:3に内分する点をD、直線ADと円Oの交点のうち、A以外の点をEとする。BE:CEを求めよ。

以上が問題です。解答を読むと直線AEが直径となることを使っているようなのですが、なぜAEが直径となるのかがわかりません。
レベルが低くて申し訳ありませんが、よろしくお願いいたします。
0851132人目の素数さん垢版2018/01/16(火) 11:02:46.83ID:LuK1EJ41
「S の空でない部分集合 O が開集合であるための必要十分条件は、
O の任意の点 x に対して、 O が x の近傍となっていることである。」

と書いてあるのですが、なぜ、

S の部分集合 O が開集合であるための必要十分条件は、
O の任意の点 x に対して、 O が x の近傍となっていることである。

と書いていないのでしょうか?
0853132人目の素数さん垢版2018/01/16(火) 12:18:15.49ID:ss82hpah
>>852
何ヵ所かどういう操作をしているか分からない箇所があるが、そもそも最初の式の2行目と4行目を足したら0 0 0 0になる時点で値が0だと気づいてもよい
0854132人目の素数さん垢版2018/01/16(火) 12:24:10.20ID:5KR+oO46
>>849
直径になることは使わずに解いた
直径になるかどうかは確認してない

余弦定理よりBC=2√7
よってBD=8/√7、CD=6/√7
次に△BED∽△ACDより、BE:ED=1:1/√7
同様に△CED∽△ABDより、CE:ED=1:2/√7
これを比べて、BE:CE=2:1
0856132人目の素数さん垢版2018/01/16(火) 12:41:13.99ID:5lLu6+BT
0°≦180°、tanθ=2のときの1/(1-sinθ)+1/(1+sinθ)


1/(1-sinΘ)+1/(1+sinΘ)
={(1+sinΘ)+(1-sinΘ)}/(1-sinΘ)(1+sinΘ)
=2/(1-sin^2Θ)
=2/cos^2Θ ・・・@
までわかりましたが
0857132人目の素数さん垢版2018/01/16(火) 12:43:52.12ID:5lLu6+BT
1+tan^2Θ=1/cos^2Θの公式つかうまでわわかるのですが
答えあいません
0858132人目の素数さん垢版2018/01/16(火) 12:46:11.82ID:w2ZUfV0D
>>857
答えは10
0860132人目の素数さん垢版2018/01/16(火) 12:53:08.78ID:5KR+oO46
>>859
あと確認したが確かに直径になっている(長さから計算した)
解答ですぐに直径を使ってるということは、計算なしで直径を見抜く方法があるはず
俺は初等幾何すら苦手だが、学校で先生にでも聞いてくれ、勉強になると思う
0861132人目の素数さん垢版2018/01/16(火) 12:54:26.05ID:w2ZUfV0D
sin=2/√5
1-sin=(√5-2)/√5
1+sin=(√5+2)/√5
1/(1-sin)=√5/(√5-2)
1/(1+sin)=√5/(√5+2)
1/(1-sin)+1/(1+sin)=√5/(√5-2)+√5/(√5+2)=(√5(√5+2)+√5(√5-2))/(√5-2)(√5+2)=5+5=10
0863132人目の素数さん垢版2018/01/16(火) 13:34:21.58ID:5lLu6+BT
放物線y=X^2+6x+5kはx軸と2点A.Bで交わる。ただしKは自然数とする。1)点A.B及びC(0.-10)を通る放物線の頂点をPとすると、Pの座標は(-ア、イ)2)kを求めよ
a=-2までわかりましたが
頂点はx=-3を代入
頂点はx=-3はどこからでたのですか?
0864132人目の素数さん垢版2018/01/16(火) 13:49:24.62ID:IkdhXsCM
わざとか知らないけど、ダメな人って高率で主語をぼかすよね
0865132人目の素数さん垢版2018/01/16(火) 14:09:21.70ID:5lLu6+BT
>>856
2/cos^2Θ

1+tan^2Θ=1/cos^2Θ
1+4=5
なので2/5ではないですか?
0866132人目の素数さん垢版2018/01/16(火) 14:15:07.53ID:Qrn7DXLS
>>851
空であるか空でないか?細かいことは気にしない!俺は空でないという条件入れ忘れるけど気にしてないよ。
0868132人目の素数さん垢版2018/01/16(火) 14:26:05.03ID:lU+fYz1U
nd=In(de^s)+In(nu/s)
この方程式をほじくり回すと
se^(nd)=nude^s
となるらしいが解けるやつおる?
0869132人目の素数さん垢版2018/01/16(火) 14:31:34.36ID:xCUnWJat
>>860
俺も直径になる理由がわからん
BE : CEは三角形の面積比を考えればすぐだけど…
(△ABE : △ACE = AE : CE , △ABE = ABxBExsin∠ABE , △ACE = ACxCExsin∠ACE以下略)

AEが直径だったら、何か楽な解法が出てくるのかそっちの方も気になってきた。
0871132人目の素数さん垢版2018/01/16(火) 14:40:12.85ID:cSlSNMMB
>>849
オレの計算だと直径でねーぞ
AE=16/√7 , ホントの直径=4√7/√3
になった
座標で計算して
A=(0,0), B=(4,0), C=(3,3√3), D=(24/7,12√3/7), E=(32/7,16√3/7)
円中心 O=(2,4/√3)
だったから検算してみな
0873132人目の素数さん垢版2018/01/16(火) 14:49:43.94ID:uyeT8nXH
すみません!
解答には、三角形ABE:三角形ACE=1/2・4・BE:1/2・6・CE=2BE:3CE(以下略)
とだけ書いてあったので、私がAEは直径なのだと思って勘違いしていました!
解答は、sin ABEとsin ACEを略していたのですね。
申し訳ありませんでした。
ただ、受験生向けの問題集で、そこまで略さなくてもいいのに、とは思いますが…
とにかく、考えさせてしまって
すみませんでした!
0874132人目の素数さん垢版2018/01/16(火) 15:10:59.95ID:xCUnWJat
>>869
なんか式グチャグチャやねん
頭の中だけで書こうとしたのは大間違いだった
もう理解してくれたみたいなので書きなおさないねん
ねんねん
0875132人目の素数さん垢版2018/01/16(火) 16:12:05.69ID:w2ZUfV0D
>>865
どういう計算?
0876132人目の素数さん垢版2018/01/16(火) 16:14:51.92ID:w2ZUfV0D
>>868
くらだん
0877132人目の素数さん垢版2018/01/16(火) 16:16:52.00ID:w2ZUfV0D
>>863
放物線の対称軸
0878132人目の素数さん垢版2018/01/16(火) 16:31:55.55ID:ss82hpah
>>849
実際、AFが外接円の直径になるように点Fをとった場合、
外接円の中心が辺の二等分線の交点であること等を利用して比を求めるとBF:CF=4:1になった
この結果と>>854の結果が違っていたので少し悩んだ
なお、直径AFと辺BCの交点Pは、辺BCを8:3に内分する
0879132人目の素数さん垢版2018/01/16(火) 17:01:52.71ID:ss82hpah
>>849
さらにこの問題、角Aの大きさによらず解くことができる
>>854で示された相似性△BED∽△ACDと△CED∽△ABDを使えばBE:CE=AC・BD:AB・CDが得られるので、辺や線分の比だけでBE:CEを求めることができる
0880132人目の素数さん垢版2018/01/16(火) 17:34:28.16ID:34r4iPoF
>>839
A君とB君あわせて60分で大盛りカツ丼を四杯完食可能

ゆえに、大盛りカツ丼一杯であれば15分で完食する
0882132人目の素数さん垢版2018/01/16(火) 17:44:31.07ID:MucaOH+c
広義積分(0→∞) 1/{(1+x^3)^(1/2)}dxの収束発散を調べよ。


という問題で、f(x)≦g(x)となるような関数g(x)=M/x^αを見つければ収束すると考えていて。
g(x)=1/x^(3/2)と置けば良いと思うのですが、これで正解でしょうか?
0884132人目の素数さん垢版2018/01/16(火) 17:51:50.10ID:0KhH+q/U
本当に、その範囲でそういう不等式が成り立っているのか、ということですね
0885132人目の素数さん垢版2018/01/16(火) 18:21:46.47ID:34r4iPoF
>>817
40万か2万5千円か可能性が半々、
つまりチェンジした場合の期待値は21.25万円
プレイヤはノーリスクですでに10万円手に入っている
0886132人目の素数さん垢版2018/01/16(火) 18:42:16.59ID:5lLu6+BT
2/cos^2Θ がなんでsin=2/√5なるのかも
1-sin=(√5-2)/√5なるのかもわかりません
0887132人目の素数さん垢版2018/01/16(火) 20:08:13.04ID:ss82hpah
>>856
>tanθ=2のとき
>1/(1-sinΘ)+1/(1+sinΘ)
>={(1+sinΘ)+(1-sinΘ)}/((1-sinΘ)(1+sinΘ))
>=2/(1-sin^2Θ)
>=2/cos^2Θ ・・・@
の続きはこう
=2(1/cos^2Θ)
=2(1+tan^2Θ)
=2(1+2^2)
0888132人目の素数さん垢版2018/01/16(火) 22:11:49.94ID:PWgP6/+/
微分方程式です
yは全部y(x)と思ってくださいm(__)m

(6)教えてください
右辺が x・e^ax のような時の解法がわかりませんm(__)m


https://i.imgur.com/jJUyszG.jpg
0890132人目の素数さん垢版2018/01/16(火) 22:58:52.26ID:AcMYtlGT
lim(x→0)(sinx)/x=1 を仮定せずに(sinx)'= cosx を導けるか??
センター試験で弧度法の定義も出た事もあり、弧度法を上手く使えば0<x<π/2では成立する事を簡単な積分の知識で示せました

使うのは逆写像の微分法と弧長積分位でしょうか?
多分間違えてはないとは思うけど何かミス等有りましたらお教え下さいm(_ _)m
https://i.imgur.com/LYQD7ZN.jpg

私は解析関数は積分を用いるべきだという宗派です(°▽°)
これって一般角に拡張するのは大変ですよね...

ℝ-{-π/2+2nπ|n∈ℤ}の元xに対して
sinx=-sinXと定義する。但しXは X∈(-π/2,π/2]である整数nを用いてx=X+nπ と表せる元の事。
この様なXは各xに対して一意に定まるのでwell- defindである。
{-π/2+2nπ|n∈ℤ}の元xに対してはsinx=sin(-π/2)=-1と定義する

とかでいけないですかね?
0891132人目の素数さん垢版2018/01/16(火) 23:00:04.44ID:PWgP6/+/
>>889
和の部分はどこから判断が…(>_<)
今回の場合だと最大2回微分てとこから1+2で3乗…?
0892132人目の素数さん垢版2018/01/16(火) 23:03:30.31ID:MbyXf0Hd
>>891
テキストに書いてあると思うが
右辺が x*e^(4x) だから (1次式)*e(4x) の形の特殊解があるとするのがふつうだが
この問題は D^2-8D+16 = 0 が 4 を2重解に持つので x^2 がかかる
0893132人目の素数さん垢版2018/01/16(火) 23:29:18.82ID:ce41RVCQ
>>888
y(x)= u(x)e^(4x)を与式に入れる。

0 =(左辺)-(右辺)={u "(x)- x}e^(4x),

u(x)=(1/6)x^3 + c1・x + c0,

y(x)={(1/6)x^3 + c1・x + c0}e^(4x),
0894132人目の素数さん垢版2018/01/16(火) 23:44:26.95ID:PWgP6/+/
私微分方程式した事ないから分からないですが、
(D-4)^2(y)=D^2y-8Dy+16yと定義する。
これは線形写像であり、 D-4の逆写像は…

って感じでいけないかな…??

なんかググったらこんなの出てきました…!
(D+a)は可微分写像全体の集合から可微分写像全体の集合への全射なので右逆写像が存在し、それは(1/(D+a))で与えられる。
つまり(D+a)(1/(D+a))(y)=yを満たす。よって(D+a)(y)=q(x)の解の1つとして(1/(D+a))(q(x))が与えられるって事かな?

https://i.imgur.com/yk9JqI8.jpg

これ使えば今回は(1/(D-4))を右辺に二回作用させるだけで解の1つが出てくる?(x^3 exp(4x)/6が出てきました)

今回はなんかexp(4x)とexp(-4x)が出てきて良い感じに消えてくれました笑

微分方程式少し調べたけどかなり奥が深そうですね!微分作用素の話になるのかな…??
0895132人目の素数さん垢版2018/01/17(水) 00:05:21.09ID:KlbWP3TU
>>882

1/(1+x^3)= t とおく。

(与式)=(1/3)∫[0,1]t^(-5/6)(1-t)^(-2/3)dt
=(1/3)∫[0,1]t^(p-1)(1-t)^(q-1)dt
=(1/3)B(p,q)
=(1/3)Γ(p)Γ(q)/Γ(p+q)
=(1/3)Γ(p)Γ(q)/√π
= 2.80436421065090852235

ここに p=1/6,q=1/3,p+q=1/2.
0896132人目の素数さん垢版2018/01/17(水) 00:17:02.53ID:KlbWP3TU
>>882

R >> 1 とする。

∫[0,R]f(x)dx
≦ ∫[0,1]dx + ∫[1,R]x^(-3/2)dx
= 1 + 2(1 - 1/√R)
< 3  (上に有界)
なので、単調増加な∫[0,R]f(x)dx は収束する。

ところで、f(x)って何?
0897132人目の素数さん垢版2018/01/17(水) 00:53:54.23ID:PuOM8/9R
x、yが実数であるとき、次のア〜ウのうち、真である命題をすべて挙げたものとして正
しいものを一つ選べ。
ア x>1 かつy>2 ならば、x+y>3 である。
イ x2−4x+3=0 ならば、x=1 である。
ウ xy=xzならば、y=zである。
1 ア 2 アとイ 3 イとウ 4 アとウ 5 アとイとウ
教えてください
0898132人目の素数さん垢版2018/01/17(水) 04:41:35.97ID:tAuiMZtD
>>811
今更だけど、>>787(>>789)は次のように書き直し。2)に致命的な間違いがあった。
-1<α、β<1 なる2解α、βって、仮定の段階で与えられていて既に存在している状態ではなかったのね。

よって、xy-平面上で、2次関数 y=ax^2-x-1 のグラフCの頂点は A(1/2a、-1/4a-1) で、Aのy座標は、y=-1/4a-1。
2次方程式 ax^2-x-1=0 の判別式をDは、D=1+4a。
1):a>0 のとき。このとき、D>0 だから、2次方程式 ax^2-x-1=0 の異なる2つの実数解α、βは確かに存在する。
そして、グラフCは下に凸となる。
x=1 とすると、y=a-2 だから、Cはxy-平面上の点 (1,a-2) を通る。
x=-1 とすると、y=a だから、同様に考えると、Cは点 (-1,a) を通る。
ここで、2解α、βについて、仮定から、-1<α,β<1。
また、2次多項式 ax^2、ax^2-x-1 について、x^2 の各項の単項式 ax^2 の各係数はaで同じだから、
2次関数 y=ax^2-x-1 のグラフCは2次関数 y=ax^2 のグラフを或る方向に平行移動して重ね合わせることが出来る。
従って、グラフCを描いて考えると(中間値の定理を使って考えると)、
-1<α,β<1 なることと、a-2>0、a>0、-1/4a-1<0 がすべて成り立つこととは同値である。
故に、aについて a-2>0、a>0、-1/4a-1<0。この連立不等式を解くと、aが存在し得る範囲は a>2。
これは、場合分けに当たり設定した条件 a>0 は満たす。故に、aの存在範囲は a>2。
2)、a<0 のとき。-1<α,β<1 なる解α、βがあるとする。
すると、α、βは2次方程式 ax^2-x-1=0 の異なる2解だから、D=1+4a>0、故に a>-1/4。
そして、1)と同様に考えると、-1<α,β<1 なることと、a-2<0、a<0、-1/4a-1>0 がすべて成り立つこととは同値だから、
仮定からaについて a-2<0、a<0、-1/4a-1>0。この連立不等式を解くと、aが存在し得る範囲は a<-1/4。
これは、場合分けに当たり設定した条件 a<0 を満たす。故に、a<-1/4 となる。
しかし、a>-1/4 と a<-1/4 とは相反し矛盾する。故に、背理法から、a<0 のとき -1<α,β<1 なる解α、βは存在しない。
1)、2)から、-1<α,β<1 となる条件は、a>2。
0899132人目の素数さん垢版2018/01/17(水) 05:19:35.15ID:PV3WiWDX
広義積分の存在で

f(x)が[a,∞)で連続とする。
|f(x)|≦M/x^λが成り立つようなM>0、λ>1が存在するならば、無限積分∫(a→∞)f(x)dxは存在する。


この定理を応用(?)すると
g(x)=M/x^λとして、|f(x)|≦g(x)で、∫g(x)が収束するならば、f(x)も収束すると考えてるんですが、間違ってますか?
0900132人目の素数さん垢版2018/01/17(水) 05:21:45.51ID:PV3WiWDX
∫g(x)→∫g(x)dx

f(x)も収束する→∫f(x)dxも収束
0901132人目の素数さん垢版2018/01/17(水) 06:28:13.66ID:YoDVUQg0
>>890
昔見たものだからよく覚えてないけど解析的性質が色々と成り立ってる事をチェックしたりしてた
0902132人目の素数さん垢版2018/01/17(水) 07:20:23.49ID:PV3WiWDX
>>890
> lim(x→0)(sinx)/x=1 を仮定せずに

屁理屈だけどさ。仮定しなきゃ良いだけなんだからlim(x→0)sinx/x=1を証明しちゃえば良いんでね?
0907132人目の素数さん垢版2018/01/17(水) 15:26:35.64ID:YoDVUQg0
(1)関数l(y)はなぜ[0,1]から[0,π/2]への全単射となるのか.
(2)“同様の操作”とはなんなのか.
(3)cosx,sinxはなぜ単位円上の点なのか.
(4)なぜsinxは0<x<π/2で微分可能なのか(逆関数の微分法はf,f^(-1)がともに連続であり,かつfの定義域と値域はともに開集合で,f'≠0でなければ適用できません.)

以上の(1)-(4)のことが説明できれば写真に載っていることは正しい
0908132人目の素数さん垢版2018/01/17(水) 17:42:28.97ID:7zXythQj
>890
cosはy=sinθのときのx座標でいいやろ
>907は(1)だけわかれば
(2)なし
(3)定義
(4)自明
ちなみにf^-1の連続性のチェックは不要
0909132人目の素数さん垢版2018/01/17(水) 17:58:55.76ID:PuOM8/9R
x、yが実数であるとき、次のア〜ウのうち、真である命題をすべて挙げたものとして正
しいものを一つ選べ。
ア x>1 かつy>2 ならば、x+y>3 である。
イ x2−4x+3=0 ならば、x=1 である。
ウ xy=xzならば、y=zである。
1 ア 2 アとイ 3 イとウ 4 アとウ 5 アとイとウ
教えてください
0911132人目の素数さん垢版2018/01/17(水) 19:07:05.37ID:mMOCfm1F
バカなのでわかりません。
10a/(a + 10) = 9.5 のときのaを求めます。
詳しく解いてください。
a = 190 です。
0912132人目の素数さん垢版2018/01/17(水) 20:49:40.32ID:hfgsLxjx
>>911
バカなのは問題にしない
礼儀だ、貴様に欠けているのは
だからこんな問題も解けんのじゃw
0913>>911垢版2018/01/17(水) 21:16:15.67ID:mMOCfm1F
別の問題を解いていたら同じ形になった。
わからない。
0914132人目の素数さん垢版2018/01/17(水) 21:16:57.71ID:9qRKZbfk
そんなに無礼か?
0915132人目の素数さん垢版2018/01/17(水) 21:40:45.35ID:PuOM8/9R
>>909
教えて下さいお願い致します
0917132人目の素数さん垢版2018/01/18(木) 00:09:29.54ID:pQ9CzRPh
>>909
2
0919132人目の素数さん垢版2018/01/18(木) 09:18:33.85ID:8rC65/QE
これらの意味について聞かれたのですが

https://i.imgur.com/wmRCBlV.jpg

まず、微分係数というものの定義を極限で与える。
y=f(x)が各点で微分係数を持つ時、f(x+Δx)-f(x)でΔyを定義します。lim(Δx→0)Δy/Δx=f'(x)より
Δy/Δx=f'(x)+ε (Δx→0⇒ε→0)と表せる。故にΔy=f'(x)Δx+εΔx となる。 ここでΔyの挙動を支配するのはf'(x)Δxであるから、この部分を特にdyで表してyの微分と定義する。f(x)=xとすれば直ちにdx=Δx が得られる。
結果としてdy=f'(x)dx が得られる。

てのが一応厳密な微分の定義の1つかな…
解析概論は確かこんな感じに話してた筈

てな訳でdxはx'Δx つまりはΔxと等しい。d/dxはただの記号(ちゃんと言うなら作用素?)
dy/dx はdy=f'(x)dxを形式的にdxで両辺を割って得られる形式的な式(f'(x)をこのように書く!と定義する本も多い)
Δx:ただの変化量

が答えになるのでしょうか?解析苦手民で間違えてるかもなので教えてください
0920132人目の素数さん垢版2018/01/18(木) 09:41:40.67ID:I6nQhhU/
>>919
1変数の実数関数なら適当に答えていいよ
dx=Δx
でいいし
0921132人目の素数さん垢版2018/01/18(木) 09:48:42.98ID:8rC65/QE
そうですね笑

私の解釈は合ってますか?
0922132人目の素数さん垢版2018/01/18(木) 10:19:40.79ID:+FYEpL0w
>>911
> 10a/(a + 10) = 9.5 のときのaを求めます。

10a/(a+10)=9.5=95/10=19/2

20a/(a+10)=19

20a=19(a+10)
20a=19a+190
20a-19a=190
a=190
0923132人目の素数さん垢版2018/01/18(木) 10:29:41.11ID:cqzkfcI1
>>918ありがとうございました
0924132人目の素数さん垢版2018/01/18(木) 10:32:18.30ID:cqzkfcI1
https://i.imgur.com/eb9ZZom.png

∠A=36゚、BC=2,AB=ACの二等辺三角形ABCがあり∠Bにおける角二等分線が辺ACと交わる点を点Dとするとき次の問にこたえよ
Cos72=あ
教えて下さい
0925132人目の素数さん垢版2018/01/18(木) 10:35:20.32ID:gMzlxS4N
>>919
dyをそのように解釈するならば、dyはxとdxの2変数関数として定義されているわけですから、dy/dxは形式的な割り算ではなくて、普通の割り算です
0927132人目の素数さん垢版2018/01/18(木) 12:25:42.96ID:cqzkfcI1
>>926
全部の辺はわかりました
0929>>911垢版2018/01/18(木) 13:11:25.47ID:6gtDKxCZ
>>922
10a/(a+10)=9.5=95/10=19/2 この項たしかにそうなりますね。
移項くらいしか技がないのでムズかしかったです。
ありがとうございました。
0931132人目の素数さん垢版2018/01/18(木) 15:00:33.70ID:I9qtByTh
ラグランジュ補間多項式において、
Pn(x)-P n-1(x) = bn (X-X0)(X-X1)・・・(X-Xn-1)
としたときのbnはどう表されるか
0932132人目の素数さん垢版2018/01/18(木) 16:24:52.08ID:3VMTZnuL
>>924 >>927

AB - CD = AC - CD = AD = DB = BC,
題意より
x - 4/x = 2,
これを解いて
x = 1 + √5,
第二余弦定理より
cos(72゚)=(2・2 + x・x - x・x)/(2・2・x)= 1/x =(√5 -1)/4 = 0.309017
0933132人目の素数さん垢版2018/01/18(木) 16:34:58.51ID:cqzkfcI1
>>10
志井和夫「どうだい?わが党員にならないかい?」
0934132人目の素数さん垢版2018/01/18(木) 17:02:08.18ID:siE9kQwG
尋常じゃないくらい頭が悪いのですが、東京大学理学部数学科に入りたいです。
どうすれば良いでしょうか?
ちなみに白チャートすらまったく理解できません。
0935132人目の素数さん垢版2018/01/18(木) 17:25:23.98ID:+FwP6CtR
位相の基礎的な話が完成するのに30年もかかったそうですが、
なぜそんなに時間がかかったのでしょうか?
0936132人目の素数さん垢版2018/01/18(木) 17:29:50.40ID:cqzkfcI1
>>932第二余弦定理より
cos(72゚)=(2・2 + x・x - x・x)/(2・2・x)
どんな公式でどんな風に当てはめたのですか?そんなのならってません
ありがとうございました
0937132人目の素数さん垢版2018/01/18(木) 18:45:37.96ID:siE9kQwG
切実な質問です。

高校数学の白チャートを読んでいるのですが、さっぱり分かりません。
何がいけないのでしょうか?
単に小学校の算数、中学数学を理解できていないから理解できないだけなのでしょうか?
それか、やはり自分が頭悪すぎだからなのでしょうか?
もし前者なら、もう一回、小学校の算数と中学数学を勉強し直せば、
高校数学も理解できるようになる可能性は少しはあるのでしょうか?
それとも後者もしくは前者と後者両方だから、キッパリと諦めるべきなのでしょうか?

誰か教えてください。
0938132人目の素数さん垢版2018/01/18(木) 18:56:53.87ID:8rC65/QE
aを実数とする。このとき、曲線y=e^xとy=(x-a)^2の両方に接する直線が存在するようなaの値の範囲を求めよ

すみません。この問題がいくら考えても分からないので方針だけでもいいので教えて下さい。
自分は、それぞれの曲線のs,tにおける接線を求めてなんかしようとしたのですが、結局何をしたいのかが分からなくなってしまいました。
答えは a≦2(log2−1) です。
0939132人目の素数さん垢版2018/01/18(木) 19:15:05.61ID:EIuESj3T
[2](2)の解答を教えてください。
ほぼ習ってない内容で全くわかりません
https://i.imgur.com/UXcICIs.jpg
0940132人目の素数さん垢版2018/01/18(木) 19:46:41.51ID:uA1NJQSm
>>938
一方が2次関数なので,接線の処理としては判別式を使いたいところ.
とすると,f(x)=e^xのx=tにおける接線y=[tとxの式]を考え,これと
y=(x-a)^2での連立から(x-a)^2-[tとxの式]=0
を考える.xの2次方程式よりこの判別式をDとしたとき,
D=0となるaとtが存在すればよい.あとは$a=[tの式]$くらいにして
y=aとy=[tの式]
の交点が存在することから,グラフ確認程度かな.
0941132人目の素数さん垢版2018/01/18(木) 20:04:53.79ID:8rC65/QE
曲線y=e^x上の点(t,e^t)における接線の方程式はy'=e^xよりy=e^t(x-t)+e^t=e^tx-(t-1)e^t

また曲線y=(x-a)^2上の点(s,(s-a)^2)における接線の方程式は
y'=2(x-a)よりy=2(s-a)(x-s)+(s-a)^2
=2(s-a)x-(s^2-a^2)…A

すると@とAが一致するとき
e^t=2(s-a)…B
-(t-1)e^t=-(s^2-a^2)
よって(t-1)e^t=(s-a)(s+a)…C
が成り立つ

するとBよりs=e^t/2+a
となるからこれをCに代入すると
(t-1)e^t=(e^t/2)(e^t/2+2a)
従ってt-1-e^t/4=a…D (0<e^tだから)

よってDを満たすようなtが存在するようなaの値を求めればよいことになる

従ってf(t)=t-1-e^t/4と置くと
f'(t)=-e^t/4+1となるので
f'(t)=0⇔e^t=4⇔t=log4=2log2

よってy=f(t)の増減とグラフの概形を書けば(省略)y=aと交点を持てばそのtがDを満たすことになる

ゆえに答えは前述のものとなる

1つだけ分からないのですが、f(t)をt→∞とした時∞−∞となってその不定形の解除のやり方が分からないです。多分−∞に発散するのは予想つくのですが…
0942132人目の素数さん垢版2018/01/18(木) 20:17:55.56ID:7/ryK/Yh
>>941
指数関数≫n次関数だから、この不定形は発散を既知として良いのでは。
それを示せという問題を除いて。

やるなら、

fが十分大きなtで単調減少であることを示す(減少しないことの簡単な記述)
さらに、f'が十分大きなtで単調減少であることを示す(収束しないことの簡単な記述)

でどう?
0943132人目の素数さん垢版2018/01/18(木) 20:20:38.83ID:uA1NJQSm
>>941
f(t)=e^t(t/(e^t) -1/(e^t)-1/4)
でどう?ただ,感覚的に,
e^tとtの速さはe^tの方が断然早いから
で良いと思うけどね.
0945132人目の素数さん垢版2018/01/18(木) 20:50:18.00ID:6oNojZJz
原点Oと直線x=lからの距離の比がe:1であるような点p軌跡を求めよ

点pの座標を(x,y)として関係式をたてたら
(1ーe^2)x^2+y^2+2lxe^2ー(el)^2=0
になって典型問題同様eの範囲で分類したのですがうまくいきませんでした。
どうすれば答えが得られますか?
0946132人目の素数さん垢版2018/01/18(木) 20:50:48.98ID:8rC65/QE
なるほどです。てっきり乗除の時だけその考え方が出来るのかと思ってました。勉強になりましたありがとうございます。
0947132人目の素数さん垢版2018/01/18(木) 21:09:41.65ID:BqMeO5gJ
>>945
その式が答えじゃだめなの?
分類で言うと、l≠0としてe<1が楕円、e=1が放物線、e>1が双曲線ってことになるけど
0948132人目の素数さん垢版2018/01/18(木) 21:25:27.08ID:zPOScIdY
Sを次で与えられる円筒とする。 {(x,y,z):x^(2)+y^(2)=4,0<=z<=4}
さらに(2,0,2)における単位法線ベクトルが(1,0,0)となるようなS上の連続的に変化する単位法線ベクトルの集合によって向きつけられているとする。この時f(x,y,z)=xyに対し面積分∫S f dAを計算せよ。
誰かこれ解いてくださいお願いします;;
0949132人目の素数さん垢版2018/01/18(木) 21:32:29.15ID:6oNojZJz
一応分母がぐちゃぐちゃになるのを除けば
楕円と双曲線の式は立ったのですがe=1のときだけy^2=1ー2lxになってy^2=4pxに出来ません
これ計算すれば成立したりするんですか・・・?
0951132人目の素数さん垢版2018/01/18(木) 21:59:13.18ID:6oNojZJz
移動した状態という可能性を完全に忘れてました
スレ汚してすいません
0954132人目の素数さん垢版2018/01/19(金) 01:36:49.66ID:4j7tlg3r
>>936

頂点Aから底辺BCに垂線AHを下ろす。
BH = CH = 1
cos(72゚)= BH/AB = 1/x = =(√5 -1)/4 = 0.309017
cos(36゚)= AB/(2BC)= x/4 =(√5 +1)/4 = 0.809017
0955132人目の素数さん垢版2018/01/19(金) 02:33:53.86ID:f3n/lm/W
https://i.imgur.com/x6zeRr3.jpg

どうやら一般的には置換積分で解く問題で
普通に簡単な方法で解くと上のような答えになりますが
これが採点者に x-log((e^x)+1)+C と同じ解と気付かれず0点になる可能性はありますか?
0956132人目の素数さん垢版2018/01/19(金) 09:25:04.45ID:c1VU5nlZ
文字が多いからダメ
0957132人目の素数さん垢版2018/01/19(金) 10:03:01.21ID:fdzpQ2yR
質問です。2次元座標上の座標(X、Y)から角度R方向へ距離3進む場合ってどんな式になるんでしょうか?
0960132人目の素数さん垢版2018/01/19(金) 13:06:40.35ID:0LyVXeRR
大至急です!
x.y.zのグラフにおいて線形独立だとそれらは平面上にあるってのは合ってますか?
よろしくお願いします!!
0961132人目の素数さん垢版2018/01/19(金) 13:18:17.95ID:c1VU5nlZ
アホすぎる
0962132人目の素数さん垢版2018/01/19(金) 13:23:29.30ID:BqcKMvTP
大至急ってのなら、問題を写真にとるくらいしようぜ。
何が言いたいのか全く分からんわ
超エスパーすると多分間違ってると思うけど、問題わからなすぎ。
0964132人目の素数さん垢版2018/01/19(金) 13:53:15.51ID:aRkS6GuN
画像は答えについてなんですが、
z=0だとなんで最後の答えになるのでしょうか?
https://i.imgur.com/sxBDObY.jpg
0966132人目の素数さん垢版2018/01/19(金) 14:31:26.00ID:BqcKMvTP
>>964
さっきのに比べたらまだエスパーしやすい

固有ベクトルは、z=0、x任意、y任意だから、z=0の条件で残るx,y成分を自由に動かせればいい。
ので、適当にz=0だけ満たした線形独立なベクトルを2本用意すればよくて、
なんでもいいから(1,0,0)と(0,1,0)を選んだってこと。
(1,0,0)と(0,1,0)である必要はなくて、例えば(1,0,0)と(1,1,0)とか(3,5,0)と(1,8,0)とかでも別にかまわない。

x,y任意、z=0を満たす平面を表すベクトルを表すといえば分ってもらえるのかな。
半年くらい前に授業でやったはずの、核とか像とか線型部分空間とか思い出せばなんかわかるかも。
0967132人目の素数さん垢版2018/01/19(金) 14:39:12.78ID:hHDLcllb
目一杯忖度してみる
ベクトル(x y z)のzが0だと言ってるのだから代入したら(x y 0)となる
(x y 0)には変数が2つ含まれ、線形独立な2つのベクトルの線形結合で表すことができる
このとき、線形独立な2つのベクトルは、z=0になるように取る。取り方は無数にあるが、(1 0 0)と(0 1 0)を使えば話は簡単だろう
つまり(x y 0)は(1 0 0)と(0 1 0)の線形結合h1(1 0 0)+h2(0 1 0)で表すことができる。
この式は(h1 h2 0)に等しいからh1=x,h2=yとすれば(x y 0)と等しいことがわかるだろう
0969132人目の素数さん垢版2018/01/19(金) 17:58:42.44ID:0LyVXeRR
ありがとうございます!理解できました!
あと、ついでと言ったらなんですが...
アクチュアリー目指してる人やアクチュアリーの人いますか?数学科からなる職として割と多いらしく、居たら少し話を聞きたいです

どんな仕事かは調べたり少し話に聞いたけど、実際勤めてる人が具体的にどんなことをしてるのか知りたいのと、

アクチュアリー会の雰囲気とか、入った方がいいのかとかを知りたいのと、

試験勉強はまず何から手をつけていいのかもよくわからないので、そういうことも知りたいです。

というか実際何もわかってないんで何か知ってたら教えてほしいって感じです。
ネットで調べてもなんか情報がフワフワしてるので…
0970132人目の素数さん垢版2018/01/19(金) 20:25:17.19ID:+q0UCBP5
いないよ
じゃあね
0971132人目の素数さん垢版2018/01/19(金) 21:18:51.76ID:4u7vwxRW
金融他業種だけど、小難しい数学を使うというよりは、基本的な統計学一通りと、会計・保険業法の知識が要るんではないかと思われる(本業じゃないから推測ね)。
アクチュアリー協会が過去問出してるからとりあえず通販で買ったら?
それすらおっくうであれば、別の進路考えてもいいと思う。数学科出てることが不利になる業界は基本的にはないはずだし、今景気良いから興味ある分野に飛び込むのも十分有りかと。
0972132人目の素数さん垢版2018/01/19(金) 22:10:18.66ID:Zv+fnsLu
cosA+cosB+cosC(三角形の内角)の最大値を加法定理を用いずに求めよ。
ただしsincosカーブの凸性を認めるとする。

この問題教えてください...
0973132人目の素数さん垢版2018/01/19(金) 23:31:19.64ID:4u7vwxRW
少なくとも2つの角は90°未満なのでそれをA,Bとする
Cを固定(∴A+Bを固定)すると
cosカーブが0°以上90°以下で上に凸より
(1/2)*cosA+(1/2)*cosB≦cos((A+B)/2)
等号はA=B=(A+B)/2 のとき成立
よって 2cosA+cosC の最大を求めればよい
今Cも90°未満とすると、再び凸性より
2cosA+cosC=3*((2/3)*cosA+(1/3)*cosC)
≦3*cos((2/3)A+(1/3)C)=3*cos60°=3/2
C>90°のときは、最大でも√2未満にしかならない
0975132人目の素数さん垢版2018/01/20(土) 00:17:06.77ID:JK2wBsnV
小学生の問題だな。
0976132人目の素数さん垢版2018/01/20(土) 00:24:21.48ID:74immcem
>>974
難問か?
字は読みづらいけど・・・
PQAの面積なら、Aを少し下にずらして、PQAを直角三角形にすれば終わりだろ
PQAとDとの距離を求められるかと言えば、A,P,Qを通る平面で切断すれば、Dと平面APQ
の距離は求まるだろう
0977132人目の素数さん垢版2018/01/20(土) 00:26:04.09ID:bEiI/N73
面PADに対して平行に Qの所に同じ三角形作ったら行けますかね
0978132人目の素数さん垢版2018/01/20(土) 00:45:12.48ID:JK2wBsnV
中学入試でも易しい部類だな。
0979132人目の素数さん垢版2018/01/20(土) 00:59:15.05ID:74immcem
>>977
ちょっと真面目に返事する
まず、何を求めたいの?
△PQAの面積を求めたいなら
PQ‖AEだから、AEの中点をMとすると、△PQA=△PQX
PX⊥PQだから、△PQX = PX x PQ / 2

>面PADに対して平行に Qの所に(以下略)
って、何が行けるの?
0980132人目の素数さん垢版2018/01/20(土) 01:28:37.79ID:zeJuZjrp
>>974
問2は線分PQを伸ばして辺BCとの交点を考えた方が筋がいい
交点から更に頂点AとDへ補助線を引く
0981132人目の素数さん垢版2018/01/20(土) 03:10:51.46ID:yx33Mqat
ああああああああああああああああああああああああああああああああああああああああああああああああああああああああああ
ああああああああああああああああああああああああああああああああああああああああああああああああああああああああああああああああああああ
ああああああああああああああああああああああああああああああああああああああああああぁぁぁぁぁぁぁぁぁぁぁぁぁぁぁぁぁぁぁぁぁぁぁぁぁぁぁぁぁぁぁぁぁぁぁぁぁぁ
ぁぁぁぁぁぁぁぁぁぁぁぁぁぁぁぁぁぁぁぁぁぁぁぁぁぁぁぁぁぁぁぁぁぁぁぁぁぁぁぁぁぁぁぁぁぁぁぁぁぁぁぁぁぁぁぁぁぁぁぁぁぁぁぁぁぁぁぁぁぁぁ
ぁぁぁぁぁぁぁぁぁぁぁぁぁぁぁぁぁぁぁぁぁぁぁぁぁぁぁぁぁぁぁぁぁぁぁ 👀
Rock54: Caution(BBR-MD5:0be15ced7fbdb9fdb4d0ce1929c1b82f)
0982132人目の素数さん垢版2018/01/20(土) 03:38:44.81ID:bEiI/N73
>>973
>cosカーブが0°以上90°以下で上に凸より
(1/2)*cosA+(1/2)*cosB≦cos((A+B)/2)

私も,当然だ(自明)と思います。が
これは厳密には証明する必要があります。

>C>90°のときは、最大でも√2未満にしかならない

根拠は?
0983132人目の素数さん垢版2018/01/20(土) 09:25:09.58ID:3l6AVmW2
凸性を仮定して、と問題文にあるので。
C>90°なら、A=B=45°-C/2よりcosA<√2/2,cosC<0
0984132人目の素数さん垢版2018/01/20(土) 09:29:20.53ID:3l6AVmW2
あーごめん、鈍角の可能性除くのはもうちょっと吟味要るね
0985132人目の素数さん垢版2018/01/20(土) 09:30:35.50ID:2JsoyV0W
1/4a-1/2b+4=0.16a+4b+4=0をとけ教えて下さい
0988132人目の素数さん垢版2018/01/20(土) 10:12:51.90ID:fdRXR8NV
束論はなぜ廃れたのでしょうか?
0990132人目の素数さん垢版2018/01/20(土) 10:20:08.84ID:dD6Obrd8
正方行列Aについて
Aの最小多項式が重根をもたないならばAは対角化可能である

ことを証明して下さい 逆は不要です
0991132人目の素数さん垢版2018/01/20(土) 12:28:56.01ID:gB7kB20f
Aは次数と同じ数の固有ベクトルを持つので、これらを並べたPを使って
A P = P * 対角行列 と表せる。
Pは逆行列を持つのでAは対角化可能。
0992132人目の素数さん垢版2018/01/20(土) 12:37:47.33ID:dD6Obrd8
>>991
Aの行(列)の数と同じだけの数の固有ベクトルがとれたらAが対角化可能なのは当たり前です
それを証明して下さい
0995132人目の素数さん垢版2018/01/20(土) 13:25:23.65ID:2JsoyV0W
>>987
意味わかりませんどことですか?
0996132人目の素数さん垢版2018/01/20(土) 13:33:36.21ID:BdHhmenA
>>989

2xx-3x-2 = (2x+1)(x-2),

A ={x|x<-1/2 or 2<x},

B ={x|-1<x<a},

0<a≦2 のとき A∩B ={x|-1<x<-1/2},
A∩B∩Z = φ

a>2 のとき A∩B ={x|-1<x<-1/2 or 2<x<a},
a≦3 のとき A∩B∩Z = φ
a>3 のとき A∩B∩Z ={x∈Z|3≦x<a}

よって 3<a≦4
0998132人目の素数さん垢版2018/01/20(土) 13:46:33.90ID:gB7kB20f
まず、異なる固有値に対応する固有ベクトル同士は一次独立。
例えば
Φ(x) = (x-1)^2(x-2)
φ(x) = (x-1)(x-2) として
固有値1に対応する固有空間の次元は2で固有ベクトルが2つ取れる
というのは自明ではないのか。
0999132人目の素数さん垢版2018/01/20(土) 14:09:26.77ID:74immcem
普通の教科書に載ってるようなことを証明してくださいとか
上から目線でいうやつに敢えてかまう必要もないと思うぞ

何人かたむろしてる荒らしのうちの一人だろ
10011001垢版Over 1000Thread
このスレッドは1000を超えました。
新しいスレッドを立ててください。
life time: 29日 23時間 1分 12秒
10021002垢版Over 1000Thread
5ちゃんねるの運営はプレミアム会員の皆さまに支えられています。
運営にご協力お願いいたします。


───────────────────
《プレミアム会員の主な特典》
★ 5ちゃんねる専用ブラウザからの広告除去
★ 5ちゃんねるの過去ログを取得
★ 書き込み規制の緩和
───────────────────

会員登録には個人情報は一切必要ありません。
月300円から匿名でご購入いただけます。

▼ プレミアム会員登録はこちら ▼
https://premium.5ch.net/

▼ 浪人ログインはこちら ▼
https://login.5ch.net/login.php
レス数が1000を超えています。これ以上書き込みはできません。

ニューススポーツなんでも実況